REVIEW NINJA MCQ BEFORE REG, CPA REG - Business Law Section, SuperFast CPA REG, SuperFast CPA REG, CPA Exam Regulation - Individual Taxation, REG CPA

Pataasin ang iyong marka sa homework at exams ngayon gamit ang Quizwiz!

The penalty for late filing of a partnership return is:

$195 per partner for each month, or part of a month, that the return is late, up to 12 months.

The maximum deduction on interest paid during the current year on a qualified education loan is

$2,500. A taxpayer with a qualified education loan can deduct the loan's interest, provided the student loan was taken out solely to pay qualified education expenses and was not from a person related to the taxpayer or made under a qualified employer plan. The student must be the taxpayer, taxpayer's spouse, or taxpayer's dependent, and the student must be enrolled at least half-time in a program leading to a degree, certificate, or other recognized educational credential at an eligible educational institution.

There are four elections that are made at the partnership level:

*Taxable year* and *accounting method* *Cost recovery* methods and assumptions Treatment of *research and development costs* Amortization of *organization costs* and *start-up costs*

Decreases to AAA:

-Corporate distributions (may not reduce AAA below 0) -Separately and non separately stated expense items and losses - Nondeductible expenses that relate to income other than tax exempt income

1. Nonliquidating distribution (withdrawal) - Basis used - Stopping point 2. Liquidating - Basis used - Stopping point

1. - NBV asset taken - Stop at zero 2. - Partnership interest - Must zero out account

Corporation liquidation=: Corporation distributes assets to shareholders 1. Corporation 2. Shareholder

1. FMV < Basis > = Taxable gain/loss 2. FMV < Stock basis > = Taxable gain/loss

1. Gift taxes paid on a gift 2. Estate taxes paid on inherited property

1. Affect basis of gifted property to the donee 2. Never affect basis of inherited property to the beneficiary

Guaranteed payments to partner 1. Partnership 2. Partner

1. Allowable tax deductions 2. Self-employment income

Priority ranking: conflicting interests in collateral

1. Buyer in ordinary course of business of inventory that servers as collateral for security agreement created by the seller; holders in due course of negotiable instruments; holders of possessory liens 2. Holder of properly perfected PMSI in collateral 3. Holder of perfected security interest in or judicial lien attached to collateral (lien of a trustee in bankruptcy) 4. Holder of unperfected security interest in the collateral (Attach only) 5. The Debtor

Illegality 1. Failure to have license required to protect public (CPA, attorney, doctor) 2. License required merely to raise revenue

1. Contract is void 2. Enforceable

Statute of limitations: which contracts must be in writing? MY LEGS

1. Contracts where consideration is MARRIAGE 2. Contracts by which terms can't be performed within a YEAR 3. Contracts involving interests in LAND 4. Contracts by EXECUTORS to pay estate debts out of personal funds 5. Contracts for the sale of GOODS for 500 or more 6. Contracts to act as SURETY (pay the debt of another)

Termination of actual authority automatically by operation of law upon any of following events:

1. Death of either principal or agent 2. Incapacity of the principal 3. Discharge in bankruptcy of the principal 4. Failure to acquire a necessary license 5. Destruction of the subject matter of the agency 6. Subsequent illegality

Chapter 7 and 11: Voluntary cases

1. Debtor files a petition for relief 2. Debtor need not be insolvent but must pass income tests 3. Spouses may file jointly

1. Charity 2. Gifts 3. Political Contributions

1. Deductible 2. Not deductible 3. Not deductible

Proving negligence: elements

1. Duty of care 2. Breach 3. Causality 4. Injury

Spouses: Homeowner's exclusion 1. Ownership requirement 2. Use

1. Either must meet 2. Both must meet. If one spouse doesn't qualify for use requirement, one spouse may still take 250,000 exclusion

Employer liability: 1. Employee's negligence 2. Intentional torts

1. Employer liable 2. Not liable: not within scope of employemtn

Intrastate sales: 1. Rule 147 2. Rule 147A

1. Entire issue must be offered and sold only to residents of that state; issuer must do at least 80% of business in that state, purchasers can't resell the securities for 6 months to nonresidents of that state 2. Not state resident; internet is okay

Individuals typically cash basis. In order to be tax deductible, item must have been:

1. Incurred as an expense 2. Paid or charged before year end

Involuntary conversion: 1. Realized gain exists 2. Recognized gain 3. Basis is 4. Losses 5. When loss, basis is 6. Holding period includes

1. Insurance proceeds > adjusted basis in the converted property= realized gain 2. Insurance proceeds - amount reinvested Realized gain - recognized gain= gain not recognized 3. Cost of replacement property - any gain not recognized 4. Are recognized 5. Replacement cost 6. Period original property was held

Boot received 1. Gain recognized 2. New basis

1. Lesser of realized gain or boot received 2. Adjusted basis of property given up + Gain recognized - boot received

Reorganizations: Corporation is tax payer 1. Event 2. Income 3. Basis 4. Tax attributes

1. Nontaxable 2. None 3. NBV 4. No change

Firm offer rules

1. Seller must be a merchant 2. Offer must be in writing and signed by the merchant 3. Offer must give assurances it will be kept open for a certain time

Realized loss recognized unless: WRaP

1. Wash sale losses 2. Related party losses 3. And 4. Personal Losses

Net operating losses 1. Offset income? 2. Carryback 3. Carryforward

1. Yes 2. Two years 3. 20 years

Corporation 1. Limited liability 2. Entity taxation 3. Owner taxation 4. Suitable for IPO

1. Yes 2. On earnings 3. On dividends 4 Yes

Business losses or casualty losses related to business

100% deductible

Large corporation must pay

100% of tax shown on current year return

how do 1231 gains and losses work?

1231 gains are carried back against 1231 losses and ordinary income is 'recaptured'. Gains go back 5 years.

what is section 1250 property?

1250 property includes business land and most real property that is subject to depreciation

Rented > 15 days and used for personal greater of:

14 days or 10% of rental days: PERSONAL/RENTAL RESIDENCE. *Rental expenses deductible to extent of rental income *No rental loss allowed *Expenses prorated. - Taxes and interest amount x (rental months/12) - Utilities and depreciation amount x (rental months/total use months*rental+personal*)

If an individual rents a personal residence for a period of time less than _____ days during a tax year, the rents he or she receives are not included in gross income and the associated expenses are not deductible as a rental expense.

15

what time period are section 197 items amortized over?

15 years

Employers with quarterly payrolls of at least ________ or who employ at least __________ must participate in FUTA

1500; one person for 20 weeks in a year

For AGI adjustment for student loan interest

2500

CPA must return a completed copy of each return for

3 years after the close of the return period

Farm income averaging

3 years- schedule J

According to the Securities Act of 1933, no sales activity is allowed within ________ days before registration

30

how much of a capital loss can be deducted in one year for individuals?

3000- anything more than that is carried forward as a STCL.

Securities issued under Rule 506, for private offerings, may be sold to how many unaccredited but sophisticated investors?

35 or fewer

what is the MACRS life of a non-residential real property such as an office building?

39 years. MACRS requires that such a building is depreciated on the straight line bassis.BUT- the mid-month convention applies to this type of property which means no matter when it is placed into service it is considered to be mid-month.

if you (an individual) have ST cap losses of 10k and LT cap losses of 10k how much can you deduct of each one in the current year?

3k can be deducted, and the ST losses are deducted first.

Form 8-K must be filed within _________ days after a major event/change in the company

4

What is the statute of limitations under the UCC?

4 years from breach

what is the typical length of the statute of limitations on a contract?

4-6 years (from date of breach)

if you incorporate your business and you put all assets into the corp in exchange for stock, what % do you have to own immediately afterwards to qualify as tax free?

80%

the 1933 securities act applies to sales of securities, including stocks, bonds and notes that are issued for periods over ___________

9 months

Domestic production deduction: amount

9% of lesser of: 1. Qualified production activities income 2. Taxable income (disregarding QPA deduction)

Which of the following is not a required element of a "fraudulent transfer" under the Bankruptcy Code? A. The transaction must have occurred within two years of the date of bankruptcy. Correct B. The purchaser must have been aware of the insolvency of the transferor. C. The asset must have been transferred for less than fair value. D. The transferor must have been insolvent at the time of the transfer.

A "fraudulent transfer" is a transfer made by a debtor within two years prior to the date of bankruptcy in which the debtor did not receive fair value in exchange for the asset. There is no requirement that the transferee be aware of the insolvency of the transferor.

A CPA firm must do which of the following before it can participate in the preparation of an audit report of a company registered with the Securities and Exchange Commission (SEC)?

A CPA firm must register with the Public Company Accounting Oversight Board (PCAOB) before it can prepare an audit report. The PCAOB was created to oversee the auditors of public companies in order to protect the interests of companies registered with the Securities and Exchange Commission.

A trust in which the beneficiaries are given a future right to trust income or corpus and the $14,000 gift tax exclusion is retained is termed a:

A Crummey trust is a "safe harbor" rule that allows the annual gift tax exclusion on gifts to a trust. The gift of a future interest provision does not apply to gifts to a Crummey trust if trust assets go to the beneficiaries on or before age 21. Giving the right to the trust income (generally only up to $14,000 per donee, per year) meets the requirement even though the beneficiaries do not actually withdraw the funds.

Describe a Roth IRA.

A Roth IRA has the following characteristics: Contributions are not deductible Earnings accumulate tax-free Qualified distributions are tax-free Contribution limits are: $5,000/$10,000 after taking into account contributions to all other IRA accounts

What is the tax treatment of a business loss?

A business loss may be deducted against other sources of income. When the loss exceeds the amount of income, the excess is a net operating loss. A net operating loss is then permitted to be carried over - either forward or back. 2-year carryback 20-year carryforward

PMSI loses: garage sale rule

A buyer of consumer goods resells the goods to another customer buyer, secondhand buyer will take free of an automatically perfected PMSI in consumer goods as long as secondhand buyer HAD NO NOTICE OF SECURITY INTEREST. **If secured party filed to perfect, secondhand buyer is subject to the security interest. Secured party can repossess from secondhand purchases because secondhand purchaser had notice

List the requirements for a Health Savings account deduction.

A health savings account enables workers with high-deductible health insurance to make pre-tax contributions of up to $3,050 to cover costs. No contributions are allowed once a taxpayer becomes covered by Medicare Parts A or B. Any amount paid or distributed used to pay the qualified medical expense of an account holder is excluded from income. If not used for qualified expenses: it is includible in income and subject to a 10% penalty.

Noll gives Carr a written power of attorney. Which of the following statements is correct regarding this power of attorney?

A power of attorney delegates authority from the principal (Noll) to the agent (Carr). This authority may be general or it may be limited or specific. It must be signed only by the principal (the agent need not sign the power of attorney), and it has force and effect for an indefinite time, unless otherwise stated, but will not be effective after the death of the principal. (A Last Will and Testament is required or some other testamentary document such as a trust is required for disposition after death.)

Can a married dependent be claimed as an exemption?

A taxpayer will lose the exemption for a married dependent who files a joint return unless the return is filed solely for a refund of all taxes paid or withheld for the year. Note: Persons claimed as dependents on another's tax return will not be allowed a personal exemption on their own return.

What is the definition of a writ of certiorari?

A writ of certiorari is a petition from the taxpayer or government asking the Supreme Court to review the decision of a lower court. In a tax case, such a decision will usually involve a case in which the Courts of Appeals have issued conflicting opinions about the case, or a case that concerns a large number of taxpayers or a large amount of tax revenue

Which of the following types of debtors are not eligible for relief under Chapter 11 of the Bankruptcy Code?

A. Individuals Incorrect B. Railroads C. Airlines D. Stockbrokers Chapter 11 of the U.S. Bankruptcy Code is available to every business, whether organized as a corporation, partnership, or sole proprietorship, and to individuals. This saves businesses from being liquidated. However, a stockbroker or a commodity broker is prohibited from filing under Chapter 11 and is restricted to Chapter 7 bankruptcy.

Additional Deduction: 65 or older/and or blindness A. 1 qualified 1. Single: 65 or blind 2. Single: Both 65 and blind 3. Married: 65 or blind 4. Married: Both 65 and blind B. 2 qualified 1. Each 65 or blind 2. Both 65 and blind

A. 1 qualified: 1. 1550 2. 3100 3. 1250 4. 2500 B. 2 qualified 1. 2500 2. 5000

Business casualty losses: Loss A. partially destroyed B. Fully destroyed

A. Limited to lesser of 1. Decline in value of property (Change in FMV of property) 2. Adjusted basis of property immediately before casualty B. Amount of loss =adjusted basis of property **Less any insurance proceeds

Both parties to a contract mistaken as to material fact regarding the contract

Adversely affected party can AVOID the contract. Contract is void **Doesn't apply to mistakes as to value, value is a matter of opinion

Innocent misrepresentation

All elements of fraud except scienter. Makes contract voidable by party who relied on the misrepresentation

What are the exceptions to the passive activity loss rules?

An taxpayer may deduct rental activity losses if either of the following two conditions are met: Mom and Pop exception - deduct up to $25,000 (per year) of losses attributable to rental real estate annually if the individuals are actively participating/managing, and own more than 10% of the rental activity. Real estate professional - More than 50% of the taxpayer's personal services during the year are performed in real property businesses, and the taxpayer performs more than 750 hours of services.

Allowable organizational expenditures for partnerships:

Deduct up to 5000. Each 5000 amount reduced by the amount > 50000 Includes: Fees paid for legal services in drafting the partnership agreement, fees paid for accounting services, fees paid for partnership filing

Nonqualified stock options: employer taxation

Deduct value of the stock option as a business expense in the same year the employee is required to recognize option as ordinary income

Expenses from hobby

Deducted to extent of gross income from hobby. Taken as itemized deductions: (In this order) 1. Mortgage interest and property taxes 2. All other hobby expenses except depreciation expense deductible as miscellaneous itemized deductions 3. Depreciation last as itemized deductions

Work uniform for taxpayer and employees

Deductible business expense schedule C

Artwork donated to local museum

Deductible to amount of basis Unless held for > 1 year: then FMV can be deducted

Investment interest expense

Deduction from AGI

Partnership may elect to have a tax year other than generally required tax year if:

Deferral period for the tax year elected is not more than 3 months

Minors ratifying contracts

Can't do it until they reach majority age

DNI: capital gains and losses Estates & trusts

Classified as principal and must remain with estate or trust to be taxed at the estate or trust level. Not included in DNI calculation

Which of the following minerals do not have a percentage depletion rate of 15% for mining in the United States?

Clay mining deposits can be written off using a percentage depletion rate of either 5% or 7.5%, depending on the end use of the clay. Gold, silver, copper, and iron ore mining use the 15% depletion rate for the cost recovery of the capital investment in the mining properties

When a security interest in collateral is perfected and the collateral is subsequently moved to another state

Collateral is temporarily perfected for 4 months in the state in which the collateral is moved

All aspects of the environmental audit program should be reviewed at appropriate intervals to assure that they are implemented as specified in the formal operating procedures. Which item should be reviewed at the end of each audit? A. Completion and quality of working papers Incorrect B. System used for selecting subjects C. System used for determining locations of audits D. System used for determining time period covered by the audits

Completion and quality of working papers should be reviewed at the end of each audit. Reviewing the systems for selecting subjects, determining locations of audits, and the time period covered by the audit may require only annual review.

GR: all offers can be revoked. Exception:

Consideration is paid to keep the offer open (option contract). If consideration was not given to keep the offer open, it is not an option

Personal holding company tax

Corporations set up by high bracket taxpayers to channel investment income into a corporation and shelter that income through low normal tax of the corporation instead of paying higher individual tax rates on that income Taxed an additional 20% on personal holding company net income not distributed

% of completion revenue recognition

Cost incurred/total expected cost = work done/total work= % earned x profit

Worthless stock and securities

Cost is treated as a capital loss, as if sold on last day of taxable year in which they became worthless

Foreign Tax credit

Credit or deduct as itemized deduction. No limit on deduction, but credits limited to the lesser of 1. Foreign taxes paid 2. Taxable income from all foreign operations/(taxable income + exemptions) x US Tax liability before credit on worldwide taxable income

What is the order of distribution of assets in a partnership at liquidation?

Creditors, Partners, Profits/Losses

Excess IRA contributions

Cumulative 6% excise tax each year until excess is corrected

Which of the following statements correctly describes the funding of noncontributory pension plans? Incorrect A. All of the funds are provided by the employees. B. All of the funds are provided by the employer. C. The employer and employee each provide 50% of the funds. D. The employer provides 90% of the funds, and each employee contributes 10%.

"Noncontributory pension plans" are those retirement plans in which all of the funds are provided by the employer. The employee does not contribute.

are medical expenses that you paid with a credit card deductible when you pay them or when you pay off the credit card?

they are deductible in the year you paid the medical expenses, not when you pay off the credit card.

how do capital losses work for corporations?

they can be carried back 3 years and forward 5 years but they can only reduce other capital gains. they do not lower taxable income

Absent agreement, when does the title pass?

upon completion of delivery

Can spouses file voluntary bankruptcy jointly?

yes

Does a general disclaimer, such as "goods are sold as is", disclaim implied warranty of merchantability?

yes

Is promising to ship an item, or just promptly shipping the item, considered to be acceptance in a UCC contract?

yes

Is the surety's own incapacity or bankruptcy a valid defense?

yes

Is there a trustee appointed for CH 13 Bankruptcy?

yes

what is the standard deduction for a trust or an estate in the fiduciary income tax return?

zero

Minority

Disaffirm contract anytime while a minor. Voidable Adult doesn't have right to rescind merely because minor may disaffirm

Acceptances are effective when

Dispatched (mailed, emailed, faxed, etc): Mailbox rule

What are the four fundamental corporate changes that require shareholder approval? (DAMS)

Dissolution, Amendments to articles of incorporation, Mergers, Sale of substantially all corporate assets

Trust may deduct amounts

Distributed to beneficiary up to the DNI (less adjusted tax-exempt interest)

Capital gains distribution

Distributions by a corporation with no earnings and profits, and shareholder has recovered entire basis. Taxable GI

What types of dividends do not receive favorable tax treatment?

Dividends received from the following do not receive a favorable tax rate: Regulated investment companies Real estate investment trusts Employer stock held by ESOP Short sale positions Certain foreign corporations

State and local income taxes and state and local general sales taxes

Elect to deduct one or the other

Meals and lodging furnished by the employer

Employee doesn't include in income if furnished for convenience of the employer on the employer's premises. Lodging must be required as a condition of employment

Work opportunity credit

Employers who hire employees from a targeted group 40% of first 6000 first year's wages per employee 40% of 3000 to certain summer youth

UCC: modifications to the contract

Enforceable without consideration

Dividends received deduction not limited to taxable income when

Entire DOD creates an operating loss

How does a noncorporate shareholder treat the gain on redemption of stock that qualifies as a partial liquidation of the distributing corporation?

Entirely as a capital gain

Employee service discounts

Excludable discount limited to 20% FMV of services. Excess reported as income

Employee Merchandise Discount

Excludable discount limited to employer's gross profit %. Excess reported as income

Moving deduction: Employer reimbursements

Excludable from income to extent amounts qualify as deductions

Life insurance proceeds because of death of the insured- beneficiary

Exclude from GI

Medicare benefits received under SS Act

Exclude from GI

Employees at educational institutions studying at the undergraduate level who receive tuition reductions A. Graduate students

Exclude tuition reduction from GI A. Receive reduction only if engaged in teaching or research activities and only if tuition reduction is in addition to pay for teaching or research **Reductions must be on nondiscriminatory basis to be deductible

Stock split

Excluded from GI. Shareholder allocates original basis over total number of shares after split.

Explain how passive activities are deducted.

Expenses related to passive activities can be deducted only to the extent of income from all passive activities. A net passive activity loss may not be deducted against wages, salaries, and other active income or against portfolio or capital gains income.

Tuition and fees deduction: qualified higher education expenses

Expired 12/31/2016 4000

If artwork held for > 1 year

FMV can be deducted

establishes a state-run system of insurance to provide income to workers who have lost their jobs

FUTA (Federal Unemployment Tax Act)

Mars, Inc., manufactures and sells DVD players on credit directly to wholesalers, retailers, and consumers. Mars can perfect its security interest in the DVD players it sells without having to file a financing statement or take possession of the DVD players if the sale is made to: A. retailers. B. wholesalers that sell to distributors for resale. Correct C. consumers. D. wholesalers that sell to buyers in the ordinary course of business.

For consumer goods, a security agreement is automatically perfected upon attachment if the purchaser buys on credit or the secured party lends to the consumer the funds to make the purchase. A financing statement or possession of the DVD players is not required.

Explain the Homeowner's Exclusion.

Gain Exclusion for Personal Residence: $250,000 for single taxpayers $500,000 for married taxpayers To qualify for the exclusion: Taxpayer must have owned and used the property as principal residence for two years or more during the five year period ending on the date of sale or exchange. Either spouse for a joint return must meet the ownership test. Both spouses must meet the use requirement with respect to the property. The is no age limit. No rollover to another house is required. The exclusion is renewable.

Describe the netting procedures for short and long-term capital gains and losses.

Gain and losses are netted within each tax group. Short-term gains and losses: If there are any short-term losses (this includes loss carryovers), they are first offset against any short-term gains that would be taxable at the ordinary income rates. Any remaining loss would be used to offset any long-term capital gains at the 28% rate group. Then offset any long-term gains from the 25% group. Finally, they offset any long-term gains at the 15% tax rate. Long-term gains and losses: Any long-term losses from the 28% rate group will offset any net gains from the 25% rate group then the 15% rate group. Any remaining losses from the 15% rate group will offset any net gains from the 28% rate group and then offset gains from the 25% rate group.

A has a basis of $6,000 for a 1/3rd interest in ABC partnership. A sells his interest for $14,000. What should A report as a gain on the sale?

Gain is recognized if cash distributed is in excess of the partner's basis in the partnership interest. The sale for $14,000 results in a gain of $8,000 and a tax-free return of capital of $6,000. An investment in a partnership is a capital asset that gives rise to a capital gain.

Discuss Involuntary Conversions of property.

Gain may be deferred if insurance proceeds are reinvested in property that is similar or related in service or use within 2 years for personal property or 3 years for business property. A realized gain exists when insurance proceeds are greater than the adjusted basis of the converted property. Note the difference between realized gain and recognized gain: Gain not recognized if proceeds reinvested in qualified replacement property Basis is cost of replacement property less any gain not recognized. Losses recognized and basis is replacement cost. Holding period includes period that original property was held.

Partnership: excess cash

Gain recognized to partner only to extent that cash distributed > adjusted basis of partner's interest in the partnership Cash < Basis > = Gain

Sale of partnership interest: usually capital. Exception: (ordinary income, not capital gain)

Gain that represents partner's share of hot assets: - Unrealized receivables if exchanged for cash - Appreciated inventory (as if exchanged for cash) - Recapture income regarding depreciable assets owned by the partnership

Sale of partnership interest: G/L on transfer=

Gain/loss= Amount realized for the sale - adjusted basis of the partnership interest Beginning capital account % of income < loss > up to sale = Capital account at sale date percentage of liabilities =Adjusted basis < Amount received > = Gain or loss Amount received includes: Cash, assumption of liabilities, FMV of property

List those miscellaneous expenses not subject to the 2% AGI floor.

Gambling losses Federal estate tax paid on income in respect to a decedent

Explain the exemption rule for children of divorced parents.

General Rule: The parent who has custody of the child for the greater part of the year takes the exemption. (determined by the time test, not the divorce decree). It does not matter whether the parent actually provided more than one-half of the child support. If the parents have equal custody, the parent with the higher adjusted gross income will claim the exemption. Exception: A noncustodial parent is not allowed a dependency exemption based solely upon a divorce decree. The custodial parent may revoke the release of the exemption claim.

What are the rules for Inherited Property Basis?

General Rule: Date of death FMV becomes basis. Alternate Valuation date: if elected by the executor, the FMV on the alternate valuation date may be used to value all of the estate property. (the earlier of 6 months later or the date of distribution/sale) Property acquired from a decedent is automatically considered to be long-tern property regardless of how long it actually has been held.

Give some examples of taxable and nontaxable interest.

General Rule: all interest is taxable. (Unless specifically excluded) Taxable - Federal bonds, Industrial Development bonds, Corporate bonds, Premiums from opening savings account, Part of proceeds from installment sales, Interest paid on late payment of federal and state refunds. Non-taxable - State and local government bonds, Bonds of a US possession, Series EE bonds (when used for education or taxpayer was over 24 years old when received).

Business entities can make a distribution of appreciated property to owners not taxable:

General partnership, limited liability partnership

Every transfer of money or property for less than adequate or full consideration in money

Gift

Offer and acceptance: the offer

Gives recipient (offeree) the power to form a contract by accepting before the offer is terminated

Guaranteed payments to a partner

Gross income

UCC: Delivery: Seller's basic duty

Hold conforming goods for the buyer and give the buyer reasonable notice to enable the buyer to take delivery.

Shareholder contributing property (not services) in exchange for corporation common stock has no gain or loss if

IRC section 351: 1. 80% control: own at least 80% voting stock and 80% nonvoting stock 2. No receipt of boot: -Cash withdrawn -Excess debt put into corporation

Liability Under the 1933 Act Section 11: civil Liability for Misstatement

Imposes civil liability for misstatements, whether or not intentional in registration statements

Series EE (US Savings Bonds)

Issued after 1989- tax exempt when: 1. Used to pay for higher education of taxpayer, spouse, dependents 2. Taxpayer over 24 when bond issued 3. Married taxpayer files jointly 4. Taxpayer meeting certain income requirements *Phaseouts

Only people who must register

Issuer, underwriter, dealer

Jane took her watch to the jeweler in order to have it repaired. The jeweler did not agree to do the repair on credit and expected to be paid in cash when Jane returned to pick up her watch. If Jane refuses to pay for the watch:

Jane took her watch to the jeweler in order to have it repaired. The jeweler did not agree to do the repair on credit and expected to be paid in cash when Jane returned to pick up her watch. If Jane refuses to pay for the watch: A. the jeweler must still return the watch to Jane and then file suit for the amount due for the repairs. B. the jeweler can recover based on his mechanic's lien. Incorrect C. the jeweler will have an artisan's lien on the watch whether or not he returns the watch to Jane. D. the jeweler could sell the watch to satisfy the outstanding amount due for the repair. An artisan's lien is a common law security device whereby a creditor can recover for work done on personal property of the debtor. If the debtor fails to pay for the work performed, the creditor can retain possession of the property and sell it in satisfaction of the lien. An artisan's lien is a possessory lien. If the lien holder voluntarily returns possession of the property to the debtor, the lien no longer exists. Finally, a mechanic's lien arises from the making of improvements to real property

Ultramares

Limits accountant's liability for negligence to: 1. Parties in privity 2. Intended third party beneficiaries

S Corporation shareholder permitted to deduct (on personal income tax return): Pro rata share of S Corp loss subject to limitation:

Loss limitation= Basis + Direct shareholder loans - distributions

Custodial fees and union dues

Miscellaneous itemized deductions subject to 2% limitation

Under which conditions is an on-site inspection of a workplace by an investigator from the Occupational Safety and Health Administration (OSHA) permissible?

OSHA may make an on-site inspection of a workplace *at the request of employees*. However, the search must be done *either under authority of a search warrant* or with the *consent of the employer*. The search warrant may be issued as a result of a *routine investigation*; it need not be based on probable cause.

Lack of reasonable care

Ordinary negligence. CPA is liable to anyone they know should expect will rely on his or her work.

Distribution of inventory results in what type of gain?

Ordinary, not capital

1933 Act regulates

Original issuances of securities

what are reasonable grounds to OMIT an answer on a client's tax return?

SSTS #2 says that only if both 1) the info isn't readily available, and 2) the answer is not significant in terms of taxable income or loss

In contrast to individual taxpayers, corporate capital gains taxed at

Same rate as ordinary income. No special capital gains rates apply

Capital interest acquired for services rendered FMV: partnership:

Taxable event. Valued at liquidation value. Partnership capital interest acquired for services rendered is ordinary income to the partner

What is your basis in INHERITED property?

The FMV of the property at the date used to value the property. *this can be the date of death or the alternate date as chosen by the executor

What are the four types of Individual Retirement Accounts?

The four IRA Accounts are: Deductible IRA Non-Deductible IRA Roth IRA Coverdell Education Savings Account

Disclosed principal: agent to 3rd party for contract duties

Third party can't hold authorized agent liable on the contract

Medical Expenses: AGI test

To the extent they exceed 10% of taxpayer's AGI are deductible

Identify the characteristics of the Business Healthcare Tax credits.

Up to 35% of the employer's cost of the plan premiums provided the employer contributes at least 50% of the costs of health coverage. May offset the alternative minimum tax; it is not refundable, and can be carried forward for 20 years. If expense were used to qualify for the credit, they are not allowed as a tax deduction for employee benefits expense.

how are gambling gains/losses treated for tax purposes?

Winnings from gambling activities are reported within the income of the taxpayer. Losses are shown as miscellaneous itemized deductions. However, the losses deducted cannot exceed the amount of gambling winnings being reported

are an insurance company's securities regulated by the securities act of 1933?

YES

What three duties does the principal always owe the agent?

compensation, reimbursement, remedies

What kind of remedies can be given under section 11?

damages only (no rescission thats only section 12)

Acceptances are generally effective when they are _________

dispatched

Who funds Worker's Compensation?

employer

Who has priority in perfected Non PMSI vs perfected Non PMSI?

first to file or perfect

where are tax preparer fees listed on a tax return?

it is a schedule A itemized item, even if the person earned all their money is from a schedule C sole proprietorship

how is an expenditure made for medical reasons that improves a residence treated for tax purposes?

it is an itemized deduction subject to a 7.5% of AGI threshold to the amount of the cost over the value it added to the home. If the cost was 10k and it increased the home's value by 8k, then 2k is deductible.

when the alternate valuation date is elected after a death, what is the basis for the property distributed?

it is the fair market value on the date that is six months after the date of death

what basis does a corp take in land contributed by an owner?

it takes the owner's basis plus any gain the owner recognized on the transaction.If an owner contributes land with 40k of basis but the corp gives him 10k in cash, the corp's basis in the property is 50k

A partnership is required to have the taxable year of:

its majority (over 50%) partners or if none, then the tax year of all its principal partners. If no taxable year can be established under these rules, then the tax year with the least aggregate deferral must be used. The aggregate deferral is calculated based on the number of months from the partnership year-end to the partner's year-end, multiplied by the partner's ownership percentage. In this case, test both the November 30 year-end and the December 31 year-end. (Partnership year-end - Partner year-end) × Ownership %.

how to do a wash sale question

let's say you sold 500 shares at a loss and then purchased 100 back within 30 days at a lower price. the % of shares you bought back is the percentage of the loss you're not allowed to take. So if selling the 500 shares got you a loss of $1,000, buying back 100 shares is 1/5 or 20% of the shares you sold. So $200 of loss would be disallowed.

difference in startup costs for a corp and organization costs for a corp?

nothing really- they are separate and each have the 5k rule but they are different. organization costs and startup costs are NOT lumped together

What is the criteria for filing: Married, filing jointly?

At the end of the year: Married and living together as husband and wife, or Living together in a recognized common law marriage, or Married and living apart, but not legally separated or divorced

With a shipment contract, the risk of loss passes to the buyer when _________

goods are given to carrier

What must always happen before title can pass?

goods must be identified

what does article 2 of the UCC govern?

governs the sale of new and used goods, by both merchants and non-merchants

Shareholders' inspection rights

Can inspect for any PROPER PURPOSE, but can be denied inspection for improper purposes

If aggregate adjusted basis of property contributed to a corporation by each transferor/shareholder > aggregate FMV of property transferred,

Corporation's basis in the property is limited to aggregate FMV of property

Cancellation of debt

Debt forgiveness. Gross income

Land: 179 deduction

Doesn't qualify

Calculating Qualified Production Activities Income

Domestic production gross receipts < COGS > < Other directly allocable expenses or losses > < Proper share of other deductions > = Qualified production activities income

Adoption fees: Medical expenses

Don't qualify for credit

Partnership: profits interest acquired for services rendered

Don't recognize ordinary income at the exchange

Contract doesn't specify the liability of each surety

Each surety is liable for a pro rata share determined by the number of solvent sureties

After tax cash flow

Earnings after tax + amortization + depletion + depreciation

FUTA funding

Employer only. Deductible for employer. 6% on first 7000 per year of compensation for each employee

Accident insurance: premiums paid by taxpayer

Exclude

De minimis fringe benefits

Excluded from income. Ex: Employee's personal use of a company computer

Tax to add to basis:

FMV - basis= X X/FMV= % % * amount of tax= add to basis

what are capital assets for a corporation?

For a corporation, capital assets are normally limited to investments in stocks, bonds, and land (held in hopes of appreciation in value). Buildings, equipment, and inventory are bought to produce revenues and do not qualify as capital assets.

Interest expense on a home equity line of credit for amount borrowed to finance business

Fully deductible on schedule C

Remedies: specific performance: used with

Land or unique items, not services

Unilateral mistake- when is it a defense?

Material fact- other party knew or should have known of the mistake. Contract voidable

Are punitive damages available in breaches of contract?

NO

For property tax to be deductible

Needs to be based on the value of the item

Realized loss like kind exchanges

Never recognized

Benefits received from a Roth IRA

Nontaxable

Custodial fees for retirement accounts

Not a business expense

Transfer made to repay debt that the debtor incurred in the ordinary course of business

Not a voidable preference

Charitable gift to needy family

Not deductible

Workers compensation

Not taxable

Statute of limitations for breach of contract usually begins to run

On occurrence of the breach

Business meals

Only 50% deductible

If the agency is coupled with an interest (Principal borrows money from agent):

Only the agent has the power to terminate the agency relationship

Depreciation- ordinary business income or K-1

Ordinary business income

Fraud in the execution

Party is deceived into signing something he doesn't know is a contract. Makes a contract void

Explain how medical expenses are deducted.

Payments made for medical expenses are deductible when made on the behalf of yourself, your spouse, and your dependents. Expenses must be paid or charged during the tax year. Qualified medical expenses to the extent they exceed medical insurance reimbursement and are 7.5% if the taxpayer's AGI are deductible.

Unilateral contract

Promise is exchanged for an act

FICA: Federal Insurance Contributions Act

Provides workers and their dependents with benefits in case of death, disability, or retirement. Funded by both employers and employees, including self employed individuals

Wash sale basis

Purchase price + disallowed loss

Phaseout of personal exemptions

Reduces exemption by 2% for every 2500 (1250 MFS) by which AGI > -MFJ/SS: 313800 -HOH: 287650 -Single: 261500 -MFS: 156900

Identify the statutory period for a taxpayer to receive a refund.

Refund claim: later of: Three years from the date the return was filed or the original due date of the return, or Two years from the time the tax was paid.

What are the two rules under Regulation D for private offering exemptions?

Rule 504 and 506

Identify the various filing statuses.

Single Married, Filing Jointly Married, Filing Separately Head of Household Qualifying Widow(er) with dependent child

LLP: File

Statement of qualification with state

An employer having an experience unemployment tax rate of 3.2% in a state having a standard unemployment tax rate of 5.4% may take a credit against a 6.2% federal unemployment tax rate of: A. 3.0%. B. 3.2%. Correct C. 5.4%. D. 6.2%.

The employer receives a credit of the state's standard unemployment tax rate—5.4% in this case. The credit is 5.4% on the first $7,000 paid to an individual employee. Under normal circumstances the effective federal tax rate would be 0.8% (6.2% - 5.4%).

Under Chapter 11 bankruptcy, the following are required:

The filing of a reorganization plan Confirmation of the reorganization plan by the court Opportunity for each class of claims to accept the reorganization plan NOT- LIQUIDATION OF DEBTOR LIKE UNDER CH 7

Discuss how annuity investments are taxed.

The investment portion of the annuity is divided by a factor representing the number of months over which the investment will be recovered. If the taxpayer lives longer than the recovery period any further payments received are fully taxable. If the taxpayer dies before the recovery period ends, the unrecovered portion is a miscellaneous itemized deduction on their final tax return.

What is the tax treatment of unearned income of a child under 18?

The net unearned income of a dependent child under the age of 18 years of age is taxed at the parent's higher tax rate. Calculation: Total unearned income Less: $1,900 Net unearned income Note: Report the net unearned income on the child's tax return. (Parents may elect to include income on their own return if the income is between $950 and $9,500 and consists solely of interest and dividends.)

Once buyer has rejected nonconforming goods

Title reverts to the seller

Gain/loss on year end sale of listed stock- which date?

Trade date

Financial balance sheet basis

Use FMV

what is a type a reorg?

a statutory merger- target ceases to exist, but target shareholders now own stock in acquiring corp

Who can terminate an agency relationship?

any party at any time (But they might not have the right to and might get sued)

the benefit of using the installment sales method is removed by:

being added to taxable income as an adjustment

directors will not be held liable to the corporation for acts performed or decisions made in good faith, in a menner the director believes to be in the best interest of the corporation, and with the care an ordinary prudent person in a like position would exercise

business judgement rule

what a bond is amortized for tax purposes, what method is used?

constant yield to maturity method

An insider must register with the SEC their initial ownership and subsequent changes in ownership. An insider is a

director, officer, or person who owns more than 10% of a class of securities registered under the 1934 Act. These are the individuals that must register with the SEC.

If there is a material breach of contract under common law, what is the non-breaching party liable to?

discharge contract

if principal's existence and identity are known to third party, agent is not liable to third party under the contract

disclosed principal

Liability under a general partnership is ___________ and _________

joint; several

what are the 2 categories of qualified moving expenses?

moving the household goods, and lodging during the move. temporary living expenses are NOT deductible

Attachment alone is not enough to ensure a creditor receives claims against a debtor over third parties. What must a creditor do to gain rights over third parties?

perfect

the goods and delivery must conform exactly to the contract without any defects, including warranties

perfect tender

if you have preferred and common shareholders, 270k in your profits and earnings account, and you pay 200k dividends to preferred shareholders and 100k dividends to common shareholders, how do they each report them?

preferred report 200k of dividends, and common shareholders report 70k of dividends and 30k of nontaxable return of basis. When corporate distributions exceed earnings and profits, the earnings and profits are first allocated to the distribution made to the preferred shareholders with any remainder to the common shareholders

bilateral contract

promise for a promise

unilateral contract

promise in exchange for an act

What is the purpose of the Securities Act of 1933?

provide investors with sufficient investment information

an offer for the sale of goods generally only needs to include ____________

quantity

Who is the creditor's committee usually made up of in a CH 11 case?

seven largest unsecured creditors (usually)

limited partners are like _______

shareholders

2 ways intent can be established:

show that the defendant acted recklessly, or that the defendant knew of the misrepresentation

What is the exception to the Statute of Frauds, for contracts for the sale of goods (UCC) over $500 that do not need to be in writing (SWAP) ?

specifically manufactured goods, written confirmation between merchants, admitted contracts in court, performed contracts

In general, in the case where a taxpayer borrows money to buy property that is held for investment, the interest that the taxpayer pays is investment interest and it can be deducted up to the amount of investment income. However, a person cannot deduct interest incurred to produce

tax-exempt income (municipal bonds) and passive activity or rental real estate activity (rental income). Investment income generally includes gross income from property held for investment such as interest, dividends, annuities, and royalties.

under circular 230, if you find an error on a client's tax return after he sends it, you are required to:

tell the client- it's up to them what to do next

an offer to all shareholders to purchase stock for a specified price for a specified period of time

tender

In non-carrier cases, where the seller is NOT a merchant, risk of loss passes to the buyer on the seller's _____________

tender of delivery

what happens if a owner/employee is getting paid 300k and 50k of that is determined to be unreasonable compensation?

that 50k will then be taxed at a dividend but it also becomes NOT-deductible for the corp

An employee who has had Social Security tax withheld in an amount greater than the maximum for a particular year may claim:

the excess as a credit against income tax, if that excess resulted from correct withholding by two or more employers. If the taxpayer only worked for one employer and too much Social Security tax was withheld in error, then the taxpayer cannot claim the excess as a credit against income tax. In this case, the employer should make an adjustment and refund to the employee, or the employee can use IRS Form 843, Claim for Refund and Request for Abatement, to claim a refund.

what is the AMT exemption regarding 40k?

the exemption is 40k minus 25% of the AMT income exceeding 150k.If you have 200k of AMT income then 50k applies to the exemption. That would be 40k - (50k x .25) which equals 27,500

how are doubtful accounts treated for tax purposes?

the expense has to be incurred- meaning only the amount of accounts actually written off are able to be deducted

if you give someone a check for 120k and they don't deposit it within 30 days and then there is a bank failure, you are off the hook for:

the extra 20k. The FDIC will cover 100k.

which senate committee considers new tax legislation?

the finance committee

With a destination contract, the risk of loss passes to the buyer when _________

the goods are delivered by common carrier

Tax avoidance

the legal use and application of the tax laws and cases in order to reduce the amount of tax due

what rate are capital gains for corporations taxed at?

the ordinary tax rate for the corp. it's not a different rate like it is for individuals. when a net capital loss is either carried back or forward it is always treated as a short term loss

The holding period of a partnership interest acquired in exchange for a contributed capital asset begins on the date:

the partner's holding period of the capital asset began.When a partner contributes property to a partnership, his adjusted basis in the partnership interest increases by the adjusted basis of the property contributed. This is called a carryover basis. Generally, the holding period will "attach" to the basis used for the property.

in a non-liquidating distribution of property, how do you report the transaction?

the same as if you'd sold the property at its fair market value. If you had stock you bought for $10 per share, and you gave it to an owner of the corp when it was worth $20 per share, you'd report $10 of capital gains per share.

In a general partnership, matters regarding fundamental changes can be approved by a _________ vote

unanimous

defense that arises due to abusing a position of trust or confidence

undue influence

for a corp, how long can you carry forward excess charitable contributions?

up to five years

details of an output contract

when a customer agrees to buy all their materials from one supplier. courts decided that parties probably know about how much a supplier will order, so that is enough consideration to support a contract. If a contract cant be concluded within a year of its execution it must be supported by a signed writing under the statute of frauds

is interest on municipal arbitrage bonds and interest on US treasury notes taxable to a corp?

yes- both are taxable.

are estimated state income taxes deductible?

yes- fully deductible as an itemized deduction schedule A

are real estate taxes on a principal residence deductible?

yes- fully deductible as an itemized deduction schedule A

is interest on funds borrowed for working capital deductible?

yes.

is debt relief considered boot?

yes. that and cash

What must be filed for a limited partnership to be formed?

certificate of limited partnership (with state)

what types of entities are exempt from the registration requirements of the securities act of 1933 when they issue securities?

charities, railroad company, farmer's cooperative, or banks

Under the Securities Act of 1933, what kind of liability can be imposed for misstatements, whether intentional or unintentional, in registration statements

civil

No investor may invest in aggregate more than ________ in Regulation Crowdfunding in a 12-month period

$107,000

business gifts up to $____ per person can be deducted

$25

To be eligible for Regulation Crowdfunding exemptions, the issuers must have assets less than _____________; also may not sell more than ___________ in securities under Regulation Crowdfunding in a 12-month period

$25 Million; $1.07 Million

The simplified home office deduction can be claimed starting in tax year 2013. Using the optional method, you simply deduct :

$5 for every square foot of your home office. However, the deduction is capped at $1,500 per year; so it can only be used for offices up to 300 square feet.

The lifetime gift tax exclusion is:

$5.45 million

A small corporation is exempt from AMT tax. In general, to be considered a small corporation for this purpose, the corporation must have average annual gross receipts under:

$7.5 million for the preceding 3-year period. A special rule applies for the first two years a corporation is in existence. All corporations are exempt from AMT for their first tax year. The second tax year of a new corporation is its initial test year. The gross receipts test for a corporation's initial test year is $5 million to qualify as a small corporation exempt from AMT. For all later years, the gross receipts test is $7.5 million.

Given current E&P, Accumulated &P, and total distributions....taxable dividend income to one shareholder=

(Current + Accumulated)/total distributions * shareholder's total distribution- taxable dividend income

What is necessary to form a general partnership?

- 2 or more persons - Who agree (expressly or impliedly) - Carry on as co-owners a business for profit - No requirement for writing, unless partnership is to last for more than one year

What happens if the debtor is a consumer and the creditor chooses to retain the collateral for possession?

considered full satisfaction

S corporation with no C Corp E&P 1. Distribution to extent of basis in stock - Tax result - Treatment 2. In excess of basis of stock - Tax result - Treatment

1. - Not subject to tax, reduces basis in stock - Return of capital 2. - Treated as long term capital gain. If stock held > 1 year - Capital gain distribution

Detailed alternative computation of basis to shareholder

1. Adjusted basis of transferred property (including cash) + FMV of services rendered + Gain recognized by shareholder - Cash received - Liabilities assumed by the corporation - FMV of nonmoney boot received = Basis of common stock

Filing requirements for trust and estate income tax years: 1. Trusts 2. Estates

1. Calendar year end 2. Anytime

Statute of Frauds: contracts for the sale of goods 500 or more must be evidenced by a writing signed by the party being sued. Exceptions:

1. Contracts for specially manufactured goods (not suitable for sale to others) 2. A merchant sends another merchant a written confirmation of a contract that is sufficient to bind the sender, it will also bind the recipient if she doesn't object within 10 days. Merchant's confirmatory memo rule

Estates and trusts 1. Distributions made by entities 2. Income distributed to beneficiaries

1. Deductible by the entity 2. Taxable up to maximum amount of taxable income to beneficiary if distributed to them (DNI)

Disclosure of tax return information: when can tax preparer disclosure information without taxpayer's consent?

1. Disclosures allowed by any provision of the IRC and disclosures pursuant to a court order 2. Allowable uses (preparation of state and local tax returns and preparation of declaration of estimated tax) 3. Disclosures and uses permitted by US Treasury Regulations for quality peer reviews and administrative orders **Confidential client information may be disclosed to any party if client consents to release of information

Payment of property dividends 1. Corporation has no E&P: Dividend: 2. After E&P created:

1. Dividend would not be taxable income Then corporation gain increases/creates corporate E&P 2. Dividend to shareholder is now taxable income (to the extent of E&P)

Duties of agent to principal: LORA

1. Duty of loyalty 2. Obedience 3. Reasonable care 4. Duty to account: Agent can't commingle principal's property with agent's property 5. Subagent: agent hires assistant, subagent owes duty of care to both agent and the principal

Disqualified Dividends

1. Employer stock held by ESOP 2. Amounts taken into account as investment income 3. Short sale positions 4. Certain foreign corporations 5. Dividends paid by credit unions, mutual savings banks, building and loan associations, mutual insurance companies, farmer's cooperatives

Taxpayer can't deduct contribution to IRA when BOTH conditions present

1. Excessive AGI Single/HOH: 62000- 72000 Joint: 99000- 119000 2. Active participation in another qualified plan **Not considered active participant in plan just because spouse is an active participant **Max deductible for individual who is not an active participant, but whose spouse is, is phased out for taxpayers with modified AGI > 186,000 but less than 196,000

Scholarships and fellowship grants: 1. Degree seeking student 2. Non degree seeking student

1. Excludable only up to amounts actually spent on tuition, fees, books, and supplies. NOT ROOM AND BOARD. No services are to be performed as a condition to receiving the grant; grant not made in consideration for past, present, or future services of the grantee 2. Scholarships and fellowships fully taxable at FMV

General rule for taxable events and basis: corporations 1. Taxable: Income; basis 2. Nontaxable: income, basis

1. FMV; FMV 2. NONE; NBV

S Corp 1. Tax year 2. Tax on corporation?

1. GR: Dec. 31 required year end Return due by March 15th 2. None. All earnings pass through to shareholders. Shareholder reports income and pays tax

Holding period of partnership interest 1. Property previously a capital asset or section 1231 asset in hands of partner: 2. Property is ordinary income asset

1. Holding period= starts when initially purchased by partner 2. (Inventory). Holding periods begins on the date property is contributed to the partnership

Types of qualified stock options

1. Incentive stock options (ISO) 2. Employee purchase plans (ESPP)

1. Gambling winnings 2. Gambling losses

1. Included in GI 2. Deductible to extent of gambling winnings. Deductible on schedule A as itemized deduction, but not subject to 2% of AGI limitation on miscellaneous itemized deductions

1. Alimony received 2. Alimony paid

1. Income 2. Deductible

Gift loans not in excess of 100,000 1. Foregone interest included by lender and deducted by borrower 2. Borrower's net investment income < 1000

1. Limited to borrower's net investment income for year 2. Foregone interest is 0

Elements of fraud (intentional misrepresentation) MAIDS

1. Misrepresentation of material fact 2. Actual and justifiable reliance by plaintiff on the misrepresentation 3. Intent to induce plaintiff's reliance on the misrepresentation 4. Damages 5. Intent to deceive (knowing the statement was false)- SCIENTER

Exceptions to Passive Activity Loss Rules: Can deduct rental activity losses if either of the two conditions are met:

1. Mom and pop exception: Deduct up to 25,000 per year of net passive losses for rental real estate if TP is actively participating/managing and own at least 10% of the rental activity. Subject to phaseouts. 25000 reduced by 50% of excess of AGI 100000. Eliminated completely if AGI > 150000 2. Real estate professional= not passive; active * > 50% personal services performed real estate * >750 hours services in real property businesses during year

Corporation AMT: Regular taxable income: Adjusted current earnings: Increase/decrease

1. Municipal interest income 2. Organizational expense amortization 3. Life insurance proceeds on key employees 4. Difference between AMT and ACE depreciation 5. Dividends received deduction under 20% ownership (70%)

1. Individuals who make below market loans 2. Borrowers

1. Must report any foregone interest as interest income 2. May be able to deduct foregone interest, unless it's personal interest

Bad debts: Specific charge off method 1. Accrual basis TP 2. Cash basis TP

1. Must use direct write off method for purposes. Tax deductible when specific A/R is written off 2. No deductions. It was never income

1. Under sales article, promise to ship or prompt shipment 2. Seller ships goods that don't conform to the contract 3. Seller notifies buyer that nonconforming goods are shipped as accommodation to the buyer

1. Offer to buy goods for current or prompt shipment can be accepted by either a promise to ship or prompt shipment. 2. A shipment of nonconforming goods is an acceptance and a breach of contract 3. Shipment is not an acceptance. Counteroffer

Income Baskets

1. Ordinary 2. Portfolio 3. Passive 4. Capital- sale of capital assets

1. Gift- bargained for exchange? 2. Past or moral consideration: something already given or performed before the promise is made:

1. Promise to make a gift are unenforceable because of lack of consideration. No bargained for exchange 2. won't satisfy the bargained for exchange requirement

1. Preexisting legal duty rule: 2. Exception

1. Promise to perform or performance of an existing duty is not sufficient consideration 2. Each party offers to give something different from what was originally promised

Reduced tax rate of 20% (in 39.6% bracket) 15% (most taxpayers) 0% (15 or 10% brackets) Provided for:

1. Qualified Dividends 2. Long term capital gains

Child and dependent care credit- who is eligible?

1. Qualifying child under 13 2. Disabled dependent of any age unable to care for himself 3. Spouse disabled unable to take care of themselves

Exchange of like-kind business/investment assets (except inventory, stock, securities, partnership interests, etc) (tangible) *Qualifying property

1. Real property used in trade or business or for investment exchanged for other real property used in trade or business or investment 2. Personal property: must be exchanged for other personal property that has the same general use

Nonqualified stock options: 1. Adjusted basis 2. Capital gain or loss

1. Recognized ordinary income What stock was selling for x shares + Exercise price Exercise price x shares 2. Selling price - adjusted basis

Section 179 election 1. What happens to basis? 2. Election not allowed when?

1. Reduction of basis. Cost - 179 election 2. Year there is a loss

State and local tax refunds 1. Itemized in prior year 2. Standard deduction in prior year (1040 EZ) 3. Interest income on late refund

1. Refund taxable 2. Nontaxable refund 3. Taxable income

Corporation 3 taxes

1. Regular tax 2. Accumulated earnings tax; personal holding company tax 3. Alternative minimum tax

UCC 1. Sale on Approval 2. Sale or return

1. Risk on seller until approval 2. Risk on buyer until returned (consignment)

an agreement between a debtor and at least two creditors that the debtor pays the creditors less than their full claims in full satisfaction of their debt

creditor's composition

Restrictions on S Corp

1. Stock held by no more than 100 persons 2. Shareholders must be individuals, estates, or certain trusts 3. Corporation must be domestic 4. Only one class of stock .5. Foreign shareholders prohibited

Unreasonable position taken by the taxpayer Position deemed unreasonable unless

1. Substantial authority for the position (regardless of disclosure) exists (33-50) 2. Reasonable basis for a disclosed position exists (20%) 3. Reasonable to believe that a tax shelter or reportable transaction position would meet the more likely than not standard ( > 50%)

Distribution of corporate earnings 1. To extent of AAA 2. To extent of C E&P

1. Tax free 2. Taxable dividend

Contributions to a traditional IRA are deductible in arriving at AGI unless *both* of the following conditions apply:

1. The individual is actively participating in another pension or profit sharing plan AND 2. AGI on the tax return exceeds 71000 (single), 118000 MFJ

What is the maximum amount of shareholders for an S-Corp?

100

what is the number of days per year that determines whether a property is treated as a rental or not?

15 days. If a property is rented for less than 15 days, no rental income is includable and expenses attributable to the rental are not deductible. The property taxes are still deductible in full on Schedule A

what is the statutory amortization period for a covenant not to compete that is related to business acquisition?

15 years

What amount of creditors must accept the debtor's reorganization plan in CH 11?

2/3 in amount

General business credit: R&D tax credit

20% of increased in qualified research over a defined base amount

Lifetime gifts and death time transfers: unified estate: credit

2141800 unified estate and gift tax credit exempts from the gift tax cumulative, nonexcluded gifts having a value of 5490000

How long is a merchant's firm offer held open for if no time is stated?

3 months

2017 Exemption amount

4050: Taxpayer, spouse, dependents

Traditional IRA: additional catch up contribution

50 years old or older allowed additional contribution (adjustment) of 1000 For AGI deduction Principal and earnings withdrawals both taxable

what is the maximum deduction in one year for a section 1244 loss?

50,000- 100k if filing jointly

Beginning on January 1, 2016, the standard mileage rates for the use of a car (also vans, pickups or panel trucks) is:

54 cents per mile for business miles driven, 19 cents per mile driven for medical or moving purposes, and 14 cents per mile driven in service of charitable organizations.

Which of the following bodies promulgates standards for audits of federal financial assistance recipients? A. Governmental Accounting Standards Board B. Financial Accounting Standards Board C. Government Accountability Office Incorrect D. Governmental Auditing Standards Board

=c

Uniform capitalization rules don't apply to inventory acquired for resale if taxpayer's average gross receipts the last 3 years aren't

> 10,000,000

Individual subject to a gift tax if they give

> 14000. Exclude first 14000 of gifts made to each donee File form 709

Personal schedule `

A

One of the elections a new corporation must make is its choice of an accounting period. Which of the following entities has the most flexibility in choosing an accounting period?

A C corporation can adopt any tax year. Generally, C corporations could choose the last day of any month to end their year. S corporations must use the calendar year for tax reporting. Trusts and personal service corporations must use a calendar year. Generally, partnerships will use a calendar year.

A CPA will be liable to a tax client for damages resulting from all of the following actions, except:

A CPA will be liable to a tax client for damages from: failing to timely file a client's return, failing to advise a client of certain tax deductions, and neglecting to evaluate the option of preparing joint or separate returns that would have resulted in substantial tax savings for a married client.

What is the Foreign Tax credit?

A credit for foreign income taxes paid to a foreign county or US possession. Credit is limited to the lesser of: Foreign taxes paid or Taxable income from all foreign operations Taxable income + Exemptions Credit can be carried back one year or carried forward ten years.

When can a taxpayer receive an Adoption credit?

A credit is available for all reasonable and necessary expenses, costs, and fees. Per child: $13,170 Special needs child: $13,170 Medical expenses do not qualify.

Explain the Employer Pension credit.

A credit of 50% of first $1,000 of start up costs for establishing a new qualified pension plan for three years. (Up to $500 per year.)

Describe the characteristics of a Deductible IRA.

A deductible IRA has the following features: Earnings are tax deferred Withdrawals are taxable Deduct contributions up to $5,000/$10,000 Allows a catch-up contributions for persons 50 years and over of $1,000 Receive a tax credit for contributions made.

The following are considered when computing a partner's basis in a partnership:

A general partner's partnership basis is increased by his or her share of all recourse debt. A limited partner's partnership basis is increased by their share of recourse debt *IF they share in partnership losses*. The partners are allocated all recourse debt of the partnership in calculating the tax basis of their partnership investment. If a limited partner is not allocated a portion of recourse debt, then the general partner's allocation of debt must be adjusted upward to offset the limited partner's reduced allocation.

The selection of an accounting method for tax purposes by a newly incorporated C corporation:

A new corporation selects its accounting method by selecting the appropriate box when it files its first tax return. If a different accounting method is desired later, the corporation must obtain IRS approval.

Wash Sale

A security (stock or bond) is sold for a loss and is repurchased within 30 days before or after the sale date Disallowed loss

Earned income credit:

A special refundable tax credit may be available for low-income workers who have a principal residence in the United States. It represents a form of negative income tax—workers may receive money from the government even though they do not have a tax liability. a. The earned income credit (EIC) is equal to a percentage of a limited amount of earned income. Taxpayers with qualifying children receive greater benefits—a greater amount of income is eligible for a higher credit percentage. b. When the taxpayer's adjusted gross income (or earned income, if greater) exceeds a threshold amount, the EIC is phased out. (1) "Earned income" includes only taxable compensation; it does not include nontaxable employee compensation. (2) No credit is allowed to those with "disqualified income" (i.e., investment income or unearned income) in excess of $3,350 for 2016. (3) No credit is allowed for those failing to provide correct Social Security numbers for themselves, spouse, and qualifying child. (4) The maximum amount of credit for 2016 is $6,269 with three or more qualifying children, $5,572 with two qualifying children, $3,373 with one qualifying child, and $506 with no qualifying children. c. The government provides an easy-to-use EIC table to determine the correct credit. d. Qualifying children: To be eligible for the earned income credit, parents must have children that can meet the following tests: (1) Relationship: The child must be a "qualifying child". The child may, however, provide over half of his or her own support. (2) Residency: The child must live in the taxpayer's residence over half of the year; foster children for the entire tax year. (3) Age: The child must be: (a) under age 19, (b) a full-time student under age 24, or (c) permanently and totally disabled. e. Individuals without qualifying children may be eligible for this credit if: (1) they (or their spouse) are at least 25 years old, but not more than 64 years old, at the end of the year and (2) they cannot be claimed as a dependent by another taxpayer.

What is the criteria for filing a tax return?

A taxpayer must file a return, if their income is greater than the sum of: The personal exemption, plus The regular standard deduction, plus The additional standard deduction amount for taxpayers 65 or over, and/or blind.

When will a deduction for contributions made to an IRA not be permitted?

A taxpayer will not be permitted to deduct a contribution to an IRA when both the following conditions are exist: Excessive AGI: If AGI exceeds $66,000 or $109,000; Active participation in another qualified plan

Taxpayers using a flexible spending account for a health savings account are allowed:

A taxpayer's company is able to choose an allowance of either a $500 carryover balance or a grace period through March 15 of the following year on flexible spending accounts. Previous to 2014, there was a "use it or lose it" policy.

What is a use tax?

A use tax is a tax imposed for the storage, use, or purchase of personal property. It is similar to the sales tax, but is imposed on items not covered by sales tax

What is a Wash Sale loss?

A wash sale exists when a security is sold for a loss and is repurchased within 30 days before or after the sale date. The basis of the repurchased security is equal to the purchase price of the new security plus the disallowed loss on the wash sale. The date of acquisition of the repurchased security is the date of acquisition of the original security. If a sold security results in a gain and it is repurchased within 30 days, the taxpayer cannot use "substituted basis" They must use the new purchase price as the basis.

Present interest gifts A. Examples B. Treatment

A. - Outright gifts of cash or property - Trust income interests where annual or more frequent distribution is mandatory - Life estates - Estates for a term certain - Bonds or notes - Unrestricted transfers of life insurance policies B. Qualifies for annual exclusion and removed from the estate

- Business activity - Corporate consequence - Shareholder consequence A. Liquidation B. Reorganization

A. - Completely ceases - Taxable - Taxable B. - Continues - Nontaxable - Nontaxable

Future interest gifts A. Examples B. Treatment

A. - Reversions - Remainders (distributed some future time) - Trust income interests where accumulation of income by a trustee is mandatory and accumulations distributable some future time at discretion of trustee - Present interests without ascertainable value B. Doesn't qualify for annual exclusion, will not be removed from the estate

When a corporation has a group term life insurance plan available for employees, the following is considered to be discriminatory:

A. the plan can exclude employees who have worked for the corporation less than two years. Incorrect B. the plan will offer $3,000 of group term life insurance, up to a maximum of $50,000, for each $10,000 of salary earned by the employee. C. the plan will offer $50,000 of group term life insurance to officers and $10,000 of group-term life insurance to all nonofficers. D. the plan can exclude part-time employees. Since the plan offers $50,000 of group term life insurance to officers and $10,000 of group term life insurance to nonofficers, this plan would discriminate in favor of key employees. When a corporation has a group term life insurance plan, it is not discriminatory to exclude employees who have worked for the corporation less than three years, to offer group term insurance which varies with the amount of compensation, or to exclude part-time employees.

Corporation: organizational expenditures and start up costs: included costs

A. Fees for legal services in drafting corporate charter, bylaws, minutes of organization meetings, fees paid for accounting services, fees paid to state of incorporation

Incomplete Gifts A. Gift tax? B. Examples

A. Not subject to gift tax B. - Conditional gifts - Revocable gifts: Donor reserves the right to revoke the gift or change the beneficiaries

Gift recipients A. Taxed? B. Basis?

A. Pays no gift tax, not taxable income to the recipient B. Donor's basis + gift tax paid

Complete gifts A. Gift tax? B. Examples

A. Subject to gift tax B. - Even though donee not yet born, provided identity can later be ascertained - Despite possibility property may revert to the donor some future time

Net long term capital gain

Add to beginning basis partnership

Net capital gains of a corporation

Added to ordinary income and taxed at the regular rate. No lower, special rate.

Complete liquidation: Partner's basis for distributed property

Adjusted basis of his/her partnership interest reduce by money actually received. Recognized gain only to extent money received > partner's basis in partnership

You paid boot: Basis in new property

Adjusted basis of property given up + boot paid

What are the Adjustments for the Alternative Minimum Tax calculation?

Adjustments are timing differences: P Passive Activity Losses A Accelerated Depreciation N Net operating Loss of the individual taxpayer I Installment income of a dealer C Contracts - percentage completion vs. completed contract T Tax "deductions" I Interest deductions on some home "equity loans" M Medical deductions (limited to the excess over 10% AGI) M Miscellaneous deductions not allowed E Exemptions (personal) and standard deduction Timme adjustments do not generate a AMT credit for future years. They are always adds to regular taxable income.

List the Adjustments for AGI.

Adjustments for AGI include the following: Educator Expenses IRA Student Loan Interest Expense Tuition & Fee Deduction Health Savings Account Moving Expenses One-half Self Employment FICA Self-employed Insurance Self-employed Retirement Interest Withdrawal Penalty Alimony Paid Attorney Fees Paid - Discrimination/Whistleblower cases Domestic Production Activities Deduction Exam can use phrase: "Deductions to arrive at AGI"

S Corp election terminated or revoked: New election can be made

After 5 years. Or ask IRS permission

Subrogation

After paying principal debtor's obligation, surety may enforce any rights the creditor had against the principal debtor. Includes the right to enforcement of any security interest and any priority in bankruptcy the creditor had

When principal terminates an agent's actual authority:

Agent continues to have apparent authority until principle notifies third parties who might have known of the agency - Actual notice: terminate apparent authority to old customers - Constructive notice: potential customers

Partially disclosed and undisclosed principal

Agent liable to 3rd party

Nonaccountable plan

All benefits included in GI

Effect of a sale of collateral

All subordinate claims are wiped out. No right of redemption by subordinate security interest holders or the debtor

At what level must all tax cases start?

All tax cases must start at the trial court level. Tax Court, U.S. District Court, and U.S. Court of Federal Claims are the three courts for federal tax disputes.

Fuller was the owner and beneficiary of a $200,000 life insurance policy on a parent. Fuller sold the policy to Decker, for $25,000. Decker paid a total of $40,000 in premiums. Upon the death of the parent, what amount must Decker include in gross income?

Although life insurance death proceeds are generally not taxable to the recipient, there are special rules if the policy is transferred for value. In that case the death proceeds are taxable, except to the extent of basis. Basis in such a policy consists of: consideration paid for the policy, premiums paid under the policy, and interest expense on debt incurred to finance the policy which would not have been allowed as a deduction previously.

In the U.S. Tax Court, where may cases be heard?

Although the U.S. Tax Court is located in Washington, D.C., judges travel to different locations. There is a "Request for Place of Trial" form that must be submitted

A deduction for qualified education expenses can't be taken if

American opportunity or lifetime learning credit taken in the same year

Individual sold 1250 asset at a gain and included gain with other 1231 gains

Amount = to lesser of -Recognized gain on sale of 1250 asset -SL accumulated depreciation 1250 asset Taxed at max rate 25% Gain in excess of that amount taxed at preferential rates of 0, 15, 20

What is the basic formula for determining a gain or loss?

Amount Realized (Adjusted Basis of Asset Sold) Gain or Loss

Taxable portion of small business stocks

Amount of gain that exceeds 10,000,000 Includable portion of gain: taxed at regular rates

Generally, a disclosed principal will be liable to third parties for its agent's unauthorized misrepresentations if the agent is:

An employee (not an independent contractor)

1. Rule 10b-5 2. To recover damages for 10b-5:

Antifraud provisions in connection with the purchase or sale of any security. Applies whether or not securities are of a registered company. Anyone who buys and sells can be liable. 2. - Plaintiff bought or sold securities - Plaintiff suffered a loss - Material misrepresentation or material omission of fact - PLAINTIFF MUST SHOW SCIENTER (defendant intended to deceive or made false statements with a reckless disregard for the truth) . Negligence on the part of the defendent not sufficient. - Plaintiff must have RELIED on defendent's misrepresentation - PLaintiff must show that a means of interstate commerce was involved

Basis adjusted downward

Any depreciation taken by taxpayer on that asset

Who can be held liable under section 11 for the Securities Act of 1933?

Anyone who signs registration statement

Holding period inherited property acquired from decedent

Automatically long-term regardless of how long it has been held

MACRS 5 year property includes

Automobiles, light trucks, computers, and copiers

Damage award compensation for lost profit

Award is income

What is the mnemonic for securities that are exempt from registration with the SEC?

BRINGS (securities issued by Banks, Railroads, Insurance Policies, Not-for-profits, Government securities, Short-term commercial paper)

When is a gain not taxed?

Based upon the belief that a taxpayer's investment has no substantially changed, the gain is excluded from tax. These exclusions are: H Homeowner's exclusion I Involuntary Conversions D Divorced Property Settlement E Exchange of like-kind business investment assets I Installment sales T Treasury and capital stock transactions (by corporation)

To determine charitable contribution deduction for corporations; use taxable income:

Before charitable contributions and dividends received deduction

Anticipatory repudiation

Buyer or seller indicates in advance of performance that he will not perform

Workpapers: prospective purchaser of CPA firm's practice

CPA may allow a prospective purchases to review confidential workpapers, but the CPA may not turn over the workpapers to a purchaser without the client's permission

NET self employment income (deduct expenses) used for

Calculation of GI

Simple partnership basis Basis=

Capital account + Partner's share of liabilities

What is a capital asset?

Capital assets are property (real and personal) held by the taxpayer, such as: Personal automobile Furniture and fixtures Stocks and securities Copyrights Interest in a partnership Goodwill Literary, musical, or artistic compositions that have been purchased Real property and personal property not used in a business Other assets held for investment

what are the types of capital assets for individuals?

Capital assets for individual taxpayers are personal property, such as household furniture and tools, and investment property, such as real estate and securities such as stocks and bonds.

What must be filed for a limited liability partnership to be formed?

Certificate of Limited Liability Partnership (With state)

Limited partnership: file

Certificate of limited partnership with the state

Which of these can be filed voluntarily: CH 7, CH 11, CH 13?

Ch 7 and CH 11

Increasing marginal tax rate: marginal tax rate=

Change in tax liability/change in taxable income

Bankruptcy: trustee required for: Not required for

Chapter 7: liquidation Chapter 13: adjustment of debts of individuals with regular income Chapter 11: reorganization; debtor remains in possession of the estate's assets

Liquidated damage clause

Clause in a contract that specifies what damages will be if there is a breach. Must be reasonable and not a penalty

Affiliated group: tax purposes

Common parent directly owns: 1. 80% or more of the voting power of all outstanding stock 2. 80% or more of value of all outstanding stock of each corporation

Return of capital

Company distributes funds but has no earnings and profits. Taxpayer reduces (but not below zero) basis in common stock held. EXEMPT FROM GI.

Tax of UBI

Computed at corporate income tax rates

Counteroffer

Considered to be a rejection (terminates the original offer) and an offer (original offeror is now the offeree who may accept or reject)

Describe the features of a Coverdell Education Savings account.

Contributions are non-deductible; maximum contribution per beneficiary is $2,000. Contributions must be made for any child under the age of 18 Earnings accumulate tax-free Distributions are tax-free to the extent they are used for qualified educational expenses of the beneficiary Amounts must be withdrawn before beneficiary reaches age 30 years Distributions made directly to a beneficiary are taxable and assessed a 10% penalty The balance may be rolled over to another family member

Distributions from C corporations are considered dividend income (ordinary income) to extent of:

Corporation's current or accumulated E&P. Excess distribution treated as non taxable return of capital to extent of shareholder's basis. Excess over ^^: Capital gain

Personal holding company definition

Corporations > 50% owned by 5 or fewer individuals and have 60% of adjusted ordinary gross income consisting of: 1. Net rent 2. Interest that is taxable 3. Royalties 4. Dividends from unrelated domestic corporation ***Not subject to accumulated earnings tax

Commute from day to evening job

Deductible

UCC: risk of loss

Depends on time delivery is made. If goods are damaged or destroyed after risk of loss has passed to the buyer, the buyer isn't discharged from the contract, must still pay the contract price

A variation of the contract that changes a compensated surety's risk

Discharge surety only if the change is material and increases surety's risk of loss

What is the criteria for filing Single?

Does not qualify for another filing status Unmarried or legally separated from spouse at December 31.

Empty house and available for sale

Doesn't disqualify gain for exclusion

Holding period

Donor's holding period unless basis becomes FMV, then holding period starts at the date of the gift

What is the defendant's number one defense against a Section 11, 12, or 17 lawsuit under the Securities Act of 1933?

Due Diligence

How many years can student deduct portion of student loan interest?

Duration of time that interest is paid

Rental income or loss: schedule

E; passive activity

Tax evasion

Efforts, by illegal means and methods to not pay taxes

COD

Excess debt assumed by the buyer

Cancellation of debt income: Partnerships

Excess of amount of the debt discharged over the FMV of partnership interest that partnership transfers to the creditor

ISO: FMV of stock on exercise date > option price

Excess x number of shares Ordinary gain AMT preference

Personal physical injury or illness award

Exclude

Life insurance dividend: ownership of insurance with a mutual company (premium return)

Exempt from GI

Stock dividend 1. Same stock 2. Different stock

Exempt from GI. 1. Same stock: original basis divided by total shares 2. Original basis allocated based on relative FMV of different stock

Tax savings amortization, depletion, or depreciation=

Expense x marginal income tax rate

What types of debts of an individual are not discharged under CH 7 or 11?

FATWED; Fraud, Alimony, Taxes due within 3 years, Willful and Malicious injury, Educational Loans, Debts undisclosed

provides workers and their dependents with benefits in case of death, disability, or retirement (includes self-employed)

FICA (Federal Insurance Contributions Act)

When services are paid for in property, how much is included in income?

FMV at the time of receipt.

Prizes and awards

FMV is taxable income unless 1. Winner selected for award without entering into contest 2. Assigns award directly to governmental unit or charitable organization

What amount is subject to tax for gift?

FMV. Even if gift basis to donee is rollover basis.

A director of a corporation is a

Fiduciary

Corporations: tax rate on AMTI

Flat 20%

What items are used in the computation of the built-in gains tax liability for an S corporation?

Flat 35% tax rate: Yes; Deduct unexpired NOLs and C corp. capital losses: Yes Both of the items are used. The maximum corporate rate is used to prevent tax avoidance by converting C corporation income to S corporation income. Unexpired benefits of the C corporation are deducted (NOLs, capital losses, AMT credits, and business credit carryforwards).

Sole proprietorship

Flow through entity reports income on 1040; Schedule C. LLC & only owner

Nonresident alien withholding

Foreign person is subject to US tax on its US source income. US source income received by a foreign person subject to US tax rate of 30%

Who can hold third party liable?

Generally only principal- even if principal undisclosed

Exemption: Birth or death during the year

Get full exemption; not prorated

500 threshold only applies to

Goods contracts (200 land contract must be in writing, 400$ 3 year service contract must be in writing)

AMT vs regular tax

Greater is the total tax liability

Standard deduction for a dependent

Greater of 1. 1050 2. Earned income + 350

Penalty for "willful or reckless" conduct: fraud; no good faith

Greater of 5000 or 50% of income the preparer derived with respect to the tax return or refund claim

Maximum amount can deduct for depletion on tax return

Greater of cost or percentage depletion method

Corporation: basis of property (Corporation receives)

Greater of: 1. Adjusted basis (NBV) of the transferor/shareholder (plus any gain recognized by the transferor/shareholder) 2. Debt assumed by the corporation (transferor may recognized gain to prevent a negative basis)

What is the basic tax formula?

Gross Income (Adjustments) Adjusted Gross Income Standard Deduction OR Itemized Deductions (Exemptions) Taxable Income Federal Income Tax (Tax Credits) Other Taxes (Payments) Tax Due OR Refund

Bargain purchases from employer

Gross income

Individual Income Tax Formula

Gross income < Deductions for AGI > = AGI < Deductions from AGI (greater of itemized or standard deduction) > < Exemptions > = Taxable Income x Tax rate = Gross tax liability < Tax credits and prepayments > = Tax due or refund

Unrelated business income

Gross income from any unrelated trade or business regularly carried on, minus business deductions directly connected therewith *Not substantially related to the organization's tax exempt purposes. TAXABLE

Johnson worked for ABC Co. and earned a salary of $100,000. Johnson also received, as a fringe benefit, group term-life insurance at twice Johnson's salary. The annual IRS-established uniform cost of insurance is $2.76 per $1,000. What amount must Johnson include in gross income? A. $100,000 B. $100,276 C. $100,414 Incorrect D. $100,552

Gross income includes group-term life insurance premiums carried by his or her employer to the extent that the cost exceeds the sum of the cost of $50,000 of such insurance. Johnson received a $200,000 ($100,000 × 2) policy. This is $150,000 above the tax-exempt amount. Therefore, $150,000 ÷ $1,000 × the $2.76 factor = $414 of taxable income as a result of the benefit from the employer.

Define Gross Income.

Gross income is defined as all income from whatever source derived, unless specifically excluded.

Net rental income or loss

Gross rental income + Prepaid rental income + Rent cancellation payment (FMV) + Improvements in lieu of rent < Rental Expenses > = Net rental income or loss

Calculating maximum allowable deduction for Keogh plan: shortcut:

Gross self-employment income x 20% (shortcut)= maximum allowable deduction

List the types of interest expenses that can and cannot be deducted.

H Home mortgage interest (On up to $1,000,000) interest for home equity is the lesser of $10,000 or the FMV of the property reduced by the amount outstanding. I Investment interest Limited to net investment income P Personal interest NOT DEDUCTIBLE P Prepaid interest Must be allocated over the period of the loan E Educational loan interest IS AN ADJUSTMENT (IT IS NOT AN ITEMIZED DEDUCTION)

MACRS other than real property: Mid-quarter convention

If > 40% of depreciable personal property is placed in service in the last quarter of the year, mid-quarter convention must be used

What amount of insurance premium is includible in gross income?

If a shareholder owns less than 2% of the company, the correct answer would be zero. If they own more than 2% of the company, the insurance premiums paid by the S corporation are fully taxable to the shareholder (and deductible to the corp)

What are the conditions that require a taxpayer to make estimated tax payments?

If both of the following are met: $1,000 or more tax liability If the taxpayer's withholding is less than the lesser of: 90% of current tax, or 100% of last year's tax

Life Insurance Premiums

In excess of 50,000: Gross income

Suretyship must be

In writing: Statute of frauds

Business loans: Cash basis taxpayers deduct interest

In year paid or year to which interest relates: whichever is later

Income from hobby

Include in gross income

Employee achievement awards not in form of tangible personal property

Included in GI

Unemployment compensation

Included in GI

Property & Services Obtained

Income- Basis at FMV

Although chapter 11 is intended primarily for business debtors,

Individual is eligible for relief under 11

Explain the Net Capital Loss Deduction and the carryover rules.

Individual taxpayers realizing a net long- or short-term capital loss may only recognize (deduct) a maximum of $3,000 if the amount realized from other types of gross income. A joint return is treated as one person. If the husband and wife file separately, the loss deduction is limited to half ($1,500). Carry forward is an unlimited amount of time until exhausted. It retains its character as either long- or short-term in future years.

Unreimbursed employee business expenses

Itemized deduction

Difference between a joint venture and a general partnership

Joint venture formed for a single transaction or project or a related series of transactions or projects

Perfected security interest vs Judicial Lien

Judicial lien have priority if it attached (sheriff seized the property) before security interest was perfected. If security interest perfected before judicial lien attached, it has priority

179 deduction- ordinary business income or k-1

K-1

Keogh Contributions

K-1 separately stated items

Describe the limitations to Keogh Plans.

Keogh plans are for self-employed taxpayers. Deduction is limited to: $49,000 or 25% of net self-employed earnings The maximum annual contribution cannot exceed the amount deducted. Limited to the lesser of $49,000 or 100% net earnings if compensation is less.

Basis of property acquired by gift when it is subsequently sold at a loss:

Lesser of 1. Donor's basis 2. Value at the time of the gift

Net earnings from self employment goes to

Line 12: 1040

What is required to prove Rule 10b-5, antifraud provisions, which is fraud in connection with the purchase or sale of any security?

MAIDS (Material Misrepresentation, Actual, Induced, Damages, Scienter)

AMT exemption amount

MFJ: 84500 - (25 % x (AMTI - 160900)) Single: 54300 - (25% x (AMTI - 120700)) MFS: 42250 - (25% x (AMTI - 80450))

Unreimbursed employee business expenses: travels, meals, lodging

MUST BE OVERNIGHT. All expenses incurred for meals and lodging while away from home overnight may be deducted Travel: 100% deductible

List some nontaxable miscellaneous items.

Miscellaneous nontaxable items include: Life insurance proceeds Gifts and inheritances Medicare benefits Workers compensation Personal injury or illness award Accident insurance - paid by taxpayer Foreign-earned income exclusion - up to $91,500; must meet tests: Be a bona fide resident of a foreign country for an entire year. Be present in the foreign country for 330 days out of any 12- consecutive month period

ESSP: Option cannot be exercised

More than 27 months after the grant date

Mortgage interest paid on commercial rental property

NOT deductible

How are the four types of capital gains treated for tax purposes?

Net capital gain rules: Long-term: holding-period - more than one year; tax rate - 15% is max, 0% if taxpayer is in 10 - 15% income bracket Short-term: holding-period - One year or less; tax rate - treat as ordinary income. 1250 Gain: if not treated as ordinary income tax at 25% Collectibles/Business Stock: Long-term gains on this type are taxed at 28%.

Inventory: Gross profit=

Net sales + ending inventory - beginning inventory - inventory purchases

If principal undisclosed, apparent authority?

No apparent authority, no ratifying

Corporation formation: no gain recognized

No boot received/liabilities don't exceed basis

Lost, misplaced, broken property

No casualty loss deduction

Explain the tax treatment of Related Party transactions.

No deduction is allowed for losses on sales to related parties. Basis Rules: Sell Higher Use Relative's basis to determine gain Relative's Basis ------------------ Sell Between No gain or loss Lower purchase price by relative -------------- Sell Lower Use Purchase price to determine loss The holding period starts with the new owner's period of ownership.

State the type of Personal Loss that is nondeductible.

No deduction is allowed for the loss on a non-business disposal or loss.

Purchase of asset: Income of partnership & partner's basis

No effect

Sale price to unrelated party between original cost basis and related party purchase price

No gain or loss recognized

Parent/subsidiary liquidation: Gain or loss

No gain or loss recognized by either parent corporation or subsidiary corporation when parent who owns at least 80% liquidates its subsidiary

Contribution of property to a partnership in return for a partnership interest

No gain/loss recognized

If property declined in value while held by giftor:

No gift tax is added to the basis

SEC Rule 504 can only be used by companies not reporting to the SEC, usually closely-held companies. No registration is required if no more than $1 million of securities are sold within a 12-month period with the following:

No limit on the number of investors No general public offering or advertising No restrictions on resale by the investor If the securities are registered exclusively under a state blue-sky law that allows it, general solicitation and general advertising may be permitted. However, sales are allowed only to "accredited investors."

Preparer required to obtain supporting documentation?

No, unless preparer has reason to suspect the accuracy of the information provided by the taxpayer (client)

State income taxes for business- schedule C business expense?

No. Itemized deduction.

If you have rental homes can you deduct charitable contributions from the rental revenue?

No. You would have your rental revenue and expenses on a schedule E of a 1040 and a charitable contribution isn't a necessary expense of maintaining rental properties. But, you could include the charity as an itemized deduction on the schedule A of a 1040.

Can suspended losses from passive activities be carried back?

No; only forward

What is a non-capital asset?

Non-capital assets are: Property normally included in inventory or held for sale to customers in the ordinary course of business Depreciable property and real estate used in a trade or business Accounts and notes receivable arising from sales or services in the taxpayer's business Copyrights, literary, musical, or artistic compositions held by the original artist Treasury stock

Principal undisclosed: effect on actual authority:

None

Name some nontaxable fringe benefits.

Nontaxable fringe benefits include: Life insurance proceeds Accident, Medical, and Health insurance (Employer Paid) De minimis fringe benefits Educational expenses (up to $5,250) Qualified employee discounts Qualified pension, profit-sharing, and stock bonus plans Flexible spending arrangements

Reduced Homeowner's Exclusion

Number of years of non qualified use/total years of ownership x total gain= Gain not eligible for exclusion

what are 'hot assets' of a partnership?

ONLY inventory and unrealized receivables

Offer revocable unless

Offeree paid consideration to keep the offer open (option)

Regulation A

Offering circular must be filed with the SEC

UCC rules, nonmerchant seller: risk of loss passes to buyer

On tender of delivery

Parol evidence rule- what is allowed?

Oral or written modification made after the contract has been entered into (subsequent modifications)

Retirement plans for non partners

Ordinary business income; not k-1

What are the two types of automatic perfection?

PMSI in consumer goods and small-scale assignments of account

Children of divorced parents- which parent gets the exemption?

Parent who has custody for the greater part of the year. Doesn't matter who provided more than 1/2 of child's support.

If parents have equal custody- who gets exemption?

Parent with higher AGI

Thompson's basis in Starlight Partnership was $60,000 at the beginning of the year. Thompson materially participates in the partnership's business. Thompson received $20,000 in cash distributions during the year. Thompson's share of Starlight's current operations was a $65,000 ordinary loss and a $15,000 net long-term capital gain. What is the amount of Thompson's deductible loss for the period?

Partnership losses are deductible by a partner up to the amount of the partner's basis in the partnership interest. To compute a partner's basis to determine deductible losses, the basis is reduced by distributions first. Beginning of year $ 60,000 Less Distributions - 20,000 -------- $ 40,000 Long-term capital gain 15,000 -------- Basis for allowing deductible loss $55,000

how are passive activity losses treated on an individual tax return?

Passive activity losses are not deductible on an individual income tax return. They are carried over and used to offset passive income in the future until used up. A passive activity is a business in which the taxpayer serves as an owner but does not materially participate in the operation. Rental activities and limited partnerships are also included in this category, regardless of the owner's participation.

AMT requires which contract method

Percentage of completion

Statute of limitation- who's signature are you looking for?

Person being sued. Other party's signature is not needed

Perfected PMSI vs perfected PMSI

Priority goes to the creditor who was first to either file or perfect

If property's FMV is less than liability assumed:

Property's FMV assumed to be amount of liability assumed by shareholder

Regulation crowdfunding: form C

Provides a transaction exemption for issuances of securities made through a crowdfunding process (widespread internet solicitations of small amounts from numerous investors)

What special type of security interest has priority over all other types of security interests in the same collateral

Purchase Money Security Interest (PMSI)

a creditor sells an asset to a debtor on credit and retains a security interest in the asset (collateral) for the purchase price

Purchase Money Security Interest (PMSI)

Repair and maintenance of wheelchair

Qualifying medical expense

MACRS 3 year class

Racehorses more than 2 years old and other horses more than 12 years old, special tools

Which four groups of people are not allowed to file CH 7?

Railroads, insurance companies, banks, savings institutions

Who can't file for bankruptcy under chapter 7?

Railroads, savings institutions, insurance companies, banks, small business investment companies

Syndication costs

Raising money (offering materials). Not deductible.

Section 1250

Real business property: over 12 months (warehouse) Gains only

What contracts are governed by common law (RISE) ?

Real estate, insurance, Services, Employment

1250 recapture only applies to

Recognized gains

a simplified form of registration designed to allow small companies to make public offerings more quickly and with less cost than is required using full registration

Regulation A (Simplified Filing)

When capitalizing: warranty

Remove

Non residence

Rental property

Partnership taxable year

Required to have same taxable year as its majority (over 50%). If no tax year can be established, use year end with least income deferral. Earliest month

Correspondence audit

Review for mathematical errors

how are social security benefits taxed in the income is relatively low?

SS benefits are not taxed if the income is low. 50% becomes taxable if the income is higher, and up to 85% can be taxable if the income level is relatively high

Where do Sole Proprietorship profits or losses go?

Schedule C of Owner's 1040

Net income from self employment

Schedule C: transferred to 1040. Gross business income < Business expenses > = Profit or loss

Are scholarships and fellowships included in income?

Scholarships and fellowship grants are excludible only up to the amounts actually spent on tuition, fees, books, and supplies (not room and board) provided: The grant or scholarship is made to a degree- seeking student; No services are to be performed as a condition of the grant or scholarship; and The grant or scholarship is not made in consideration for past, present, or future services of the recipient.

What is the only way that PMSI in consumer goods does not automatically win (after HDCs and ordinary buyers)?

Secondhand consumer purchaser without notice of PMSI (garage sale rule; this would be beaten if the original buyer simply filed financing statement)

C Corp

Separate**** taxpaying entity that reports income on form 1120

Depreciable basis determined

Separately from gain/loss basis

MACRS 15 year class

Sewage treatment plants, telephone distribution plants, qualified improvement, restaurant and retail property, two way exchange of voice and data communications

MACRS 20 year class

Sewer pipes

Stock dividends generally not taxable unless:

Shareholder has a choice of receiving cash or other property. Value of taxable stock dividend is FMV on distribution date

Bought shares for cash

Shareholder has no gain

Holding period and related parties

Starts with new owner's period of ownership

What is the order for SAG WEG CTI?

Support, Administrative expenses, Gap Claims, Wage Claims (up to 13,650), Employee Benefits (up to 13,650), grain farmers, Consumer Deposits, Tax Claims, Injury Claims

Exoneration

Surety compels the principal debtor to pay the creditor and the surety himself doesn't pay

Indemnification

Surety entitled to reimbursement from his principal debtor for any amount the surety paid on behalf of the debtor

Cosureties: after surety pays:

Surety is entitled to contribution from his cosurities for their share of the payment

Surety vs. Guarantor

Surety: Directly liable on his contract Guarantor: Liable to the creditor only if the debtor doesn't perform his duty to the creditor

A security interest can't be perfected before it attaches to the collateral, but attachment and perfection can occur at the same time through

Taking possession of the collateral

Accumulated Adjustments Account (AAA)

Tax effects of distributions paid to shareholders of an S Corp that has accumulated E&P since inception are computed using AAA. AAA is zero at the inception of an S Corp

Section 501 (c) (3)

Tax exempt organizations. Private foundations and public charities

Corporation: no penalty for underpayment if

Tax for the year is less than 500

Partnership: Property subject to excess liability

Taxable event. Excess of liabilities assumed by other partners over the contributed basis is treated as taxable boot and a gain to the partner

Fringe benefits

Taxable included in gross income

Safe harbors: routine maintenance 1. Qualifying Small Taxpayer expense costs if 2. Qualifying small taxpayer 3. Eligible building

Taxpayers expense routine maintenance that they expect to occur more than once and doesn't result in betterment 1. Expense costs if they don't exceed lesser of 2% unadjusted basis of building or 10,000 2. Average annual gross receipts of 10 million or less during last 3 years 3. Unadjusted basis does not exceed 1 million

What is the Child Tax credit?

Taxpayers may claim a $1,000 tax credit for each "Qualifying Child." Refundable to the extent of the lesser of: Excess child tax credit (over tax liability); Earned income less $3,000 times 15%.

List the taxpayers that are subject to the passive activity loss rules.

Taxpayers that are subject to passive activity loss rules include: Individuals Estates Trusts Personal service corporations Closely held C corporations

who enforces anti-trust laws?

The DOJ's anti-trust division and the FTC

Discuss Adjusted Basis of Asset sold.

The adjusted basis of an asset sold is calculated for three different situations: Purchased property Gifted property Inherited property

List the basic formula for determining a gain or loss on a disposition of property.

The basic formula is: Amount realized (Adjusted basis of assets sold) Gain or loss realized

What is the basic formula to determine net rental income or loss?

The basic formula is: Gross rental income Prepaid rental income Rent cancellation payment Improvement in lieu of rent (Rental Expenses) Net rental income or loss

Implied warranty of fitness for a particular purpose

The buyer relies on any seller (doesn't need to be a merchant) to select goods suitable for the buyer's particular purpose. Seller must know particular purpose and the buyer is relying on him to select the goods

How are worthless stocks and securities treated under the tax code?

The cost (or other basis of worthless stock or securities is treated as a capital loss, as if they were sold on the last day of the taxable year in which they became totally worthless.

What is the Lifetime Learning Credit?

The credit is available for an unlimited number of years for qualified tuition and related expenses. Equal to 20% of qualified expenses up to $10,000.

American Opportunity Tax Credit (AOTC):

The credit is equal to 100% of the first $2,000 and 25% of the next $2,000. Max credit=2500 *per student* (2) Students must be enrolled no less than half-time during at least one semester during the year. (3) For taxable years beginning in 2016, a taxpayer's modified adjusted gross income in excess of $80,000 ($160,000 for a joint return) is used to determine the reduction in the amount of the credit otherwise allowable.

Under Article 9 of the Uniform Commercial Code (U.C.C.), three things are required for "attachment" to occur:

The debtor must have signed a security agreement or the goods must be in the possession of the creditor, The creditor must have given "value" (i.e., consideration to support the contract, e.g., goods on credit given to buyer) to the debtor, and The debtor must have rights in the collateral. The filing of a security agreement relates to "perfection," not "attachment."

Which of the following is an advantage of forming a limited liability company (LLC) as opposed to a partnership?

The owner may participate in management while limiting personal liability.

What is the percentage depletion rate allowed by the Internal Revenue Code for the recovery of capital invested in mining coal? A. 5% B. 10% Incorrect C. 15% D. 20%

The percentage depletion rate for coal mined in the United States is 10%. Some types of mining for clay are allowed a 5% depletion rate. The 15% rate for mining includes gold, silver, copper, and iron ore.

Are state and local refunds taxable?

The receipt of a state or local income tax refund is not taxable if the taxes paid do not result in a tax benefit in the prior year. Itemized deductions prior year - taxable Standard deduction prior year - nontaxable

List the requirements for a "qualifying relative" exemption.

The requirements are: Support test Under exemption amount (Income) Precludes dependent filing a joint return Only citizens: US, Mexico, or Canada Relative Taxpayer lives with individual for entire year.

Who must sign a financing statement for it to be valid?

debtor

If there is record of a security agreement, it must be signed by the _________

debtor (not creditor)

What is a preferential payment?

debtor finding a way to give the creditor more than they would have by going through the normal bankruptcy process

For which of the following is a partnership recognized as a separate legal entity?

The status of the partnership as an employer for workers' compensation purposes

Explain the Uniform Capitalization Rules.

The uniform capitalization rules provide guidelines with respect to capitalizing or expensing certain costs. The apply to: Property produced for use Property produced to sale Property produced for resale Costs that should be capitalized are: Direct materials, direct labor, and factory overhead Costs that should not be capitalized are: Selling, general, administrative, research & development expenses

List the exceptions to the premature distribution penalty.

There is no penalty if the premature distribution was used to pay: H Homebuyer: $10,000 maximum exclusion if the distribution is used toward the purchase of a first home within 120 days of the distribution. I Insurance (medical) - Unemployed with twelve consecutive weeks of unemployment; self-employed. M Medical expenses in excess of 7.5% of AGI D Disability E Education: College tuition, books, fees, etc. A and D Death

What types of losses are nondeductible?

These losses are nondeductible: Wash sale loss Related party transactions And Personal loss

Third parties: If principal was undisclosed or partially disclosed

Third party can hold either the principal or the agent (but not both) liable

Under the U.C.C., even if a sale is made with a disclaimer of "any and all warranties," the seller still makes implied warranties relative to

Title. Title warranties can be disclaimed only by "specific language or by circumstances" which clearly indicates to the buyer that there is a disclaimer of the warranty of title.

Portions of income not allocated entirely to one state are apportioned:

To all states in which the corporation does business. Apportionable items of income are business income.

If an individual paid income tax in Year 1 but did not file a Year 1 return because his income was insufficient to require the filing of a return, the deadline for filing a refund claim is:

To receive a refund for an overpayment of tax, a taxpayer must file a claim for refund within three years from the date on which the tax return that relates to the refund was filed, or within two years of actual payment of the tax if that date is later.

Basis of common stock received from the corporation to shareholder is:

Total of 1. Cash (amount contributed) 2. Property: Adjusted basis (NBV). Reduced by any debt on property assumed by the corporation 3. Services: FMV, taxable. Shareholder receiving common stock for services must recognize FMV as ordinary income

characteristics of 1231 assets?

Trade or business assets owned for MORE than a year << this is the ONLY classification of a 1231 asset

If 80% test not met, property transfer

Transfer is taxable. FMV is basis to corporation

Of the various administrative sources available from the IRS, which is held to be the strongest and most important source? A. Revenue Rulings B. Revenue Procedures Correct C. Treasury Regulations D. Letter Rulings

Treasury Regulations have the most weight in interpreting the IRS Code. Regulations are the interpretation of the Code by the government department responsible for administering the income tax laws. When regulations are challenged, courts will only invalidate a regulation if it does not follow the intent of Congress. Revenue Rulings are the second most important source of administrative law. Revenue Rulings are interpreted as the position of the IRS. They are much easier to challenge in court than Treasury Regulations. Revenue Procedures are often administrative in nature, such as announcing the adjustments for inflation for tax brackets, personal exemptions, and standard mileage rates. Letter Rulings are issued in response to a request from a taxpayer for a ruling on his or her personal tax situation. These cannot be cited as authority by anyone else.

Examples of a fiduciary

Trustee of a trust Executor of an estate

Who is allowed a NOL deduction?

Trusts and estates

Cosureties:

Two or more sureties of the same obligation

UCC Sales Article that governs contracts for the sale of goods

UCC Article 2

US Circuit Court of Appeals hears appeals from

US Court of Federal Claims

In what order are the following obligations paid after a secured creditor rightfully sells the debtor's collateral after repossession? Debt owed to any junior security holder Secured party's reasonable sale expenses Debt owed to the secured party

Under Article 9-504 of the Uniform Commercial Code (U.C.C.), obligations are paid in the following order when a secured creditor repossesses and sells the debtor's collateral (when the debtor is in default) in a commercially reasonable manner: first to pay expenses incurred in selling the collateral, then toward the debt owed to the secured party, and next to any junior or inferior secured parties with rights in the collateral.

Casualty loss- events must be

Unexpected

What are the criteria for filing Qualifying Widow(er) with dependent child?

Unmarried at the end of the tax year, and Is a surviving spouse that must maintain a household, which for the entire taxable year was the principal place of abode of a son, stepson, daughter, or stepdaughter, and As the surviving spouse is entitled to a dependency exemption for the child The taxpayer qualifies for this status for two years after year of the death of spouse.

Gratuitous surety

Unpaid surety. Bound if they make the promise to act as surety before consideration flows from the creditor to the principal debtor

Identify some miscellaneous expenses subject to the 2% AGI floor.

Unreimbursed expenses Educational expenses not taken as an adjustment Uniforms Business gifts ($25) Business use of home Employment agency fees Investment expenses Subscriptions to professional journals Tax return preparation fee

Tax-exempt organizations are not taxed on investment income derived from investments that are accepted as proper sources of income for a charity or trust. Which of the following types of income would be taxable income for a nonprofit?

Unrelated business income (UBI) is net income derived from: (1) the regular operation of a business activity that is unrelated to the organization's exempt purpose (includes earnings from ownership in an S corporation) or (2) debt-financed property.

Rejection not effective

Until received

Employer Payment of Employee's Educational Expenses

Up to 5250 excluded from GI

Discuss Divorced Property Settlements.

When a divorce settlement provides for a lump-sum payment or property settlement, it is a non-taxable event. The basis for the property to the recipient spouse will be the carryover basis.

Automatic stay

When bankruptcy proceedings commence, creditor may not pursue remedies against the debtor or his assets. Goes into effect the moment a bankruptcy petition is filed

Traditional deductible IRA distributions

When person retires, funds taxed as ordinary income when received

Which of the following organizations would not qualify for exemption from federal income tax?

Which of the following organizations would not qualify for exemption from federal income tax? A. College alumni association Incorrect B. Social clubs that allow only limited usage by general public C. Fraternal society not operating under a lodge system D. Political organization Under IRC Section 501(c)(10), domestic fraternal societies must operate under the lodge system to be exempt from federal income taxation. College alumni associations generally qualify for exemption from federal income tax under IRC Section 501(c)(3). If it does not meet the characteristics required by IRC Section 501(c)(3), it may still be exempt as a social club if it meets the requirements described by IRC Section 501(c)(7). A social or recreation club under IRC Section 501(c)(7) is allowed to receive up to 35% of its gross receipts from sources outside of its membership without losing its status as a tax-exempt organization. The club must have an established membership, which must be limited in some manner. Under IRC Section 527, a political organization is considered a tax-exempt organization. It is subject to tax only on nonexempt income. Examples of exempt income include contributions, membership dues, and proceeds from a political fund-raising event.

claims with a "realistic possibility" of being sustained need not be:

disclosed. these are claims that have between a 33% and 50% chance of being sustained

UCC risk of loss: not determined by

Who has title

can an S corp be owned by a bankruptcy estate?

Yes. it can also be owned by decedent's estates, or trusts.

Under the Affordable Care Act, for 2016, what is the amount for the "shared responsibility payment" on one uninsured adult?

You answered A. The correct answer is B. Under the Affordable Care Act, taxpayers who do not have health insurance are now required to pay a "shared responsibility payment." The fee is calculated two different ways, and taxpayers must pay the higher amount: 2.5% of household income; or $695 per adult and $347.50 per child under 18, with a family maximum of $2,085.

If you inherit stock and sell it within a year is it a short-term or long-term gain?

You assume the holding period of the donor. So if they had only had it 6 months you start at 6 months when you inherit it- if they had it 5 years and you sell it the day after you inherit it, it is still a long-term gain

when you contribute cash and property to a partnership in exchange for a partnership interest, how is your basis in the partnership calculated?

Your cash gives you basis in the amount of the cash.You take your basis in the land you contributeLESS a mortgage on itADD back your % of the partnership in the mortgageADD your % of recourse liabilitiesThe sum of these equal your basis

If a partner dies, according to section 30 of the Uniform Partnership Act (UPA), the partnership is:

dissolved but not terminated, since the partnership must continue until the winding up of partnership affairs is completed.

what type of standard for tax shelters has the IRS adopted?

a "more likely than not" standard- meaning if the preparer thinks it is more likely than not that the position will be upheld- they don't need to worry about the tax return preparer penalty

for interest on a series EE bond to be deductible, the person you spend it on must be:

a dependent of yours

what is a type A reorg?

a merger or consolidation under state law. it is stock for asset, the target corp dissolves

what are the rules for using real estate losses to offset ordinary income?

a natural person can offset up to 25k of nonpassive income with passive losses resulting from rental activities. You have to own at least 10% of the rental activity, and you must have actively participated.Also, the 25k allowance is reduced by 50% of the amount that the taxpayer's adjusted gross income exceeds 100k.So if you have 200k in income, you have 100k over the 100k limit, and you have to reduce your losses by half of that amount, or 50k.

for tax purposes, what will cause an end to a partnership?

a partnership terminates when it no longer does business as a partnership or if 50% or more interest in partnership capital and profits is exchanged within 12 months.

a merchant's firm offer has to remain open for:

a reasonable time

what happens if an S corp has excessive passive income? (passive income exceeds 25% of the corp's gross receipts)

a tax at the highest corporate rate is imposed on the excessive passive income

Regulation D. This regulation is a combination of the private placement and small issue exemptions. Issuers must file Form D, which lists minimal information, with the SEC. Regulation D has three separate thresholds with differing requirements:

a. *SEC Rule 504*. This rule can only be used by companies not reporting to the SEC, usually closely held companies. No registration is required if no more than $1 million of securities are sold within a 12-month period with the following: (1) No limit on the number of investors (2) No general public offering or advertising (3) No restrictions on resale by the investor (4) If the securities are registered exclusively under a state blue-sky law that allows it, general solicitation and general advertising may be permitted. However, sales are allowed only to "accredited investors." b. *SEC Rule 505*. No registration is required if no more than $5 million of securities are sold within a 12-month period with the following: (1) No general public offering or advertising (2) No more than 35 unaccredited investors and an unlimited number of accredited investors (3) No required disclosures to accredited investors (4) Distribution of audited financial statements to nonaccredited investors (5) Restricted resale by the investor c. *SEC Rule 506*. No registration of securities is required for a private placement of an unlimited amount of securities as follows: (1) To any number of accredited investors (2) To up to 35 unaccredited but sophisticated investors experienced in financial matters and able to evaluate risks involved in the investment (3) With no general public offering or advertising (4) Restricted resale by the investor (5) Distribution of audited financial statements to nonaccredited investors (6) Under Rule 506(c), a company may advertise the offering if: (a) the investors are all accredited and (b) the company has taken reasonable steps to verify its investors are accredited investors. (7) Sales by dealers of securities that have been registered and issued: (a) 90 days after offering to the public if the securities are the initial offering by the issuer or (b) 40 days after offering to the public.

under the UCC, the party to whom a check is present for payment is the:

drawee. this is the bank.

defense that arises due to unlawful use of threat or harm

duress

If specific performance is available, the non-breaching party can receive _____ , __________, or ____________ (specific performance, compensatory damages, both)

either or but not both

Who is able to deduct for FUTA?

employer not employee (deductible as ordinary business expense)

how do noncorporate shareholders treat gains on a redemption of stock that qualifies as a partial liquidation?

entirely as a capital gain

Unless there is a different agreement, profits in a partnership are shared __________

equally

for an S corp, what all will increase or decrease a shareholders basis?

everything will affect the shareholders basis- even capital gains and losses

What is the one term that must be set for a UCC contract to be enforceable?

acceptance. Without acceptance it's still just a negotiation

Under the UCC, shipment of non-conforming goods is considered to be ________ and ____________

acceptance; breach of contract

an agreement to substitute one contract for the other, and the execution of this agreement

accord and satisfaction

characteristics of ORDINARY ASSETS?

accounts and notes receivableinventorytrade or business assets OWNED FOR A YEAR OR LESS

an investor such as an institutional investor, a bank, officers or directors of the issuer, or an individual with a four-year average net worth of $1,000,000

accredited investor

what is a type c reorg?

acquisition of "substantially all" of the assets of the target solely in exchange for voting stock of the acquiring corp

_______ notice must be given by a principal to old customers to terminate apparent authority after end of agency relationship

actual

authority agent reasonably that he possesses because of the principal's communications to the agent

actual authority

In non-carrier cases, where the seller is a merchant, risk of loss passes to the buyer on the seller's _____________

actual delivery

When does the order for relief occur in an involuntary bankruptcy case?

after 20 days if the debtor does not object

if unionized employees go on strike, when must their company hire them back?

after a 'unfair labor practice' strike- not if the strike is over wages

a provision in the security agreement giving the secured party a security interest in property acquired by the debtor after the security agreement is executed

after-acquired property clause

In an agency coupled with an interest, who has the power to terminate?

agent only

a suit that the surety can bring upon the debtor (before surety pays creditor) compelling the debtor to pay if the debtor fails to pay

exoneration

What are the three required elements of an enforceable contract?

agreement (both offer and acceptance), consideration, lack of defenses

Who does the bankruptcy trustee all have priority over?

all creditors except those with prior perfected security interests or judicial liens

what are examples of 1245 property?

all depreciable personal property such as equipment and machinery. under section 1245 recapture, gains are treated as ordinary income up to the amount that was depreciated.

for corporations, how long is goodwill amortized?

amortized over 15 years

what is an example of a defect in the marketable title to real property?

an unrecorded easement

either the buyer or seller indicates in advance of performance that he will not perform

anticipatory repudiation

What method must acceptance be made in?

any method unless the offeror specifies (if you use a method other than what the offeror specified, it is considered to be a counter offer)

Securities issued under Rule 506, for private offerings, may be sold to how many accredited investors?

any number

Chapter 11 of the Bankruptcy Code calls for the reorganization or restructuring of the debt so that the debtor can continue to operate. Bankruptcy under Chapter 11 may result from either a voluntary petition filed by the debtor or an involuntary petition filed by one or more creditors. If there are fewer than 12 creditors:

any one creditor who is owed at least $15,325 (unsecured) may file petition. If there are 12 or more creditors, at least three creditors (owed collectively at least $15,325) must join in the petition. These are indexed amounts and are adjusted every three years.

Who is a defrauder all liable to?

anyone who suffered a loss

power that a 3rd party reasonably believes an agent has, based on principal's conduct or communication toward third party

apparent authority

What must be filed for a corporation to be formed?

articles of incorporation

What must be filed for a limited liability company to be formed?

articles of organization (with state)

when property is conveyed to an owner, whether as a liquidating distribution or not, it is recorded as:

as if it were sold for FMV. if it is a capital asset then there is a capital gain equal to the FMV less the basis.

When is a contract formed in a bilateral contract?

as soon as promises are exchanged

if you inherit stock worth 10k, how much tax do you owe?

assets that are inherited aren't taxable. You take a basis in them that's the same as the fair value on the date of death.

the debtor transfers some or all of his or her property to a trustee, who disposes of the property and uses the proceeds to satisfy the debtor's debts; the debtor is not discharged from unpaid debts

assignment for the benefit of creditors

A corporation must meet both the stock ownership test and the gross income test to be a personal holding company (PHC). These tests are:

at any time during the last half of the year more than 50% in value of its outstanding stock is owned, directly or indirectly, by five or fewer individuals, and at least 60% of its adjusted ordinary gross income is PHC income

how are estimated tax payments calculated?

at least 90% of current years taxes or 100% of last years taxes. BUT- if AGI exceeds 150k, then tax payments during the year must be 110% of last years taxes

A security interest is effective between a debtor and a creditor upon _________

attachment

a C corps net capital losses are carried back:

back 3 years and forward 5. Remember that these are capital losses and not NOLs.NOLs can be carried back 2 years and forward 20

in a corporate formation, gain is recognized to the extent that the liabilities assumed by the corporation exceed the:

basis in the assets contributed by the shareholder.If you put in property worth 20k, you have basis of 6k, and it has a 12k mortgage- your gain is the difference in your basis and the mortgage you're being relieved of... so 6k in this case

how does a suretyship work if 2 people are each 50% but one is released without the other one knowing?

before the one is released, either one could be forced to pay the entire amount, but they would then have the right of contribution to go after 50% from the co-surety. when one is released, the remaining surety is then only liable for 50% total.

Under the UCC can the defrauder sue for damages or rescind the contract?

both

Who funds FICA?

both employee and employer (includes self-employed)

Who contributes to the purchase of affordable coverage under the Affordable Care Act?

both employer and employee

Which four groups of people are not allowed to file CH 11 bankruptcy?

brokers, insurance companies, banks, savings institutions

Qualifying widower with dependent child

-Can file MJ for 2 years following death (unless remarries) -Surviving spouse must pay over half of cost of maintaining household where dependent child lives for the whole taxable year

Head of Household

-Not married, legally separated, can be married and lived apart for last 6 months of the taxable year -Not a qualifying widower - Not nonresident alien -Maintains household that is principal residence of a qualifying person (more than 1/2 the year)- dependent child or parent

Under the UCC secured transactions article, what will ALWAYS prevent a security interest from attaching?

failure of the debtor to have rights in the collateral

What is considered the acceptance in an auction?

fall of hammer

dividends received deduction limits:

0-20%: deduct 70%21-80%: deduct 80%81-100%: deduct 100%

Failure to pay penalty

0.5% of the unpaid taxes for each month or part of a month after the due date, but not more than 25%.

Child's unearned income- tax rate: 1) 0-1050 2) 1051 - 2100 3) 2101 - over

1) 0% 2) Child's: 10 or 15% 3) Parent's

Best Practices for Tax Advisors

1) Communicating with the client regarding the *terms of the engagement* to deterring the client's purpose & use for advice 2) Establishing the *facts* and conclusion *supported by the law and the facts* 3) Advising the client about the *importance of the conclusion reached* (whether able to avoid penalties) 4) Acting *fairly & with integrity* 5) Taking reasonable steps to ensure everyone is consistent with the above

Things to remember when looking at bankruptcy distribution problems:

1- Look at the dates! Priorities change based on the dates

1. Personal property 2. Real property

1. 1245 2. 1250

Tax rates for qualified dividends

1. 15% most taxpayers 2. 0% low income taxpayers (in 10 or 15% ordinary bracket) 2. 20% high income taxpayers (39.6% ordinary bracket)

Expense deduction instead of MACRS depreciation (Section 179 Expense) 1. Limit for new or used personal property acquired from unrelated party during the year 2. Max amount reduced dollar for dollar by property placed in service that exceeds: 3. Deduction not permitted when

1. 510,000 2. 2,030,000 3. Net loss exists or deduction would create net loss

Unified estate and gift tax credit 1. Applicable exclusion amount 2. Estate computes estate tax on tentative tax base at death and then reduces that tax by: 3. Net result generally estate tax due = 4. Deceased spouse unused exclusion

1. 5490000 2. 2141800 3. 40% x (tentative tax base at death - 5490000) 4. Surviving spouse can use it and add it to their credit

Standard Deduction 1. Single 2. HOH 3. MFJ or SS 4. MFS*

1. 6350 2. 9350 3. 12700 4. 6350: Available only if both taxpayer and spouse don't itemize

Percentage ownership: Dividends received deduction 1. 0-20% owned 2. 20% to <80% 3. 80% or more

1. 70% 2. 80% 3. 100%

Annual estate income tax 1. Required when annual income: 2. Tax Year 3. Estimated payments

1. > 600 2. - Calendar year tax return: Due April 15th - Fiscal year tax return: 15th day of the 4th month after year end (Estate elects tax year) 3. Exempt from making estimated payments its first two tax years

Agent's power to bind a principal can arise through

1. A grant of actual authority: power & right - Express authority: oral or written - Implied actual authority 2. Apparent authority or estoppel: power but no right - Principal's conduct caused third parties to reasonably believe that the agent had authority -Principal was negligent and will be estopped from denying that the agent had authority 3. Ratification: power but no right

In order to apply annual exclusion to a gift, gift must be ALL of the following:

1. A present interest 2. Complete 3. Under 14000/28000 per donee (unless paid directly for medical expenses and or education expenses and charities)

100% dividends received deduction applies to

1. Affiliated corporations 2. Small business investment corporations (SBICs): 100% deduction allowed for dividends received by a small business investment company

Partnership: order of distribution of assets

1. Creditors 2. Partners - First return on of their contribution - Divide profit if any

Taxable interest income

1. Federal bonds 2. Industrial development bonds 3. Corporate bonds 4. Part of proceeds from installment sale is taxable as interest 5. Interest paid by federal or state government for late payment of a tax refund

Taxes not deductible (FIB)

1. Federal taxes (including SS) 2. Inheritance taxes for states (federal estate pick-up tax) 3. Business (schedule C) and rental property taxes (schedule E)

Methods of perfection

1. Filing 2. Taking possession of the collateral 3. Control 4. Automatic perfection 5. Temporary perfection

Estate administrative expenses can be deducted

1. Final income tax return of decedent 2. Liability (estate tax return) But not both

Estates subject to 2 taxes

1. Income tax: due annually based on income earned during the year while estate is in existence 2. Estate tax: one time only transfer tax based on value of decedent's estate (taxed to estate before the property is transferred)

Section 509 Private Foundations: types of terminations

1. Involuntary: When private becomes public; termination by IRS if foundation commits repeated violations or flagrant violations 2. Voluntary: notifying IRS of plan to terminate

Partnership tax losses deductible by partners and can be used to offset ordinary income. For partner to deduct losses: losses must clear:

1. Losses limited to tax basis: Capital account + % of liabilities= Basis 2. Losses limited to at risk amount: Similar to tax basis, but DOESN'T INCLUDE CERTAIN NONRECOURSE LIABILITIES 3. Passive loss limitations

1. Long term capital gains tax rate 2. Short term capital gains tax rate

1. More than 1 year holding period. 0%, 15%, 20% 2. One year or less. Treated as ordinary income tax rate

If breach is: 1. Material or substantial 2. Minor

1. Non breaching party is discharged from the contract 2. Non breaching party isn't discharged, but is entitled to damages

S Corp 1. Deductible fringe benefits 2. Nondeductible fringe benefits

1. Non shareholder employees and employee shareholders owning 2% or less of S Corp 2. Shareholders owning over 2 percent is not deductible

Alimony requirements

1. Payments must be made in cash or equivalent (credit card bill) 2. Payments must end at recipient's death

Deductible taxes

1. Real estate: doesn't include street, sewer, and sidewalk assessment taxes 2. State, local, and foreign income tax 3. Property tax: assessed by state and local governments on personal property owned by taxpayer (vehicles and boats) 4. Sales tax

1. Who elects the board of directors? 2. Who elects the officers?

1. Shareholders 2. BOD

3 factors limiting losses for business type activity. Loss may be deducted only if loss if greater than:

1. Tax basis 2. At-risk basis 3. Passive loss limitations

Early withdrawal

10% penalty tax + regular tax if hasn't met exception

how long do you have to own a stock to qualify for dividends received deduction?

45 days

what year class are computers under MACRS?

5 year class

how long is a financing statement effective for?

5 years

Carryover of excess charitable contributions

5 years- FIFO basis, after current year contributions deducted, subject to % of income limitations

Form 10-K must be filed within _______ days of the fiscal year end for small corporations?

90

What is a value-added tax (VAT)?

=A form of consumption tax A value-added tax is a tax passed on to the consumer and an estimated market value added onto a product or material at each stage of the manufacturing process. Unlike the sales tax, which is collected at the cash register, the VAT is imposed at each stage of the production process.

In a sale or return (i.e. a sale where buyer has the rights to return the goods) who holds risk of loss?

buyer gets ROL when seller completes shipment; buyer holds ROL until completely delivered back to seller

What is the order of priority between conflicting interests in collateral?

buyer in the ordinary course and holder in due course, properly perfected PMSI, Non-PMSI, unperfected but attached, debtor

In noncarrier cases, where there has not been any specific agreement as to delivery or risk of loss, how will the goods normally be transferred?

buyer picks up goods at seller's place of business

What is a passive activity?

A passive activity is any activity in which the taxpayer does not materially participate.

The accumulated earnings tax does not apply to the following:

A personal holding company A foreign personal holding company A corporation exempt from tax under Subchapter F A passive foreign investment company

Which of the following taxpayers is required to use a calendar year?

A. A taxpayer that keeps no records Incorrect B. A grantor trust C. A personal service corporation D. An S corporation Taxpayers who keep no records must use a calendar year. Generally, trusts must use a calendar year; however, grantor trusts are not required to do so. Although personal service corporations and S corporations are generally required to use a calendar year, if they meet certain requirements they can elect to have a fiscal year.

Foreign earned income exclusion A. Amounts B. Tests to qualify for exclusion

A. Exclude up to 102100 of foreign earned income. Must satisfy one of two tests: B. 1. Must have been bona fide resident of foreign country for entire taxable year. 2. Physical presence in foreign country for 330 days out of any 12 month period

for a stock split basis question:

figure out the total amount the stock cost originally and multiply that by the different split ratios to get the basis per share when the stock is sold

A corporation must file IRS Schedule UTP if it:

filed Form 1120, 1120-L, or 1120-PC, has assets equal to or exceeding $10 million, issued audited financial statements, and has one or more tax positions that must be reported.

Book to tax reconciliation: charitable contributions, depreciation, other expenses recorded on books but not on tax return

Add back to net income

Partnerships: 754 election Transfer of partnership interest

Adjustment= Value of the outside basis to the transferee partner (purchase price) < His share of the partnership's inside basis of the assets > Basis adjustment is entirely a tax concept and doesn't impact BV of partnership's assets

who has priority when there are two unperfected security interests in the same collateral?

first to attach

Corporate taxable income: Reasonable salaries, office rentals, office supplies, traveling expenses

All deductible when incurred for business purposes

Dividends that represent distributions of corporation's earnings and profits A. Taxable amount 1. Cash 2. Property

All included in GI. A. 1. Amount received 2. FMV

what are the rules for a C corp to use the cash basis of accounting?

c corps cant use the cash method unless their average annual gross receipts for the previous three years do not exceed 5 million. once they pass 5 million they must use accrual method for all future years

Ratification

All or nothing. Doesn't require consideration

If goods are nonconforming, risk of loss:

Always on the seller regardless of the shipping terms

occurs when a party is deceived into signing something that he does not know is a contract

fraud in the execution

occurs when the defrauded party is aware that she is making a contract, but terms are materially misrepresented

fraud in the inducement

liable to the creditor only if the creditor is unable to collect from the debtor after exhausting all legal remedies, including demand, suit, judgement and exhaustion

guarantor of collectibility

For the risk of loss to pass in a contract, the goods must be ___________as the goods for this contract

identified

if a corp has E&P of 9k and it distributes 9k of cash to its sole shareholder, and land with a FMV of 40k and a basis of 5k, what is the shareholder's taxable income because of the transaction?

if a corp distributes appreciated property, its E&P goes up by the difference between FMV and its basis in the property.So in this case, the total E&P is raised from 9k to 44k (9+35). So the shareholder has a gain of 44k and the remaining 5k is a return of capital and is nontaxable.

what causes trust property to be included in the grantor's gross estate?

if the trust is revocable, which means the taxpayer maintains ownership or control of the property

When does the order for relief occur in a voluntary bankruptcy case?

immediately

in anticipatory repudiation, when can the non-breaching party sue?

immediately (or wait until the party is supposed to perform and sue for not fulfulling)

warranty that arises when the buyer relies on the seller to select goods suitable for buyer's particular purpose for the goods

implied warranty of fitness for particular purpose

warranty implied in every sale by merchants that the goods are fit for ordinary purpose

implied warranty of merchantability

warranty implied in every sale that is a promise that seller has good title and a right to transfer it

implied warranty of title

A CPA can charge a contingent fee when helping a client with a claim for a refund filed solely:

in connection with a determination of statutory interest or penalties.

Unless otherwise agreed upon, profits and losses in a limited partnership are shared ____________

in proportion to contribution

Unless there is a different agreement, losses in a partnership are shared __________

in the same manner as profits

what is the half year convention under MACRS?

in the year the asset is purchased, it is depreciated for 6 months no matter when it is actually bought. same thing for the year it is sold

if you estimated your taxes in 2012 as 5k and then in 2013 you found the exact amount to be 6k, you:

include the remaining 1k in your 2013 tax return

If a worker owns their own tools, are they likely an employee or independent contractor?

independent contractor

Which types of businesses can file CH 13?

individuals

Which types of businesses can file CH 11?

individuals, partnerships, corporations

Which types of businesses can file CH 7?

individuals, partnerships, corporations

is interest income on state and federal government obligations included in taxable income?

interest from federal obligations IS taxable but interest from state obligations IS NOT taxable

what items are stated separately for partnership income?

interest incomemunicipal interest incomesection 1231 gaincharitable donationslife insurance premiums on partner's livesordinary income/loss

What is the time called between the involuntary petition and the order for relief called?

involuntary case gap

What is the first step in determining whether a debtor is eligible for CH 7?

is monthly income lower than state median

what is rule 505 of Regulation D of the 1933 securities act?

it allows a securities sale of up to 5mil in 12 months without registering with the SEC. it can only sell shares to a limit of 35 non-accredited investors but unlimited sales to accredited investors

When a regular C corporation converts to S corporation status, a tax may be imposed on the net increase in value that took place on the assets during the time they were held by the C corporation. The tax is imposed on the S corporation when

it disposes of property within five years of the S election.

how are taxes determined when a company is buying and selling its own stock?

it doesnt matter- a company buying and selling its own stock is nontaxable

what is the federal acc earnings tax credit and how does it work?

it gives corps 250k to lower its income subject to the acc earnings tax, plus any dividends or fed income taxes paid.If your corp had 400k in taxable income and it paid 100k in tax, you then subtract the credit of 250k from the remaining 300k- this leaves you with 50k that is subject to the acc earnings tax

how is an interest forfeiture penalty for making an early withdrawal from a certificate of deposit treated?

it is deductible on page 1 of the 1040 to arrive at AGI

how does the foreign income tax credit work?

it's the lower of the foreign taxes paid, or the amount of US taxes that would be paid on the foreign income

co-sureties are _______ and________ liable

jointly; severally

an unrecorded mortgage has priority over a subsequently recorded mortgage if the subsequent mortgagee:

knew of the un-recorded mortgage. If another mortgage company knows about an existing mortgage- even if it's unrecorded, the unrecorded mortgage will take priority.

In determining whether the consideration requirement to form a contract has been satisfied, the consideration exchanged by the parties to the contract must be:

legally sufficient

Consideration must be:

legally sufficient. This doesn't mean fair or equal.

if a debtor is adjudged to owe a creditor money and the judgement has gone unsatisfied, the creditor can request the court to impose a ________ on specific property owned and possessed by the debtor

lien

How long does a sole proprietorship last?

life of owner (can be terminated earlier)

Partnership made up of one or more general partner and one or more limited partner

limited partnership

What is CH 7 Bankruptcy?

liquidation

What are the four duties an agent always owes a principal (LORA)?

loyalty, obedience, Reasonable care, account

for dividends rec deduction questions, when the income is a net loss:

make sure to deduct it from the income amount before applying the DRD.-20k in net income with 200k of 25% owned dividends = a DRD of 80% against 180k - NOT 200k

A Chapter 11 reorganization case

may be commenced either voluntarily or involuntarily, but does not require the appointment of a trustee or the insolvency of the debtor if a voluntary case. The reorganization plan may be filed by a creditor or creditors' committee only 120 days after the petition date if the debtor or trustee has not filed a plan.

under common law, someone who works on property and either repairs it or improves it automatically has a lien on the property, for the price of the repairs

mechanic's lien

written promises by merchants to keep the offer open; irrevocable for the time stated

merchant's firm offer

how are unreimbursed employee expenses treated?

misc itemized deduction on sched A subject to a 2% of AGI threshold

what percentage of ownership must consent to revoke the S status of an S corp?

more than 50%

what constitutes a personal holding corporation?

more than half of the value of the outstanding stock must be owned by five or fewer individuals at some point during the last half of the year. at least 60 percent of its ordinary income must be generated by dividends, interest, rents, royalties, and other passive income

defense where both parties were mistaken as to a basic assumption about the contract; if both parties are unaware, the contract is void

mutual mistake

What is all required to be included in the articles of incorporation?

name, corporation's registered agent, names of incorporators, number of shares authorized

Permanent differences between taxable income and pre-tax accounting income affect

neither interperiod nor intraperiod income tax allocation inter is for temporary differences, intra is apportionment to i/s sections.

Self-employed FICA tax is imposed on _______

net profit

when a C corp makes an S election and it has unrealized built-in gains in its assets as of the election day, they must pay a built-in gains tax on this appreciation if it is recognized within the:

next 10 years. That means if it is sold and the gain is recognized it has to pay a built-in gains tax on the appreciation at 35%

Are advertisements considered to be offers ?

no

Under the Sales Article, does it matter whether the breach is material or minor?

no (non-breaching party can be discharged no matter what)

Is an agency relationship required to be in writing?

no (not generally)

Is there a trustee appointed for CH 11 Bankruptcy?

no (not required but court may appoint)

Is minority a defense for the non-minor party in a contract?

no (only for minor)

Is an employer liable for the torts of an independent contractor?

no (only if employer authorized tortious act or work involved ultra-hazardous act)

Is fraud by the principal debtor a valid defense for the surety against the creditor?

no (only if the creditor knew about the fraud)

are cash gifts included in taxable income?

no- cash received as a gift is not included in federal taxable income, although in some situations a gift tax may apply

can incidental beneficiaries sue to enforce contracts?

no- the parties to the contract don't have their benefit in mind when they create the contract

are personal life insurance premiums deductible?

no- they are considered a personal expense

what amount of a net capital loss can a corp deduct in one year?

none- capital losses are only used to offset capital gains. they can be carried back 3 years and forward 5

IRC Section 357(a) addresses the liability that is transferred to a new corporation, along with a building, the new corporation If an acquiring corporation (Ace) assumes a liability in a Section 351 transfer, then the liability is

not treated as boot and no gain will be recognized by the contributor (Lind).

when a company owns 80% or more of another, when are gains reported for tax purposes?

not until the gain is realized by a sale to an outside party.

what does a principal have to do to terminate residual authority of one of its agents?

notify all thee agent's known customers and publish the termination in appropriate trade journals

occurs when a new contract substitutes a new party for an old party in an existing contract and all of the parties agree that the old party is immediately released from the contract

novation

is interest on United States treasury bonds taxable?

nterest revenue on United States treasury bonds is taxable on a federal income tax tax return. Interest revenue on US Series EE bonds is tax exempt but only when it meets specific rules: the bonds must have been purchased by the taxpayer who was over 24 years old and the proceeds must be used to pay college costs for taxpayer, spouse or a dependent. Interest revenue on state and municipal bonds are always tax free on a federal return.

When is a contact formed in a unilateral contract (one promise only ) ?

once performance is complete

how much of the self employment tax is allowed as a deduction FOR AGI? what form does it go on?

one half. it would go on the form 1040

When does a common shareholder get paid their dividend?

only after all cumulative dividends have been paid

which of these are included in the gross income of the recipient during a divorce: alimony, child support, and property settlement?

only alimony. alimony is included in the recipient's gross income, but child support and property settlements are not

if you are the beneficiary of a life insurance policy which is collateral for someone who owes you money, how is it taxed?

only the amount that you get paid above what you are owed is taxable. If they owe you 100k and the policy pays you out 120k, then the 20k is taxable

which of these are qualified organizational expenses for a corporate startup:-legal fees-state incorporation fees-commissions paid to broker on sale of stock

only the state incorporation fees and the legal fees. the broker fees aren't deductible at all.

in a partnership distribution- when is gain recognized?

only when cash exceeds the basis in the partnership interest

a distinct contract in which the the promisor promises to keep an offer open in exchange for consideration from the promisee

option contract

foreign currency exchange gains or losses in the course of normal business are ______

ordinary

The employer's contribution for FICA is deductible as _______

ordinary business expense

what are the 3 elements of deductible business expenses?

ordinary, necessary, and reasonable in amount

prohibits the admission of prior oral or written evidence or contemporaneous oral statements to contradict the terms of a fully integrated written contract

parol evidence rule

if third party does not know agent is an agent or does not know principal's identity, agent is liable under contract along with the principal

partially disclosed/undisclosed principal

what is the loss deductibility exception dealing with losses from rental activity?

passive losses on rental activities are deductible up to $25,000 as long as the owner is an active participant in the management. The deductibility of this $25,000 is lost gradually if the taxpayer's income is especially high. Passive losses are carried forward indefinitely so that they can reduce future passive activity gains.

If you have two perfected non PMSI interest battling for priority who wins between someone who perfected by filing and someone who perfected by taking possession?

person who filed (filing always trumps in this situation)

what is section 1245 property?

personal decpreciable equipment used in a business. equipment and machinery fall into this category

Who needs to have capacity in an agency relationship, principal or agent?

principal (agent can be a minor an incompetent)

What are the two parts of the registration statement filed with the SEC for securities?

prospectus and detailed information regarding securities

a written request for permission to vote a shareholder's shares at a shareholder meeting

proxy

a remedy that allows a plaintiff to recover a benefit unjustly conferred upon the defendant

quasi-contract

allows a principal to choose to be bound by a previously unauthorized act of his or her agent

ratification

What property is all included in the bankruptcy estate?

real and personal property plus divorce, interest, inheritance, and insurance (within 180 days after filing)

In a fraud case, the CPA is generally liable to all:

reasonably foreseeable victims of the misstatement

In an offer, the revocation by the offeror is generally effective when____________

received by offeree

What is a CH 11 Bankruptcy?

reorganization

a judicial action seeking the transfer of personal property from the debtor to the secured party

replevin

employer can be held liable for employee's torts committed within the scope of employment

respondeat superior

If the buyer rejects the goods, what happens to the title ?

reverts to seller

if one person has reason to believe that the other will not perform, he or she may make a written demand for an assurance

right to demand assurances

What is the order of distribution of a debtor's estate?

secured creditors, priority creditors (9 categories SAG WEG CTI), unsecured creditors

What happens if the debtor is a non-consumer and the creditor chooses to retain the collateral for possession?

secured party can collect any deficiency (between collateral proceeds and amount of total debt)

If the seller ships non-conforming goods, who bears the risk of loss on shipment back?

seller

in a sale on approval (i.e. a sale with a trial period) who bears the risk of loss and title ?

seller (until buyer gives approval)

What are the requirements to register with the SEC under the Securities Act of 1934?

shares traded on national exchange, $10 Million in assets and at least 2,000 shareholders, $10 Million in assets and at least 500 unaccredited shareholders

is interest expense on a loan used to purchase municipal bonds deductible?

since the loan is used to purchase bonds that generate tax-exempt income, the interest expense is NOT deductible

what is a type C reorg?

stock for asset. doesn't have to be statutory. Only voting stock and can used by acquiring. Boot is allowed but it cannot exceed 20% of the consideration provided by acquiring. Acquiring must acquire substantially all of target's assets(90% of net asset value and 70% of gross asset value)

what is a B reorg?

stock for stock, and only the voting stock of the acquiring firm is permitted

what is a type B reorg?

stock for stock. Acquirer must own at least 80% of target after the transaction. Only voting stock can be used by acquiring. No boot allowed.

What is the purpose of an automatic stay?

stops all collection efforts by creditors

Under Worker's Compensation, employers are _________ liable

strictly (meaning they are liable regardless of who is at fault, as long as injury occurred in ordinary course of business)

a suit the surety can bring upon the debtor after the surety has already paid the creditor to enforce any right that the creditor had against the debtor

subrogation

one who agrees to be liable for the debt or obligation of another

surety

who are the three parties involved in a suretyship?

surety, debtor, creditor

what is the primary thing to win a section 11 claim?

that there was a material misstatement in the registration statement on the effective date, that they can trace their shares to that registration statement, and that they suffered damages

what entity handles ethical complaints that carry national implications

the AICPA

Contributions to *Coverdell education savings accounts* must be made before:

the account beneficiaries are 18 years old (Payments of 2000 per year per beneficiary) Contributions may be made up to due date of return.

The accounting income of a trust is

the amount an income beneficiary is entitled to receive from the trust. Accounting income includes both taxable and nontaxable items of income.

what is the tax treatment when there is a like-kind exchange with boot?

the amount taxed is the lesser of the boot received or the gain.

how is a like-kind exchange taxed when no cash is received?

the basis of the new asset is the same as the basis in the asset given up plus any cash paid, and no gain or loss is recognized

What is considered the offer in an auction?

the bid

what are the requirements for EE bond interest to be tax free?

the bonds must be issued after December 31, 1989, be purchased by the sole (or joint owner) and the buyer must be at least 24 years old when bonds are purchased. The proceeds of the bonds must be used to pay the higher education tuition and fees for taxpayer, spouse, or dependent.

the person who goods must be delivered to under a negotiable bill of lading

the consignee of the bill of lading

what are the tax implications to a corp and the individual if a corp has land that it bought for 20k but is now worth 25k, and the corp distributes it to an individual? (the corp still owes 10k on the land and the individual assumes the debt)

the corp would report a gain of 5k, and the individual reports a gain of 15k. the individual is 15k better off than before the transaction.

how are incorporation fees expensed?

the first 5k can be deducted in the first year, the rest is amortized over 180 months

how are short and long term capital gains and losses treated by an individual?

the first step is to net the short term gain and loss and the long-term gain and loss.Then, you net the two together. A net capital loss of up to $3k can be deducted in any year in arriving at AGI, and any loss over that can be carried forward indefinitely.Also, when netting the overal gains and losses together, whatever is bigger determines whether it's a long term or short term

generally a lessor will not be taxed on improvements made by a lessee unless:

the improvements are deducted from lease payments as a form of rental payments. if the improvement serve as a substitute for rental payments, then it is fully taxable to the lessor

some guidelines for determining eligible exemptions:

the individual you are claiming must be a qualifying child or relative.they can't provide more than 50% of their own support- which means you provide over 50% of their support.the personal exemption amount is 3,900 for income.time at college counts as living at home- they need to live at home for more than 1/2 the year

what is the rule for interest expenses when computing AMT?

the interest has to be related to the primary residence- it can't be a home equity loan to buy a motor home.you add back in taxes to compute AMT.You add back in the 2% deductions to compute AMT.

under the UCC how much of a deposit can a seller keep if the buyer defaults?

the lesser of $500 or 20% of the purchase price- even if there was no liquidated damages clause

what is the dividend received deduction applied to?

the lower of the dividend amount or the overall income amount. if operations actually lost 20k but the dividend was 100k, the dividends deduction percentage would be applied to 80k

the section 179 deduction is limited to:

the net income from the business

What is the exception to the mailbox rule?

the offeror make state in the offer that acceptance must be received to be effective

things to remember about putting property into a partnership:

the partnership divides up any debt on the property. if you put in property with that you have 7k of basis in, and there is a mortgage of 3k, then you would have 7k of basis in the partnership except you have to subtract the amount of the debt the other partners will assume. if you're a 50% partner then you'd subtract 50% of the debt, so your basis goes down to 5.5k. 7k - 1.5k = 5.5k

what depreciation method do partnerships use?

the partnership elects the method to use, but it has to be an approved method by the IRS

who has the responsibility in a check fraud scheme

the party in the best position to detect the loss- so usually the company the bad checks are coming from

if a partner disposes of his share of a partnership and it had suspended loss caryyovers, how is it treated?

the passive losses are released and can offset any type of income

A group term life insurance plan is not discriminatory if

the plan benefits 70% of all employees.

what is the treatment of non business bad debt?

the principal is written off in the year the loan is deemed worthless and it is considered short-term. any foregone interest is not deductible

to be an enforceable written contract, what must the contract include?

the quantity of goods

which needs additional consideration to be binding: a material modification involving the sale of real estate or the material modification of a sale of goods under the UCC?

the real estate. UCC transactions can be modified without new consideration

Sales contracts over $500 are governed by:

the statute of frauds. The UCC governs even the sale of a gumball

in an involuntary conversion, how do you calculate any gain or loss?

the taxable gain is the LOWER of the actual gain on the transaction or the money left over after the replacement

in a tenancy in common, the owners have the right to pass their interests to ________

their heirs through the estate

As a general rule, corporations are not required to file schedules L, M-1, and M-2 if:

their total assets at the end of the taxable year and the corporation's total receipts for the tax year are less than $250,000.

if a partnership sells depreciable property at a gain in excess of the depreciation allowed on the property:

then it will be treated partly as a 1231 gain and partly as ordinary income

what are the rules for filing a valid involuntary petition against someone?

there needs to be 12 creditors, or one creditor needs to be owed at least $14,425 of unsecured debt.

What happens to subordinate interests (such as lower priority interests and liens) if the secured party sells the collateral?

they are all wiped out

are health insurance premiums deductible?

they are an itemized sched A deduction subject to a 7.5% AGI threshold

how are dividends on a life insurance policy treated for tax purpose?

they are not taxable. They are viewed as deduction in the expense of the policy

how are benefits under cafeteria plans taxed?

they are only taxable if they are received in cash

what is required for someone to file chapter 7 bankruptcy?

they must have debts- it can be any amount, as long as it is not an "abuse of the process"

how are premiums paid on a key-person life insurance policy treated on an M-1?

they reduce book income but not taxable income

how much capital losses can someone filing "married filing separately" deduct in a given year?

this filing status can only deduct $1,500 instead of the regular 3,000

When is an agency relationship required to be in writing?

to buy or sell land or last more than one year

mistake by one party in a contract; generally not a defense unless the other party knew or should have known

unilateral mistake

prepaid rental income is taxable when received unless:

unless it is potentially refundable

what are the threshholds and percentages for the dividends received deductions?

up to 20% ownership is 70%20-80% ownership is 80%above 80% is 100%

what are the rules and stipulations of an exempt offering under Reg D rule 506?

up to 35 nonaccredited investors may purchase shares. there is no limit on the placement of securities as long as the other requirements are met. the issuer needs to take reasonable steps to see that purchasers of the exempt offering are not underwriters and are buying for investment. the SEC must be notified within 15 days of the first sale of the securities.

if a taxpayer has a reasonably high income level but they get social security benefits, what percent of the SS is taxable?

usually 85%. So 85% of the SS benefits received would be included in taxable income

how is inventory treated as a charitable donation?

when it's donated, the deduction amount is the lower of cost or fair value

under UCC when does risk of loss pass to buyer?

when the goods are delivered to the carrier

if you file chapter 7, any inheritence you get within how many days of filing will go to your creditors?

within 180 days

if a debtor is adjudged to owe a creditor money and the debtor has property in the hands of a third party (i.e. money the debtor is owed from employer or money in a bank account) a ______________ may be sought

writ of garnishment

Are advertisements for rewards (i.e. the first five customers can buy this for $1) considered to be offers?

yes

Can attachment and perfection occur at the same time?

yes

Can a creditor file a financing statement before all steps of attachment are complete?

yes (cannot perfect until attach; however if file financing statement before attaching, once attached, priority of perfection goes back to the date of perfection)

Are worker's compensation insurance premiums deductible?

yes (for employer as ordinary business expense)

Can a secured party take possession of the collateral through self-help?

yes as long as not breaching the peace

for the repair of a rare book, if the first price agreed upon is 450 but additional repairs are needed and the price goes up to 600, is that binding?

yes because the repair of the book is a service, and additional repairs equate to additional consideration, the increase in price is binding

can partnerships use the cash basis of accounting?

yes, as long as none of the partners are a c corp. and there are no income limits for the cash basis

for a corp, are life insurance premiums paid for its executives as part of their taxable compensation deductible to the corp?

yes, they are fully deductible

how does the child care credit work?

you can get a credit for eligible childcare expenses of 35% of the the expenses if you have AGI of 15k or less. If your AGI is more than 15k, the credit goes down 1% of your expenses for every 2k of AGI above 15k, but it is not reduced to less than 20% of the expenses.

what is the income percentage limit of donating long term capital gain property to charity?

you can only deduct up to 30% of your AGI in long term capital gain property.

indemnity contract:

you promise to pay your friend if he has to pay someone 10k

suretyship contract:

you promise your friend's bank that you'll pay off his car loan if he doesn't pay it

if you put equipment into a corp worth 20k, you have basis of 6k, and it has a liability of 12k, how much gain do you recognize?

you recognize the amount of liability over your basis that you're relieved of. So you had a 12k liability with basis of 6k, so your gain is 6k.

how is the exclusion ratio computed on an annuity contract?

you take the basis over the amount expected to be received, and then multiply that percentage by the total of the payments received that year. so if you paid 50k for an annuity and it was estimated that you would be paid out a total of 100k, your exclusion ratio is 50%

formula for casualty losses:

you take the lesser of the decline in the property because of the casualty or the adjusted basis in the propertythen you deduct any insurance reimbursementsthen you have a disallowance of $100then you deduct 10% of your AGI for the yearThat leaves you with the casualty loss you're allowed

S corporation with C Corp E&P: Distributions 1. To extent of AAA - Tax result - Treatment 2. To extent of C Corp E&P - Tax result - Treatment 3. To extent of basis of stock - Tax result - Treatment 4. In excess of basis of stock - Tax result - Treatment

1. - Not subject to tax, reduce basis in stock - S Corporation profits 2. - Taxed as a dividend, doesn't reduce basis in stock - Old C Corp taxable dividend 3. - Not subject to tax, reduces basis in stock - Return of capital 4. - Taxed as LT capital gain - Capital gain distribution

Liquidation of an S Corp 1. Consequences to S Corp 2. Consequences to shareholders

1. FMV < Basis in assets > = Taxable gain/loss 2. Cash FMV property < Liabilities assumed > < Stock basis > = Taxable gain/loss ***Shareholder stock basis updated before used in the above equation. Include in basis the recognized gain by the S Corp

Homestead Exemption from Bankruptcy

1. "If the debtor was involved in any federal or state securities law violations or certain criminal acts within the past five years, there is no exemption regardless of the residency or domiciliary status of the debtor." 2.The debtor must have lived in or had been domiciled in the state for at least two years (730 days) prior to filing a petition for bankruptcy relief. 3. exemption=$23,675

Corporate net capital loss 1. Offset other income 2. Carryback 3. Carryforward

1. 0 2. 3 years 3. 5 years

Purchased goodwill 1. Tax rule 2. GAAP rule

1. Amortized on SL basis over 15 years 2. Not amortized, test for impairment

1934 act requires the following to register with the SEC:

1. Any company whose shares are traded on a national exchange 2. Companies that have > 10 million in assets and at least 2000 shareholders in any outstanding class 3. Companies that have > 10 million in assets and at least 500 shareholders who are not accredited, in any outstanding class 4. National stock exchanges, brokers, and dealers

Primary sources: authority for purposes of determining whether there is substantial authority for the tax treatment of an item

1. Applicable provisions of the IRC and other statutory provisions 2. Proposed, temporary, and final regulations construing such statutes 3. Revenue rulings and revenue procedures, tax treaties and regulations, and US Treasury Department and other official explanations of such treaties 4. Court Cases

How many creditors must join in filing a petition to commence an involuntary bankruptcy case 1. If the debtor has 12 or more creditors? 2. If the debtor has fewer than 12 creditors?

1. At least 3 creditors with aggregate unsecured claims of at least 15775 must join the petition 2. Any one creditor with an unsecured claim of 15775 may file the petition alone

Partnership complete withdrawal: 1. Nontaxable liquidation 2. Gain recognized 3. Recognize loss

1. Beginning capital account Percentage of income < loss > up to withdrawal = Partner's capital account Percentage of liabilities = Adjusted basis at date of withdrawal < Cash withdrawn > = Remaining basis to be allocated to assets withdrawn 2. To extent money received > partner's basis in partnership 3. Money, unrealized receivables, or inventory are the only assets received and if the basis of the assets received is less than the partner's adjusted basis in the partnership

S Corporation subject to which taxes?

1. Built in gains tax 2. LIFO Recapture tax 3. Passive investment income tax ^Only if S Corp had previously been a C corp

UCC: risk of loss: where no specific agreement 1. Noncarrier cases - Non merchant - Merchant 2. Carrier cases - Shipment contract - Destination contracts

1. Buyer will usually pick up the goods at the seller's place of business (buying goods at grocery store) - Risk of loss passes to the buyer upon the seller's tender of delivery of the goods of to the buyer - Merchant seller: Risk of loss passes only upon actual delivery to the buyer (when the buyer takes physical possession) 2. Parties contemplate a common carrier will be used to ship the goods (shipping goods by UPS) - Risk of loss passes to the buyer when the goods are delivered to the carrier - Risk of loss passes to the buyer when the goods reach the destination and seller tenders delivery

Unrealized built in gain occurs when:

1. C Corp elects S Corp status 2. FMV of assets > adjusted basis of assets on election date Net unrealized built in gain = FMV (beginning of period) < Adjusted basis at beginning of period > = Excess subject to built in gains tax

1. Personal property of a taxpayer not used in trade or business 2. Real property not used in trade or business 3. Property normally included in inventory or held for sale to customers ordinary course of business 4. Copyrights, literary, musical, artistic PURCHASED 5. Copyrights etc, held by original artist 6. Treasury stock

1. Capital asset 2. Capital asset 3. Noncapital asset 4. Capital asset 5. Noncapital asset 6. Noncapital asset

Capital gains and related parties: 1. All sales of non-depreciable property (land) between all related parties 2. Exceptions

1. Capital gains taxes imposed 2. -Husband and wife (basis merely transferred) -An individual and a 50% + controlled corporation or partnership (where gain is taxed at ordinary income)

Corporation shareholders: recognition of gain from boot Types that are taxable

1. Cash withdrawn 2. Receipt of debt securities (bonds)

Charity: cash or FMV of property Maximum allowable deduction

1. Cash: 50% AGI 2. FMV property: 30% of AGI for gifts of long term capital gain property to public charities

Transaction exemptions

1. Casual sales: not an issuer, underwriter, or dealer 2. Exchanges with existing holders and corporate reorganizations 3. Intrastate sales

Hardship provision: Homeowners: may be eligible for a partial exclusion if the sale was due to:

1. Change in place of employment 2. Health 3. Unforseen circumstances 4. When claimed within previous 2 years and fail to meet ownership and use requirements Number of months qualified for ownership/number of months must own home to qualify (24) x 250000 Single or 500000 Married

Partnership 1. Dissociation 2. Dissolution 3. Distribution of assets

1. Change in relationship of partners caused by any partner ceasing to be associated in carrying on the business 2. Dissolution: ends partnership. Partnership continues to exist after dissolution until business is wound up - Partner gives notice of withdrawal - Partners agree to dissolution - Court orders dissolution 3. Final accounting- terminates operations

Estate transfer tax Gross estate < Nondiscretionary deductions (liabilities ) > = Adjusted gross estate < Discretionary deductions > Name discretionary deductions

1. Charitable bequests, unlimited 2. Marital deduction, unlimited

Credits allowed against AMT

1. Child and dependent care credit 2. Adoption credit 3. Child tax credit 4. Contributions to retirement plans 5. Residential energy 6. Small business health care

Qualifying Child: CARES

1. Close relative: Child must be taxpayer's son, daughter, stepson, stepdaughter, brother, sister, stepbrother, stepsister, or descendant of any of these. Adopted & foster children count 2. Age limit: under 19 or 24 and under if full time college student 3. Residency and filing requirements: Same principal place of residence as taxpayer > 1/2 the year. Child can't file joint tax return for the year 4. Eliminate gross income test. Doesn't apply to qualifying child. 5. Support test changes: Did child contribute more than half of his or her support?

Duties of principal to agent

1. Compensation 2. Reimbursement/indemnification 3. Remedies of the agent

3 ways a partner can liquidate a partnership interest:

1. Complete withdrawal 2. Sale of partnership interest 3. Retirement or death

Dividends paid deduction for personal holding company income includes

1. Consent dividends for taxable year 2. Actual dividend distributions made

Duress 1. Harm threatened is physical force 2. Harm threatened is economic or social

1. Contract is void 2. Contract is voidable

S Corp: Tax-payers at risk amount increased by

1. Contribution of cash or other property to the corporation 2. Loans to the corporation 3. Allocable share of income undistributed

Worthless stock: Section 1244 (Small business stock) 1. Definition 2. Maximum ordinary loss deduction 3. Loss available to

1. Corporation's stock is sold or becomes worthless, original stockholder can be treated as having an ordinary loss (fully tax deductible) 2. MFJ: 100,000 All other filers: 50,000 3. Original owner of stock only

2 methods of depletion:

1. Cost depletion (GAAP) -Calculate unit depletion rate x units sold 2. Percentage depletion (non GAAP) -Preference for AMT -Tax only

Tax credits: general business credits: 1. formula 2. Unused credit carryover

1. Credit can't be greater than net income tax. (Regular + alternative minimum tax credit) MINUS the greater of - 25% of regular tax liability above 25000 - Tentative minimum tax for the year 2. Carried back one year and forward 20 years

PMSI: security interest with priority over all other security interests in the same collateral. Arises when:

1. Creditor sells the collateral to the debtor on credit, retaining security interest for the purchase price 2. Creditor advances funds used by the debtor to purchase the collateral

What will prevent a party from getting a discharge in bankruptcy?

1. Debtor not an individual 2. Fraudulent transfers or concealment of property 3. Unjustifiably failed to keep books and records 4. Prior discharge within 8 years 5. Commission of a bankruptcy crime

Fundamental corporate changes that require board and shareholder approval: not unanimous DAMS

1. Dissolution 2. Amendments to articles of incorporation (not bylaws) 3. Mergers, consolidations, and compulsory share exchanges 4. Sale of substantially all of the corporation's assets outside the regular course of business *Shareholders must be given notice and an opportunity to vote on the change. Approval requires a majority of the votes cast

Adjustments for AGI: Above the line deductions (Adjustments)

1. Educator expenses 2. IRA deduction 3. Student loan interest deduction 4. Tuition and fees deduction 5. Health savings account deduction 6. Moving expenses 7. Deductible part of self-employment tax 8. Self-employed health insurance deduction 9. Deduction for contributions to certain self-employed retirement plans 10. Penalty on early withdrawal of savings 11. Alimony paid 12. Attorney fees paid in certain discrimination and whistle-blower cases 13. Domestic production activities deduction

Estate transfer tax: FMV assets: gross estate

- FMV of property - Insurance proceeds - Incomplete gifts - Recovable transfers - Income in respect of a decadent

Itemized deductions not subject to overall limitation

- Medical and dental expenses - Investment interest expense - Casualty and theft losses - Gambling losses

Separately stated items: S corp items flow through to shareholder in a manner similar to a partnership. On K-1 to each shareholder

- Ordinary income - Rental income/loss - Portfolio income (interest, dividends, royalties, all capital gains/losses) - TAX EXEMPT INTEREST - Percentage depletion - Foreign income tax - Section 1231 gains and losses - Charitable contributions - Expense deduction for recovery property - Unrecaptured section 1250 income - Gain (loss) from sale of collectibles

Nondeductible IRA (nondeductible contributions) 1. Contribution limitations 2. Earnings on contributions 3. Distributions taxed:

1. Lesser of 5500, individual's compensation, limit not contributed to other (regular and Roth) IRA's 2. Accumulate tax free (deferred) until withdrawn 3. Previously accumulated untaxed earnings: taxable Principal contributions (not deducted when contributed): nontaxable

Annuities 1. Live longer than actuarial payout period 2. Death before full recovery

1. Longer than ___ months, further payments fully taxable 2. Unrecovered portion of investment is a miscellaneous itemized deduction on annuitant's final income tax return not subject to 2% AGI floor

Transaction between partner and partnership: 1. Related party loss 2. Related party gain

1. Losses between controlling partner (> 50%) and partnerships not allowed. 2. Gains between controlling partner (over 50% interest). Treated as ordinary in income

S Corp: 1. Allocations to shareholders 2. Shareholders must include on personal income tax return

1. Made on a per share, per day basis 2. Distributive share of each pass through item. Shareholders taxed on these items regardless of whether or not items have been distributed (withdrawn) to them during the year

FICA 1. Employee responsibility 2. Self employed responsibility 3. Deductibility

1. Make FICA contributions 6.2% gross wages up to 127200 and Medicare contributions 1.45% of entire gross wages, All income derived from labor is taxed; unearned income isn't taxed 2. 15.3% ON NET PROFITS; NOT GROSS WAGES. Only if net profits > 400 a year 3. Deductible by employer only Employer must pay the tax and collect employee's portion of the tax

Child and dependent care credit: credit computation

1. Maximum 35%- AGI must be 15000 or less 2. Minimum 20%- AGI > 43000 600 (20% of 3000) or 1200 (20% of 6000) Under 13. Max is 3000 for 1 child, but limited to LOWEST EARNED INCOME OF EITHER SPOUSE.

LLC basic characteristics

- Owners called members: limited liability - Ability to be taxed like a partnership (profits and losses flow through LLC and treated as owners' personal profit and losses; unlike corporation: taxed at corporate level and again when distributed to shareholders_ - Formed by filing articles of ORGANIZATION with the secretary of state - ONE PERSON CAN FORM AN LLC - Profit and loss allocated according to contributions - Every single member can limited liability **ALWAYS PERSONALLY LIABLE FOR OWN TORTS

Involuntary chapter 7 and 11 1. Unsecured creditors: 2. Grounds: 3. Ineligible debtors: 4. Who must join petition?

1. May petition a debtor involuntarily into bankruptcy proceedings under chapter 7 or 11 2. Generally not paying debts when due 3. Farmers and charities 4. Only creditors who are owed individually or in the aggregate at least 15775 in unsecured, undisputed debt

Gain or Loss on Disposition: 1. Amount realized 2. < Adjusted basis of asset sold > A. Purchase B. Gift C. Inherited 3. Gain or loss

1. Money received (boot), COD (boot), FMV property - selling expenses 2. A. Cost B. Rollover cost C. Step-up FMV 3. Recognized unless HIDE IT (gain); WRaP (loss)

Net operating loss 1. Offset other income 2. Carryback 3. Carryforward

1. N/A 2. 2 years 3. 20 years

Items included in the articles of incorporation:

1. Name of the corporation 2. Names and addresses of corporation's registered agent (person on whom process may be served if the corporation is sued) 3.Names and addresses of each of the incorporators 4. Number of shares authorized to be issued

SEC: 1. Assure accuracy of information filed or evaluate the financial merits of the securities being offered 2. Purpose

1. No 2. Assures the presence of information necessary for investors to make informed decisions

Sole proprietorship 1. Limited liability 2. Entity taxation 3. Owner taxation 4. Suitable for IPO

1. No 2. None 3. On earnings 4. No

Corporate capital losses 1. Offset income? 2. Carryback 3. Carryforward

1. No 2. Three years 3. Five years

Business casualty vs. individual casualty: 2 important differences concerning business casualty

1. No 100$ reduction 2. No 10% of AGI reduction 100% deductible

SS benefits received 1. Low income: 25000 S; 32000 MFJ 2. Lower middle income 3. Middle income Over 25000 S; over 32000 MFJ 4. Upper middle income 5. Upper income; over 34000 S; 44000 MFJ

1. No SS benefits taxable 2. < 50% SS benefits taxable 3. 50% SS benefits taxable 4. Between 50-85% of SS benefits taxable 5. 85% SS benefits are taxable

Reorganizations: Shareholder is tax payer 1. Event 2. Income 3. Basis 4. Tax attributes

1. Nontaxable 2. None 3. NBV 4. No change

Nonliquidating distributions 1. GR: nonliquidating distribution to a partner 2. GR: distributions of cash or property to a partner: 3. GR: basis of property received will be

1. Nontaxable to partner and partnership 2. Reduce partner's basis by cash or adjusted basis (NBV) of property distributed 3. Same as basis in hands of partnership immediately prior to the distribution

Qualified pension, profit sharing, and stock bonus plans 1. Payments made by employer 2. Benefits received

1. Nontaxable- put into trust account 2. Taxable- you withdraw/get money

Roth IRA 1. Contributions 2. Earnings 3. Distributions of principal and interest 4. Overall limit for contributions in a year

1. Not deductible when made 2. Accumulate tax-free while in Roth IRA account 3. Tax free if qualified distributions 4. Single: 5500, married: 11000

1. Farming, cash basis 2. Accrual

1. Not required to consider inventory 2. Must use inventory

Federal limitation's on state's right to impose income tax- which conditions must be present?

1. Only business activity of person within the state: solicitation of orders for sales of tangible property 2. Orders sent outside the state for acceptance or rejection 3. If orders accepted, filled by shipment or delivery from a point outside the state

Partnership 1. Limited liability 2. Entity taxation 3. Owner taxation 4. Suitable for IPO

1. Only for limited partners 2. None 3. On earnings 4. No

1. Mailbox rule makes what effective? 2. Which is effective only upon receipt?

1. Only makes acceptances effective upon dispatch 2. Revocations, rejections, and counteroffers are effective only upon receipt

Simple trust 1. What is it? 2. Required to 3. Cannot take which deduction 4. Amount of exemption in arriving at taxable income

1. Only makes distributions out of current income, can't make distributions from trust corpus (assets or principal) 2. Distribute all of its income currently 3. Can't take a deduction for a charitable contribution 4. 300 exemption

Partnership terminates when:

1. Operations cease 2. 50% or more of total partnership interest in both capital and profits is sold or exchanged within any 12 month period 3. Fewer than 2 partners (Partnership becomes sole proprietorship

Employee stock purchase plans 1. Exercise price requirement 2. Employee taxation 3. Employer taxation

1. Option exercise price may not be less than lesser of 85% of FMV of stock when granted or exercised 2. Not taxable income as compensation *Basis of stock: exercise price + amount paid for option if any *G/L on subsequent sale is capital. If holding requirements not met, gain is ordinary up to amount the stock's FMV on exercise date > option price *If option price < FMV of stock on the grant date, ordinary income recognized is LESSER of difference of FMV of stock when sold and exercise price, or difference between exercise price and FMV of stock on grant date 3. No tax deduction

General partnership: Required approval 1. Majority vote 2. Requires consent of ALL partners

1. Ordinary course of partnership's business 2. Matters outside ordinary course of business - Admitting new partners - Confessing a judgment or submitting a claim to arbitration - Making a fundamental change in partnership business (sale of partnership)

Charitable contributions limitation

1. Overall limit: 50% AGI -Cash: may be all 50% -General property: lesser of basis or FMV -Long term appreciated property: limited to lesser of: 30% AGI; remaining amount to reach 50% after cash contributions

Activities that may trigger nexus in a state in which company operates:

1. Owning or leasing tangible personal or real property 2. Sending employees into the state for training or work 3. Soliciting sales in a state 4. Providing installation, maintenance, etc. to customers within a state 5. Accepting or rejecting sales orders within the state, order accepting returns

2 types of automatic perfection

1. PMSI in consumer goods **PMSI in inventory or equipment collateral must be filed to be valid 2. Small-scale assignment of accounts

AMT Adjustments: PANIC TIME

1. Passive activity loss- timing difference 2. Accelerated depreciation (post 1986 purchase) - timing difference 3. Net operating loss individual taxpayer- timing difference 4. Installment income of a dealer- timing 5. Contracts- % completion vs completed contract- timing *6-9 included in deductions for regular tax purposes, not for AMT purpose, only increase AMTI* 6. Tax deductions 7. Interest deductions on some home equity loans 8. Miscellaneous deductions not allowed 9. Exemptions (personal) and standard deduction

Coverdell Education Savings account 1. Purpose 2. Contributions: deductible? 3. Maximum contribute per beneficiary 4. Earnings 5. Distributions of principal and interest

1. Pay for qualified expenses of designated beneficiary. Designated beneficiary: any child under age of 18 2. Contributions nondeductible 3. 2000 annually 4. Accumulate tax free 5. Tax-free to extent used for qualified education expense of designated beneficiary

Corporation paying dividend: Taxable amount 1. General rule 2. Property dividends

1. Payment of a dividend doesn't create a taxable event 2. Exception. Corporation distributes appreciated property. Corporation recognizes gain as if property has been sold FMV of property < NBV > = Corporate gain ^ Increases/creates corporate E&P

Gifts with unlimited exclusion

1. Payments made directly to an education institution on behalf of donee 2. Payments made directly to a health care provider for medical care on behalf of donee 3. Charitable gifts 4. Marital deduction

Dividends received deduction doesn't apply to

1. Personal service corporations 2. Personal holding companies 3. Personally taxed (S corporations)

Elements of a section 11 cause of action. Person wishing to sue needs to show:

1. Plaintiff acquired the stock (need not be the original purchaser) 2. Plaintiff suffered a loss (suffered damages) 3. Registration statement contained a material misrepresentation or material omission of fact. **NEED NOT PROVE INTENT TO DECEIVE, NEGLIGENCE, OR RELIANCE ON PART OF DEFENDENT *DAMAGES ARE THE ONLY REMEDY- RECISSION IS NOT AVAILABLE Anyone who signs a registration statement may be liable.

1. Torts committed by agent: who is liable? 2. Exception?

1. Principal not liable; only agent 2. Employers. Respondeat superior: an employer can be liable for an employee's torts committed within the scope of employment

Tax preference items- increase AMT

1. Private activity bond interest income 2. Percentage depletion deduction (excess over adjusted basis of property) 3. Pre-1987 accelerated depreciation

S Corp Shareholders 1. Include in GI 2. Not taxable

1. Pro rata share of S Corp's taxable income 2. Distributions to shareholders not taxable to extent that shareholder's basis exceeds the distribution. *Note: distribution does reduce basis, though

After initial formation, partners basis: 1. Increase by 2. Decrease by

1. Pro rata share of income and increase of partnership liabilities, subsequent contributions 2. Pro rata share of losses and decrease in partnership liabilities, subsequent withdrawals

US Tax Court: Decisions

1. Regular: Involves a new or unusual point of law 2. Memorandum: Only application of existing law or an interpretation of facts

Unrelated Business Income (usually taxable) 1. Organization uses unpaid workers makes business or activity 2. Articles made by disabled persons as part of their rehabilitation

1. Related and not taxable. 2. Related and not taxable

Head of Household: Living Requirements 1. Dependent (qualifying) child 2. Qualifying relative 3. Dependent parent

1. Required to live with taxpayer 2. Required to live with taxpayer 3. Not required to live with taxpayer

How can contract be terminated?

1. Revocation by offeror 3. Rejection by offerree 3. Operation of law: death of a party

Installment Sale 1. Gross profit 2. GP % 3. Gain recognized (taxable income)

1. Sale - COGS 2. GP/Sales price 3. Cash collections (excluding interest) x Gross profit percentage

Corporation: Income before special deductions includes

1. Sales 2. Dividends received 3. Cost of sales

Asset < 1 > < 2 > < 3 >

1. Section 179; 510000 maximum. Dollar for dollar reduction over 2030000 2. Bonus depreciation 2015-2017: 50% 2018: 40% 2019: 30% 3. MACRS depreciation on what is left

Types of tax exempt organizations allowed under IRC

1. Section 501 C corporations: created by an act of congress 2. Section 501 C 2 corporations: organized for exclusive purpose to hold title to property, collect income for the property, and turning over net income to exempt organization 3. Section 509 private foundations:

Distribution of the debtor's estate: claimants paid in following order:

1. Secured claimants - Properly perfected secured creditors are paid up to the value of their collateral - They are unsecured, nonpriority creditors for any deficiency 2. Priority claimants 3. General creditors who filed their claims on time

Section 3: 1933 exempt securities from registration requirements

1. Securities issued by banks and savings and loans (CD's) 2. Securities issued by NFP 3. Government 4. Regulated common carriers (securities issued by railroads) 5. ST commercial paper with a maturity date of 9 months or less 6. Insurance policies (but stocks, bonds, and similar securities of insurance companies are not exempt) 7. Securities issued under chapter 11 8. Church plan; charitable organizations

Secured transactions and security interest 1. Attachment 2. Perfection

1. Security interest effective between parties as soon as certain steps are taken to attach the interest. Once interest attaches, if debtor defaults, creditor has some right to take collateral from debtor to satisfy the debt. Attachment doesn't provide the creditor with rights against third parties who might also have an interest in collateral 2. Perfect security interest: gain rights over third parties by filing a financing statement. If debtor defaults, creditor will have rights to the collateral against both the debtor and most third parties

Keogh (Profit Sharing) Plans 1. What is it? 2. Net earnings from self-employment after Keogh deduction and one half self-employment tax 3. Maximum annual deductible amount 4. Maximum annual addition (contribution)

1. Self-employed retirement plan 2. Business income < Business expenses > = Net business income < 1/2 self employment tax> < Keogh deduction > = Keogh net earnings 3. LESSER of 54000 or 25% net (Keogh/self employed) earnings 4. May exceed deductible amount. Limited to lesser of: 54000, 100% net earnings if compensation < 54000

1. Partners in a partnership (including members of LLC) considered: 2. Performing services for S Corp, shareholders considered:

1. Self-employed, not employees 2. Employees for tax purposes. IRS requires S Corps pay shareholders who are also employees a reasonable compensation subject to payroll taxes

Percentage depletion method 1. Calculation 2. Limitation

1. Selling price x units sold= revenue Revenue x statutory rate 2. 50% of taxable income before depletion

Increases to AAA:

1. Separately non non-separately stated income and gains (except tax-exempt income and certain life insurance proceeds)

Allocation of profits and losses 1. General partnership 2. LLP 3. Limited partnership

1. Share equally 2. Share equally 3. Share P/L in proportion to the value of the partner's contributions Losses: only general partners are personally liable for that

Courts pierce the corporate veil for 3 reasons:

1. Shareholders commingle personal funds with corporate funds 2. Corporation was inadequately or thinly capitalized at the time of formation 3. Corporation was formed to commit fraud on existing creditors

1. Derivative action 2. Direct action

1. Shareholders suing on behalf of corporation against third parties 2. Shareholders suing the entity directly

1. Hobby: determine if activity is for profit 2. Horses

1. Shows profit 3 out of 5 years 2. 2 out of 7 years

MFS: Personal exemption claimed for spouse if:

1. Spouse has no gross income 2. Not claimed as dependent of another taxpayer

Formation: 1. LLP: File 2. Limited partnership 3. LLC 4. Corporation

1. Statement of qualification with state 2. Certificate of limited partnership 3. File Articles of organization with state 4. File articles of incorporation or corporate charter with state

Priority claims

1. Support obligations to spouse and children 2. Administrative expenses: costs of preserving estate, trustee fees, filing fees, attorney and accountant fees 3. Involuntary case gap claims 4. Wage claims up to 12850- only within 180 days prior to the filing 5. Employee benefit plans up to 12850- only within 180 days prior to the filing 6. Grain farmers and fishermen up to 6325 7. Consumer deposits up to 2850 8. Tax claims 9. Personal injury claims arising from intoxicated driving

Qualifying Relative: SUPORT

1. Support test: taxpayer gave > 50% support 2. Taxable gross income is under exemption amount: 4050 3. Precludes dependent filing joint return 4. Only citizens of US or residents of US, Mexico, or Canada 5. Relative OR 6. Taxpayer lives with them for whole year if non-relative

Dividends: taxable or no? 1. Current E&P (retained earnings) (by year-end) 2. Accumulated E&P (distribution date) 3. Return of capital (no E &P) 4. Capital gain distribution (No E&P, no basis)

1. Taxable dividend 2. Taxable dividend 3. Tax free and reduces basis of common stock 4. Taxable income as a capital gain

Timing of Taxable income to a partner 1. Event: Income - Tax consequence? - Basis impact? 2. Event: Withdrawals - Tax consequence? - Basis impact?

1. Taxable; increase basis 2. Nontaxable; decrease basis

Certain debts of an individual are not discharged under chapters 7 or 11: Debts not discharged:

1. Taxes due within 3 years of filing 2. Debts incurred by fraud, embezzlement, or larceny 3. Luxury goods 4. Open ended credit to consumers 5. Debts undisclosed in the bankruptcy petition 6. Alimony, maintenance, support, and settlement from marital separation 7. Willful and malicious injury 8. Operating a vehicle while intoxicated 9. Fines and penalties 10. Educational loans 11. Denial of discharge in a prior bankruptcy

Required to make estimated quarterly tax payments if

1. Taxes owed (Liability over withholding) > 1000 or more 2. Taxpayer's withholding is less than - 90% of current year's tax - 100% last year's tax: If had AGI > 150000 (75000 MFS) in prior year, 110% of prior year's liability is used

Passive Income 1. Definition 2. Examples

1. Taxpayer didn't actively participate. Only passive losses offset passive income, net passive loss is not deductible on tax return (suspended and carried forward until passive income can offset it) 2. -Rental income and royalties -Beneficiaries of trusts and investments in Partnerships, LLC's, and S Corps

Mirror image rule 1. Common law contracts 2. UCC

1. Terms of acceptance must mirror terms of the offer or there is no contract- just a counteroffer 2. An acceptance is effective even if it states new or different terms. New or different terms are ignored unless the contract is between merchants. Between merchants, minor changes can generally be made in the acceptance.

Partnership 1. Outside basis 2.. Inside basis

1. The basis a partner has in the ownership interest in the partnership. Interest has a tax basis similar to ownership interests in other property 2. Basis that the partnership itself has in the assets it owns. Can come by contributions made by partners. GR: Basis of asset contributed by a partner carries over and be basis of asset in hands of partnership . Inside basis can also come from purchases of partnership interest with partnership funds

1. Requirements of rule 504 (Reg D): Limit 2. Requirements of 506: 3. Both 504 and 506:

1. To be exempt, issuance of securities may not exceed 5 million in a 12 month period. No limitation on the number or type of investors 2. Unlimited dollar amount. Securities can be issued to any number of accredited investors and 35 or fewer unaccredited but sophisticated investors. 3. General advertising allowed sometimes; 15 day notice required to the SEC; reoffers to the public are prohibited

Federal bankruptcy code: Elements necessary to establish a preferential payment?

1. Transfer made to or for the benefit of the creditor 2. On account of an antecedent debt of the debtor 3. Made within 90 days prior to the filing (one year if creditor is an insider) 4. Made while debtor is insolvent 5. Results in the creditor receiving more than the creditor would have received under the Bankruptcy Code

1. Which moving costs are deductible? 2. Which moving costs are not deductible?

1. Travel and lodging of taxpayer and family -Transportation expenses actual out of pocket amounts for 17 cents per mile -Tolls and parking fees if used *Transporting household goods and personal effects to the new location 2. Meals, pre-move house hunting, expense of breaking a lease, temporary living expenses

Section 1245 personal property (machinery and equipment) rules 1. Loss 2. Ordinary income 3. Section 1231 (capital) gain

1. Treat as ordinary loss (no limitation) 2. Gain to extent of accumulated depreciation 3. Gain for sale price in excess of original cost. Remaining gain treated as a section 1231 gain

Appellate courts

1. US Court of Appeals 2. Federal Court of Appeals 3. US Supreme Court

Trial courts

1. US tax court 2. US district court 3. US court of federal claims

Child tax credit

1. Under age 17 2. $1,000 per child depending on income phase-outs: $110,000 / $75,000 / $55,000 3. Non-refundable but may be refundable through Additional Child Tax Credit.

Limited Partnerships 1. General partners liability: 2. Limited partners' liability:

1. Unlimited liability for debts and obligations of partnership 2. Limited to the total amount of their investment in the partnership

Differences between merchantability and fitness:

1. Warranty of merch: only made by merchants. Warranty only that goods will be fit for ordinary purposes. 2. Warranty of fitness: Can be made by any seller, but only if the buyer is relying on the seller to pick goods suitable for a particular purpose and is a warranty that they will be fit for that purpose

Health savings accounts 1. Pretax contributions 2. Excludable withdrawals

1. Workers with high deductible health insurance: 3400; 6750 for families 2. Any amount paid or distributed out of an HSA that is used exclusively to pay qualified medical expense of any account beneficiary- not includable in GI *Not used to pay for qualified medical expenses: include and subject to 10% penalty

Limited Liability Company 1. Limited liability 2. Entity taxation 3. Owner taxation 4. Suitable for IPO

1. Yes 2. Dependent on classification: can be taxed as a corporation, a partnership, or sole proprietorship 3. Dependent upon classification 4. No

S Corp 1. Limited liability 2. Entity taxation 3. Owner taxation 4. Suitable for IPO

1. Yes 2. Not generally 3. On earnings, but not generally on distributions 4. No

Ordinary and necessary transportation expenses include:

1. getting from one workplace to another when you are traveling within the area of your tax home. 2. going to a business meeting at another location other than your workplace. 3. getting from home to a temporary workplace. 4. visiting clients.

the taxable amount of a state tax refund is the *lesser* of:

1. refund received ($1,500), or 2. benefit received by using itemized instead of standard deduction on previous return ($200 = $6,400 - $6,200)

In a type B reorganization, as defined by the Internal Revenue Code:

1. the stock of the target corporation is acquired solely for the voting stock of either the acquiring corporation or its parent. 2. the acquiring corporation must have control of the target corporation immediately after the acquisition.

what is the difference in 1231, 1245, and 1250 property?

1245 and 1250 are BOTH still 1231 property, but they are each different.1245 property is depreciable personalty such as equipment in a sole proprietorship. The amount of depreciation taken on 1245 property is recaptured when it is sold and is then ordinary income. The remaining gain is 1231 gain.1250 property is depreciable realty such as land or buildings. On these you take the straight-line depreciation once it is sold and it is subject to the 1250 25% recapture tax. If there is actual depreciation over straight-line, that is subject to 1245 rules which would be ordinary income.

accrual based corps can deduct all or part of charitable contributions paid after the year end of its tax year if the contribution is paid within the first ____ months after the end of the tax year

2 and 1/2 months.if the board approved 300k to be donated on jan 31st of the next year, the corp can deduct the maximum amount this year.

The Securities Exchange Act of 1934 is applicable to any firm who the following two traits apply:

1.shares are listed on a national securities exchange 2. any firm with at least 500 shareholders and gross assets of at least $10 million.

Corporations making contributions to recognized charitable organizations: maximum deduction

10% of their taxable income

if an individual transfers equipment into a new corporation with a basis of 100k but a FV of 150k and the transaction is tax free, what basis does the business take in the asset

100k. when you transfer property into a corp and end up with 80% or more of the stock, the tax basis is retained by both parties

How long does the debtor have to create the reorganization plan if no trustee is appointed in CH 11?

120 days

how does the 1231 lookback provision work?

1231 gains are applied to past 1231 losses and treated as ordinary income. whent the past losses are absorbed the balance is a 1231 gain.if you have 100k of 1231 gain this year, but 80k 1231 loss last year, then 80k this year is ordinary income and 20k is a 1231 gain

How does a noncorporate shareholder treat the gain on the redemption of stock that qualifies as a partial liquidation of the distributing corporation?

A partial liquidation of a corporation is not equivalent to a dividend and therefore is treated as an exchange of stock for cash. The exchange should be considered a capital gain and should be treated as a capital gain for tax purposes.

Parol Evidence Rule: Prohibits

A party in a lawsuit involving a fully integrated written contract from introducing evidence at trial of -Oral or written statements made prior to written contract or oral statements made contemporaneously with the written contract -Seek to vary (contradict) terms of the contract

Miscellaneous itemized deductions

2% AGI test 1. Unreimbursed employee business expenses 2. Job related education expenses: must help maintain or improve skills for job 3. Uniforms 4. Business gifts: max of 25/person 5. Employment agency fees (job hunting expenses)- new job in same profession 6. Rental expenses for safe deposit boxes used to store investments, investment advice and newsletters 7. Subscriptions to professional journals 8. Tax preparation fee 9. Debit card convenience fees incurred to pay income tax 10. Activities not engaged in for profit (hobby)

Incentive stock option 1. Employee taxation *Basis of stock 2. Employer taxation

2. Not taxable income as compensation when granted or exercised. Capital G/L when sold *Basis: Exercise price + amount paid for option if any 2. No tax deduction for ISO

How long does a PMSI in equipment have to perfect their interest to have priority over other conflicting security interests in the same collateral?

20 days ( if perfected within 20 days of taking possession of collateral, priority goes back to the date gained possession)

Lifetime Learning Credit:

20% of the first 10,000 spent on qualifying eduction. Max credit=2000 *per family* (1) does not vary with the number of students in the household, (2) is available for an unlimited number of years, (3) applies to undergraduate, graduate, and professional degree expenses, (4) applies to any course at an eligible institution that helps individuals acquire or improve their job skills, and (5) does not require half-time enrollment for one semester. (Thus, CPE credit courses and professional seminars provided by eligible educational institutions may qualify for the credit.) For taxable years beginning in 2016, a taxpayer's modified adjusted gross income in excess of $55,000 ($111,000 for a joint return) is used to determine the reduction in the amount of the Lifetime Learning Credit otherwise allowable.

what is the total allowable medical expenses amount?

20k. only medical expenses in excess of 10% of AGI are allowable as a deduction

A registration statement with the SEC becomes effective on the __________ day after its filing unless otherwise stopped

20th

Termination of an S corp will occur if the S corp has passive investment income exceeding __________% of gross receipts for the past _________years and if during those years the S corp had

25%; 3 consecutive; accumulated earnings and profits attributable to prior C corp status

Eligible educators (kindergarten through 12th grade) deduct up to

250 qualified expenses paid. Two spouse eligible educators filing jointly: 500

what is the statute of limitations on the IRS assessing additional taxes?

3 years unless the amount of income omitted is more than 25% of the income reported on the taxpayer's income tax return

what is the gross income test amount for someone to be claimed as a dependent by someone else?

3,900

what is the medicare surtax obligation and how is it calculated?

3.8% of the lesser of net investment income or the excess of AGI over the AGI threshold which is total AGI minus 250k

Net investment income tax

3.8% to net investment income of individuals Income > 250000 MFJ 200000 S or HOH

Form 10-Q must be filed within _______ days of the fiscal year end for large corporations?

40

Normally the half-year convention applies to depreciate personal property placed in service. However, if more than ________% of the depreciable personal property is acquired in the last quarter of the year, the mid-quarter convention is used for all personal property acquired that year.

40%

The maximum generation skipping tax rate is:

40%

Form 10-Q must be filed within _______ days of the fiscal year end for small corporations?

45

How long does a debtor have to file a list of creditors owed, schedule of assets and liabilities, statement of affairs?

45 days

What are the other reports required to be filed with the SEC besides the 10-K, 10-Q, and 8-K under the Securities Act of 1934? (5% TIP)

5% or more owners, tender offers, insiders, proxies

Failure to file penalty

5% per month of tax due

Credit for elderly/disabled

5000 Single person, widow, widower, MFJ and one spouse is a qualified individual 7500 MFJ and both qualified individuals 3750 qualified individual MFS ^ Reduced by any SS payments received by the taxpayer ^Reduced by 1/2 AGI that exceeds: -Single: 7500 -MFJ: 10000 MFS: 5000 Balance x 15%= credit

Section 1244 Loss Limit

50000/year ordinary loss

Maximum 1244 loss that can be deducted by any single taxpayer in one year

50000: ordinary loss

what are the reg D offerings?

504- 1 million505- 5 million506- private placement, unlimitedthe SEC must be notified within 15 days of the first sale

Contribution to an IRA are limited in 2016 to:

5500 per year per individual (+1000 for individuals 50+)

how are corporate startup costs treated for tax purposes?

5k of organizational costs can be deducted in the first year- anything over 50k reduces the 5k that may be deducted in the first year though. Legal fees are allowed, but commissions paid to an underwriter are not. stock issue costs are not deductible either.

Form 10-K must be filed within _______ days of the fiscal year end for large corporations?

60

Adjusted current earnings (ACE) adjustment=

75% of difference between ACE and AMTI before this adjustment and the alternative tax NOL deduction

How many years must pass before a debtor is eligible for another discharge of debt?

8

Mike Smith received $10,000 (consisting of $6,000 principal and $4,000 interest) when he redeemed a Series EE savings bond in Year 6. The bond was issued in his name in Year 1 and the proceeds were used to pay for Mike's 21-year-old daughter's college tuition. Mike had not elected to report the yearly increases in the value of the bond. Mike must include what amount in gross income for Year 6 as a result of the bond redemption?

A cash-basis taxpayer, unless he elects otherwise, is required to report the total increment in value of noninterest-bearing U.S. savings bonds issued at a discount (i.e., Series E and EE) at the time the bonds are surrendered. Thus, the increment in value from the date of purchase to the date of surrender at or before maturity is to be reported as income when the bond is surrendered. As a result, when Mike redeems the Series EE bond, $4,000 in interest is taxable. The exclusion for U.S. Savings Bond Income Used for Higher Education (IRC Section 135) does not apply for this question because the bonds must be qualified U.S. Savings Bonds issued after 1989 to an individual who has reached age 24 before the date of issuance.

Which of the following acts will always result in the total release of a compensated surety? A. The creditor changes the manner of the principal debtor's payment. Incorrect B. The creditor extends the principal debtor's time to pay. C. The principal debtor's obligation is partially released. D. The principal debtor's performance is tendered.

A compensated surety is always totally released if the debtor tenders performance. The other actions indicated in the problem would not totally release the surety in all instances. A changing of the manner of payment or an extension of the debtor's time to pay would operate as a release only if these changes in the contract were deemed to be material. A partial release of the debtor will release the surety only to the extent of the amount of the debt released.

MACRS 10 year class

Boats and water transportation equipment

The general rule is that the following entities must use the accrual method of accounting:

C corporations Partnerships that have a C corporation partner Tax shelters (defined in IRC Section 461(i)(3)) There are some exceptions. The following taxpayers are not required to use the accrual method: Farming businesses Qualified personal service corporations Partnerships and corporations with gross receipts of not more than $5 million

Only three entities are permitted to freely select a fiscal year:

C corporations, estates, and tax-exempt entities. Trusts, partnerships, S corporations, and personal service corporations generally must conform their tax years to the tax years of their owners or a calendar year, unless the entity can establish a business purpose for having a different tax year. This means trusts must use a calendar year. Partnerships must use the same tax year as the partners. S corporations must use a calendar year. Generally, personal service corporations also use a calendar year.

Which of the following is not required by the Comprehensive Environmental Response, Compensation, and Liability Act (CERCLA), also known as Superfund? A. The creation of a fund to clean up hazardous waste sites B. The development of standards for management of hazardous waste by those who transport them C. The identification of sites in the United States where hazardous waste has been disposed Incorrect D. The ranking of hazardous waste sites based upon the severity of risk

CERCLA provides for the creation of a fund to clean up hazardous waste sites and is administered by the EPA. The EPA is also required to identify U.S. sites where hazardous waste has been disposed of, stored, abandoned, or spilled and to rank the sites based on severity of risk. The Resource Conservation and Recovery Act (RCRA) empowers the EPA to develop standards for management of hazardous waste by those who either generate or transport them.

Implied warranty of title

Can't be disclaimed by a general disclaimer "as is" "with all faults." Can be disclaimed only specifically "I do not warrant title" or by circumstance: judicial sale

Executive compensation

Can't deduct compensation expenses > 1000000

List the types of capital gains.

Capital gains include: Long-term gains Short-term gains Unrecaptured Section 1250 gain Collectibles and small business stock

Partner's loss in excess of basis

Carryforward indefinitely and remain suspended until basis is restablished

In a tax free liquidation of 80% or more owned subsidiary, parent corporation basis:

Carryover basis in the assets it receives from the subsidiary

At risk amount:

Cash contributions + recourse debt

Installment sales taxed when

Cash is received Revenue reported when cash payments received, even if accrual basis used for tax return

Identify the nonrefundable tax credits.

Child and dependent care credit Elderly and permanently disabled credit Education credits American Opportunity credit Lifetime Learning credit Adoption credit Retirement Savings Contribution credit Foreign Tax credit General Business credit

Identify the refundable tax credits.

Child tax credit Earned income credit Withholding tax credit Excess Social Security paid Long-term unused minimum tax credit American Opportunity credit Making work pay credit Adoption credit

Kiddie tax: net unearned income

Child's total unearned income - child's allowable standard deduction 1050 (or investment expense if greater) - Additional 1050 (taxed at child's rate) 1050+1050= 2100- income in excess of this taxed at parent's rate, but still reported on child's tax return

Consideration or subject matter of a contract is illegal:

Contract is void

Property of the bankruptcy estate

Debtor's estate: income generated from the estate, property the debtor receives from divorce, inheritance, or insurance within 180 days after filing the petition

Net capital loss deduction and loss carryover rules

Deduct a maximum of 3000- net long term or short term loss. Joint return treated as one person. MFS: loss deduction limited to 1/2 (1500) Carryforward unlimited time until exhausted. Maintains character as long or short term

Tax: Business organization and start up costs

Deduct first 5000 and remainder is amortized over 180 months

Explain the adjustment for Educator Expenses.

Deduct up to $250 of qualified expenses If both spouses are educators - $500 An eligible educator is a kindergarten through grade 12 teacher, instructor, counselor, principal, or aide in a school for at least 900 hours per school year.

Corporation: organizational expenditures and start up costs

Deduct up to 5000. Each 5000 amount is reduced by amount > 50000. Any excess amortized over 180 months GAAP= expense

Allowable start-up costs: partnerships

Deduct up to 5000. Each 5000 amount reduced by the amount > 50000 Includes: Training costs, advertising costs, testing costs incurred prior to opening the business

Points related to acquisition indebtedness

Deductible immediately

Bad debt expense

Deductible only for accrual basis taxpayers

Casualty and theft losses of nonbusiness property

Deductible to extent exceeds 100: 10% of AGI

Like kind exchange: Basis of property received when boot is received

FMV of like kind property received - Deferred gain + deferred loss Realized gain= proceeds - basis *Recognized only to extent boot received. Realized gain - boot= deferred gain

What is the criteria for filing: Married, filing separately?

File a separate return if only one spouse has income. In a separate property state - a husband and wife must report their own income, exemptions, credits, and deductions on their own tax returns. In a community property state - the income, deductions, credits are split 50/50.

What are the five methods of perfecting security interest?

Filing, Taking possession of collateral, Control, Automatic Perfection, Temporary Perfection

FUTA participation

All employers who have quarterly payrolls of at least 1500 or who employ at least one person for 20 weeks in a year Self employed don't participate

All of the following statements are correct about listed property except:

All of the following statements are correct about listed property except: A. it must be used in business more than 50% in order to be a Section 179 deduction. Incorrect B. straight-line depreciation must be used when the business use does not qualify the use of MACRS. C. it must be used in business at least 50% or more in order to claim MACRS. D. listed property includes passenger automobiles. IRS Publication 946 discusses the rules pertaining to the business-use requirement of listed property. Listed property includes cars and other property used for transportation, property used for entertainment, and certain computers. If the property is not used predominantly (more than 50%) for qualified business use, the taxpayer cannot claim the Section 179 deduction or a special depreciation allowance. In addition, the taxpayer must figure any depreciation deduction under the modified accelerated cost recovery system (MACRS) using the straight-line method over the ADS recovery period.

Exempt organizations: annual return requirement

Annual information return (990) stating gross income, receipts, contributions, disbursements, etc is required Exceptions: Religious or internally supported organizations and certain organizations who have < 5000 gross receipts are exempt from filing a form 990. Organizations that normally have less than 50000 in gross receipts file a postcard return only

Assuming that all other requirements are met, a corporation may elect to be treated as an S corporation under the Internal Revenue Code if it has:

Assuming that all other requirements are met, a corporation may elect to be treated as an S corporation under the Internal Revenue Code if it has 100 or fewer shareholders. This question tests your understanding of IRC Subchapter S requirements: 100 or fewer shareholders One class of stock Unanimous consent Shareholders must be individuals and not other forms of business.

What income and expenses are reported on a Schedule C?

Gross Income: revenue in a trade or business Cash = Amount received Property = Fair market value Cancellation of Debt Expenses: Cost of goods Salaries and commissions paid to others State and local taxes paid Office expenses Actual automobile expenses 50% of Business meals and entertainment Depreciation of business assets Interest expense on business loans (must be incurred and paid) Employee benefits Legal and professional services Bad debts written off by the direct write-off method

A partner must include on his personal income tax return

His distributive share of each pass through item

Partnership

Flow through entity reports income on 1065, K-1 Schedule E

S Corporation

Flow through entity reports income on 1120S; K-1 Not a taxpaying entity

What is the tax treatment for start-up expense incurred in May 2016?

For start-up expenses incurred after August 16, 2011, taxpayers may deduct up to $5,000 in the taxable year in which the business begins. The $5,000 amount is reduced by the amount by which the cumulative cost of start-up expenditures exceeds $50,000. Any remaining start-up expenditures not deducted are amortized over a 15-year period (180 months).

Tax preparer: endorsing and cashing client's refund checks

Forbidden (510)

Preparing returns that understate tax liability

Forbidden. But preparer has no affirmative duty to check the veracity of the facts presented by the client, with a possible exception for facts that appear implausible

Credit allowed against tentative minimum tax

Foreign tax credit

What credits reduce AMT?

Foreign tax credit Adoption credit Child tax credit Contributions to retirement plans credit Earned income credit

Details of partnership's business income and expenses reported only on

Form 1065

Domestic Production Activities: For AGI adjustment

Form 8903. Architects, engineering, construction, film production, sale rental or lease of company's manufactured equipment Deduction: 9% of qualified production activities income or taxpayer's taxable income without considering the deduction (whichever is less)

Penalty for understatement due to unreasonable position

Greater of 1000 or 50% of income preparer received for tax return preparation services

When a transfer of property is in part a sale and a gift, basis to the purchaser is:

Greater of: - Amount paid for the property - Seller's adjusted basis the time of the transfer + Amount of basis increase allowed for gift tax paid

What are the educational deductions FOR AGI?

Student loan interest: Limited to $2,500 A dependent may not claim the deduction Taxpayer must be legally obligated to pay the loan for incurred qualified educational expenses Tuition and Fees: Maximum deduction is $4,000 Expenses are for qualified higher and work-related education Cannot claim if expenses were applied to the Hope Credit and/or the Lifetime Learning Credit

Corporate AMT: Exemption amount

Subtracted from AMTI. 40000 - 25% of AMTI > 150000 AMTI in excess of 310000 is completely eliminated

Dependent for medical and dental expense purposes

Support over 50%: Yes Under $ taxable income: No Precludes joint return: No Only citizens: Yes Relative or: Yes Taxpayer lives with: Yes **Doesn't consider dependent's gross income or joint return requirement

AMT purpose

TP who take a large number of tax preference deductions pay a minimum amount of tax on their income

179 deduction: Limited to

Taxable income if it's less than the amount of the equipment purchased

Subpart F income

Taxable income includable by a US taxpayer from a controlled foreign corporation. Income that has NO ECONOMIC CONNECTION TO THE COUNTRY OF ORIGIN (FOREIGN COMPANY)

What types of taxes are deductible? Which are not?

Taxes are deductible in the year paid. Real estate taxes (not special assessments) Income taxes Personal property taxes Sales tax (income or general) Taxes that are not deductible: Federal taxes Inheritance taxes for states Business taxes

List the criteria used to determine if Social Security Benefits are taxable.

Taxpayers are classified into five categories depending on the level of provisional income, which is AGI plus tax-exempt interest plus 50% of Social Security benefits. Low Income = No SS benefits are taxable. Lower Middle Income = Less than 50% of SS benefits are taxable Middle Income = 50% of SS benefits are taxable Upper Middle Income = Between 50% and 85% of SS benefits are taxable Upper Income = 85% of SS benefits are taxable

Explain what the tax treatment is for Exchange of Like-Kind Business/Investments Assets.

Non-recognition treatment is accorded to a "like kind" exchange of property used in a trade or business or held as an investment, EXCEPT Inventory, stock, services, partnership interests, and property in different countries. Recognize gain to the extent of the lower of the realized gain or the boot received. In a like-kind exchange, the basis of property received retains the basis of property given up. Basis of property given up + Any boot paid - Any boot received (at FMV) + Any gain recognized Basis of property received

The penalty for underpayment of estimated taxes is imposed for making inadequate tax payments during the year. There are several exceptions to the penalty:

If the amount of unpaid tax is $1,000 or less If there was no tax liability on the prior-year tax return and the return was for a full year If at least 90% of the current-year tax is paid If at least 100% of the tax liability on the prior-year tax return is paid (if AGI is over $150,000—110% of the prior-year tax) Waiver for various circumstances such as retirement or disability

Explain the exception(s) to Gifted Property basis rule.

If the fair market value at the date of gift is lower than the rollover cost basis from the donor, the basis for the recipient depends upon the future selling price of the asset. Sell Higher Use "donor's basis to determine gain Sell Between No gain or loss Sell Lower Use "lower FMV at date of gift for loss

If you get stock as a GIFT, what is your basis?

If the owner's basis is less than FMV on the date of the gift, then your basis is the same as who gave it to you.But, if the FMV is less than the owner's basis at the date of gift, then you have a separate gain basis and a loss basis. Your loss basis is the lower amount, and your gain basis is the higher amount.

Bell Co. owned 20 engines, which it deposited in a public warehouse on May 5, receiving a negotiable warehouse receipt in its name. Bell sold the engines to Spark Corp. On which of the following dates did the risk of loss transfer from Bell to Spark? A. June 11 - Spark signed a contract to buy the engines from Bell for $19,000. Delivery was to be at the warehouse. B. June 12 - Spark paid for the engines. C. June 13 - Bell negotiated the warehouse receipt to Spark. Incorrect D. June 14 - Spark received delivery of the engines at the warehouse.

If there is a warehouse receipt, the risk of loss passes when the buyer receives this receipt. =c

Implied warranty of merchantability

Implied warranty that the goods are fit for ordinary purposes. No writing or oral promise required Implied when merchant or who ordinarily sells goods of the kind sold sells good Merchantability can be disclaimed by a statement that the goods are sold "as if" or "with all faults"

What expenses are not deductible on a Schedule C?

Nondeductible expenses include: Salaries paid to the sole proprietor Federal income tax Personal portion of: Automobile, travel, and vacation expenses Personal meals and entertainment expenses Interest expense State and local tax expense Health insurance of a sole proprietor Bad debt expense of a cash basis taxpayer Charitable contributions

Life insurance proceeds to beneficiary

Nontaxable fringe benefit

Annuities- exclusion amount

Investment amount/factor representing number of months the investment will be recovered x monthly amount

What are some Business Tax credits?

Investment credit Work opportunity credit Alcohol fuels credit Increased research credit Low-income housing credit Qualified childcare expenditures Welfare-to-work credit Employer provided childcare credit Small employer pension start-up costs credit Alternative motor vehicle credit Worker retention credit

If AMT depreciation is higher than ACE, difference

Is added back to get to adjusted current earnings

Gifts and inheritances

Nontaxable, but income received from property after property is in the hands of the recipient is taxable

Property settlements

Nontaxable/nondeductible

Value of services:

Not a charitable contribution deduction, but expenses incurred for things purchased to do the services are a charitable contribution deduction

Preferential payment: contemporaneous exchange for new value

Not a preference

Partnership: payments (salaries) not guaranteed

Not a tax deduction to partnership because payments are considered partner draws or partner distributions

how does the hope/american opportunity credit work?

It gives the taxpayer a credit of 100% of the first 2k and 25% of the next 2k of qualified post-secondary education expenses. So the total can be $2,500. The credit doesn't begin phasing out for married filing jointly until the income hits 160k of AGI.

Corporation: 3000 capital loss deduction

Not allowed, but can use capital losses to offset capital gains. Excess carried back 3 years and carried forward 5 years

Losses between a controlling partner (over 50%) and his controlled partnership from sale and exchange of property

Not allowed.

Breach of contract: punitive damages

Not available. Available for fraud only

Liabilities in excess of adjusted basis of total assets transferred to corporation

Not boot, but does generate gain NBV assets < Liabilities > = Excess liability= gain

Campaign contributions

Not considered a gift, nondeductible business expenses

Commuting expense from home to office

Not deductible

Loss on sale of principal residence

Not deductible

Personal/consumer interest

Not deductible

Value of services rendered to a charity

Not deductible

Interest expense used to purchase tax free bonds

Not deductible because interest earned on bonds is not taxable

Over the counter medicines

Not deductible from AGI

Child support

Not income or deductible

Employer: torts of an independent contractor

Not liable

Lifetime learning credit

Not limited to any set number of years of schooling. Not limited to college courses, but also available for graduate and professional schooling = 20% of qualified expenses up to 10000 2000 MAXIMUM***

FUTA: employee's benefits

Not limited to the contributions made on his behalf

If a couple separates and the husband gives the wife 5k before a legal separation decree is in place how is the 5k taxed?

It is not yet alimony so it is not taxable to the woman.

Difference between general partnership and LLP

LLP: Partners generally not personally liable for acts of fellow partners, employees, or agents But still personally liable for own negligence and negligence of those under direct control LLP must file with the state

when does a short-term capital gain change into a long-term capital gain?

LONGER than one year. if it is exactly one year it is still short term.

Required annual estimated tax payment for a C Corp:

Least of: 1. 100% tax liability prior year return 2. 100% of current year liability 3. 100% of estimated current year liability according to annualized income method

Dividends-received deduction:

Less than 20% 70% 20% or more, but less than 80% 80% 80% or more 100% (a) The deduction for shareholders that qualify for either the *70% or 80% deduction is further limited* to 70% or 80% of taxable income computed before the deduction for dividends received and before any deduction for net operating loss carryforwards or carrybacks. (b) This limit does not apply, however, if in deducting 70% or 80% of the dividends received, a net operating loss is either created or increased. (c) If shareholders are entitled to both 70% and 80% deductions for dividends received from different companies, the 80% dividends-received deduction is calculated first. When calculating the 70% deduction limitation, the total amount of dividends received from 20% owned companies is subtracted from taxable income. (2) The dividends-received deduction for dividends received from an affiliated company is equal to 100% of the dividends received. (a) An affiliated group exists where: i. a parent company owns at least 80% of the stock in at least one other corporation in the group or ii. at least 80% of the stock of other companies in the group is owned directly by one or more companies in the affiliated group. (The 80% stock ownership test does not apply to nonvoting stock that is limited and preferred as to dividends.) (b) Members of an affiliated group may elect to file a consolidated return if all members consent. In this situation, dividends paid among the members are eliminated from income. (3) Dividends qualify for the dividends-received deduction only if the 46-day (or 91-day) holding period is met for each dividend received. The holding period must be met within the 91-day period beginning 45 days before the ex-dividend date of the stock. If the stock is cumulative preferred stock with an arrearage of dividends, it must be held at least 91 days during the 181-day period beginning 90 days before the ex-dividend date. Days when the corporation is protected from loss on the stock by put options do not count toward the holding period requirement. The holding period requirement must be met for each dividend viewed over time.

Corporations not classified as large corporations required to pay:

Lesser of 1. 100% of tax shown on return for current year 2. 100% tax shown on return for preceding year **Can't be used if corporation owed no tax for the preceding year or the preceding tax was was less than 12 months

Corporations: foreign tax credit

Lesser of 1. Qualified foreign income taxes paid 2. Foreign tax credit limitation Pre-credit US tax X (Foreign source taxable income/income earned from both foreign and domestic sources) -->worldwide taxable income Unused tax credits: carried back one year and forward 10 years

Maximum amount that can be treated as home equity indebtedness

Lesser of: 1. 100,000 (50,000 MFS) 2. FMV of property - outstanding acquisition indebtedness

Income distribution deduction (Estate/trust)

Lesser of: 1. Actual distribution to beneficiary 2. DNI (less adjusted tax exempt interest)

Casualty loss: Loss to use in calculation for deductible casualty loss

Lesser of: 1. DECREASE in FMV 2. Adjusted basis

Partnership: subtract only

Liabilities assumed by the other partners and not the entire liability

Partner's capital account can begin with a negative balance.

Liabilities assumed by the partnership are > adjusted basis (NBV of assets contributed). Excess liability= taxable boot, and the recognized gain increase the capital account to a zero starting point.

Guarantor of collectibility

Liable only if the creditor is unable to collect from the debtor after exhausting all legal remedies, including demand, suit, judgment, and exhaustion of all supplementary proceedings

Distributable Net Income (DNI)

Limitation on amount trust or estate can deduct with respect to distributions to beneficiaries. Calculation: Estate (trust) gross income (Includes capital gains) < Estate (trust) deductions > = Adjusted total income + Adjusted tax exempt interest < Capital gains > (attributable to corpus) = DNI

a form of business in which partners are generally not liable for contracts of the partnerships or acts of the fellow partners, employees, or agents; however, like anyone else, partners are still liable for their own negligence.

Limited Liability Partnership

No license; preparer tax identification number (PTIN); have an active PTIN but no professional credentials (license)

Limited representation rights: authorized only to prepare tax returns

Related party loss

Loss recognized only to extent that sale price to the unrelated party is lower than the acquiring relative's original purchase price in the asset (FMV) Use purchase price to determine loss.

Personal (non business) bad debt

Loss treated as ST capital loss in year debt becomes worthless

What are the three things the plaintiff must show in a Section 11 defense under the Securities Act of 1933?

Loss, Acquired Stock, Material Misrepresentation (LAM)

Gifted property basis for depreciation

Lower of 1. Donor's adjusted basis at the date of gift 2. FMV at date of the gift

Basis of inherited property

Lower of 1. FMV at date of death 2. FMV at alternative lower valuation date (if elected) -6 months from date of death -Disposal date if disposed < 6 months from date of death

Earned work income credit

Lowest of: -Earned income of spouse with lesser amount -Actual child care expenditure -Maximum amount (3000 or 6000)

Section 1245

Machinery and equipment. Gains only. Personal properties used in trade or business > 12 months. Recapture all accumulated depreciation as ordinary income Lesser of gain recognized or all accumulated depreciation recaptured as ordinary income

Personal property

Machinery, Equipment, automobiles. All property not classified as real property.

Limited partnership

Made up of one or more general partners (personal liabilities for all partnership debts) and one or more limited partners (whose liability for partnership debts generally is limited to their investment) Management is responsibility of general partners Limited partner's: passive; like a stockholder

Undue influence defense

Makes contract voidable

In order for S Corp to be effective for the current taxable year, S Corp election must be made by

March 15 of THAT year. If election is made after that date, becomes effective the first day of the next taxable year (Jan 1) Election made before March 15: retroactively effective on first day of the taxable year

What are the six types of contracts that require writing by the party to be charged (MYLEGS) ?

Marriage, more than 1 Year, Land interests, Executors, Goods for more than $500, Surety

Section 509 Private Foundations: Excluded (public) organizations

Maximum (50% type) charitable deduction donees Broadly publicly supported organizations receiving > 1/3 of annual support from members of the public and less than 1/3 from investment income and UBI Supporting organizations Public safety testing organizations

Amount of IRA deduction (traditional)

Maximum deduction limited to the lesser of 5500 or individual's compensation MFJ: *Combined earnings must total at least that much 11,000 or 5500 each spouse

Anyone who may be a debtor under chapter 7

May also be a debtor under chapter 11. EXCEPT STOCKBROKER AND COMMODITY BROKER

Intangibles: tax

May be amortized using SL basis period a period of 15 years (180 months)

If proper waiver is filed, medical expenses paid for the decedent by her executor within one year of death

May be deducted on decedent's final income tax return

Individual consumer: income is too high: chapter 7 case

May be dismissed or converted to a chapter 13

Noncorporate shareholder who holds qualified small business stock for more than 5 years,

May exclude 100% of gain on sale or exchange of stock Maximum exclusion: limited to greater of 1. 10 times taxpayer's basis in the stock 2. 10 million, 5 million if MFS

Passive activity loss: Deductibility

May not be deducted against other income. Carry forward without any time limit unused passive activity losses held in suspension. -Carry forward -Offset future passive income -Deduct fully in year property sold

ISO requirement: exercise price

May not be less than FMV of stock at the date of the grant

Corporations in which an individual owns > 80% of stock of 2 or more corporations

May not file consolidated returns

Insurance companies

May not go into bankruptcy voluntarily or involuntarily under the bankruptcy code

Contributions to regular IRA's

May or may not be deductible. Adjustment is allowed for a year only if contribution is made by due date of tax return: April 15 -Earnings accumulate tax free (deferred) -Withdrawals from deductible IRA's taxable as ordinary income

List some other miscellaneous deductions FROM AGI.

Medical expenses Taxes Interest Charity Casualty Miscellaneous items

Estate transfer tax Gross estate < Nondiscretionary deductions (liabilities ) > ^ Name some

Medical expenses** Administrative expenses** Outstanding debts Claims against the estate Funeral expenses Indebtedness of property Certain taxes (taxes before death and state death taxes)

What types of medical expenses are deductible?

Medicine and drugs Doctors Medical and accident insurance Required surgery Transportation to medical facility Physically handicapped costs

Nexus

Minimum level of contact a taxpayer may have with a jurisdiction to be subject to its tax

What are the five elements of MAIDS that must all be proven by the contracting party in a fraud defense?

Misrepresentation of fact, Actual reliance, Intent to induce reliance, Damages, Scienter

Chapter 9

Municipal debt adjustment; family farmer can't seek relief under this chapter

In order for an S Corp election to be valid,

Must be agreed upon in writing, by ALL shareholders

Refinancing points

Must be amortized over period of the loan

Inventory: unsalable or unusable goods

Must be valued at expected selling price within 30 days minus costs to dispose of them

501 C- 3 Corporations- how do they lose tax exempt status?

Must meet these requirements: Penalty is the loss of tax exempt status: 1. No part of the net earnings may inure to the benefit of any private shareholder or individual 2. No substantial part of the activities may be nonexempt activities (carrying on propaganda or otherwise attempting to influence legislation) 3. May not directly participate or intervene in any political campaign

are S corps eligible to be in an affiliated group?

NO

LLC: separate legal entity from owners. LLC members:

Not personally liable for obligations of the business. All members have limited liability, different from limited partnership: at least one general partner is personally liable for all partnership debts -No restrictions on type and number of shareholders they may have

Dividend income that taxpayer treats as investment income for itemized deduction purposes

Not qualified dividend- not available for 15% tax rate

Insurance premiums paid by employer

Not qualifying medical expenses deduction from AGI

In laws and step relationships

Not related parties

Niece and nephew

Not related party

Antecedent debt of less than 600 paid within the preference period by an individual whose debts are primarily consumer debts

Not set aside as a preferential transfer

Payments to fully secured creditors within the preference period (90 days- 12 months for insiders)

Not set aside as a preferential transfer

Transfers between husband and wife (interspousal transfers)

Not subject to taxation for gift tax or income tax purposes. Unlimited marital deduction

Exception to the mailbox rule

Offeror may opt out. Offeror states in the offer that acceptances must be received to be effective. Acceptances must be received before the offer terminates

MACRS 7 year property

Office furniture and fixtures, equipment, property with no ADR midpoint not classified elsewhere, railroad track

Formal examination

Office or field audit 1. Office: IRS revenue agent does in IRS office or correspondence 2. Field: Performed at either taxpayer's home or at place of business of taxpayer's representative

If someone is going to transfer a check (order paper) to you, what two things do you need to make it valid?

Possession- meaning you need to actually have the check- and the person transferring it to you needs to have signed it.

Explain how Corporate net capital gains and losses are treated for tax purposes.

Net capital gains: Netted capital gains of a corporation are added to ordinary income and taxed at the regular tax rate. Net capital losses: Corporations may not deduct any capital loss from ordinary income. Net capital losses are carried back three years and forward five years as a short-term capital loss.

Kiddie tax- applies to?

Net unearned income of a dependent child under 18 is taxed at the parent's higher rate.

2 years after death of spouse, surviving spouse

No longer qualify. Can use head of household.

Salaries to sole proprietor

Non deductible

Exceptions to early withdrawal penalty tax (still subject to ordinary income tax on withdrawal)

Premature distribution was used to pay: 1. First time home buyer: 10,000 maximum exclusion 2. Medical insurance -Unemployed with 12 consecutive weeks of unemployment compensation -Self-employed 3. Medical expenses in excess 10% AGI 4. Permanent or indefinite disability, but not temporary disability 5. Education: College, books, fees, etc. 6. Death

Employer paid accident, medical, and health insurance

Premiums excludable when employer paid insurance premiums. Amounts paid to employee includable unless -Medical expenses reimbursed -Compensation for permanent loss or use or member or function of body

Are life insurance premiums paid by an employer taxable?

Premiums on the first $50,000 (face amount) of group-term life insurance are not included in income. Premiums paid for coverage above $50,000 should be included in gross income. The interest portion of deferred payment arrangements are income. Accelerated death benefits for the terminally ill are not income (if expected to die within 24months and if used to pay for long-term care.)

291 depreciation recapture

Primarily to corporations. Total amount of taxable recapture on real property as ordinary income= 20% of lesser of 1. Recognized gain 2. Accumulated SL depreciation Any gain in excess of that amount of ordinary income- allowed capital gains treatment under 1231

Apparent authority: secret limiting instructions

Principal who issues secret instructions to agent will limit agent's actual authority but not the agent's apparent authority (secret instructions unknown to the third party)

What are the Preference items of the Alternative Minimum Tax calculation?

Private activity bond interest income Percentage depletion the excess over adjusted basis of property Pre-1987 accelerated depreciation

Calculation of apportionment factor

Property & rent expense located within the state/total property **USE AVERAGE PROPERTY + Payroll paid to employees within the state/total payroll + Sales from sources within the state/total sales = /3 (Sum divided by 3)

MACRS: Personal property (machinery and equipment) 2. 3, 5, 7, 10 year MACRS property placed in service after January 1, 1987 3. 20 year property 4. Half year convention

Property other than real property (machine and equipment) 2. Computed using 200% double declining balance method 3. Computed using 150% double declining balance method 4. Personal property, property placed in service or disposed of during a taxable year placed in service or disposed of at midpoint of year: Makes expense lower in first year **USE TABLE TO IDENTIFY FACTORS.

UCC: warranty of title

Provides that the seller deliver goods free from any lien which the buyer lacked knowledge when the contract was made

Wash sale: basis of repurchased security

Purchase price of new security + disallowed loss on the sale Alternatively, basis of the old security - proceeds from the sale + purchase price of the new security **Gains are taxable

Deductible Medical Expenses Formula

Qualified Medical Expenses <Insurance Reimbursement> = Qualified Medical Expense "Paid" <10% AGI> = Deductible Medical Expenses

What is the formula used to determine deductible medical expenses?

Qualified Medical Expenses (Insurance Reimbursement) _______________________ Qualified Medical Expense "Paid" (7 ½ % of AGI) ___________________________ Deductible Medical Expenses

how are qualified dividends taxed?

Qualified dividends are those dividends collected from a U.S. domestic corporation or a qualified foreign corporation. To encourage investments in these companies, the dividends are taxed at the same reduced rate that applies to long-term capital gains.

American Opportunity Credit

Qualified tuition, fees, and course materials paid for student's first 4 years of college education Maximum credit: 2500 -100% of first 2000 in qualified expenses - 25% next 2000 in expenses during year 40% is refundable. So that means up to 1000 (2500 maximum credit x 40%) may be refunded

Which of the following retirement plan rollovers is taxable?

Rollover of a traditional IRA into a Roth IRA. Because the contribution to a traditional IRA is deductible and withdrawals from a Roth IRA are tax-free, the rollover is a taxable event

what is rule 506 of reg D of the 1933 securities act?

Rule 506 merely requires that the securities not be advertised to the general public and not be sold to more than 35 non-accredited investors. Any type of security may qualify for this exemption, including stocks and debentures. There is no limit on the dollar value of the issue, so long as the other restrictions are complied with.

Which of the following activities is regulated under the Federal Water Pollution Control Act (Clean Water Act)? Discharge of heated water by nuclear power plants Dredging of wetlands A. Both I and II B. I only Incorrect C. II only D. Neither I nor II

The Federal Clean Water Act regulates the discharge of pollutants into navigable waters, a term that is interpreted to include wetlands. Thus, the discharge of heated water by nuclear power plants as well as the dredging of wetlands would come within the jurisdiction of the Act.

Temporary perfection

Security interest in proceeds from original collateral is continuously perfected for 20 days from the debtor's receipt of the proceeds

Creditor files financing statement before steps of attachment complete

Security interest not perfected until it attaches to the collateral. But priority will date back to the date of filing

If an S corporation wants to be a c-corp and had annual sales higher than $10 million, the C corporation must use the_____________ basis of accounting.

accrual ($5 million or less would be needed for cash basis).

What does a plaintiff need to prove Section 11, civil liability, under the Securities Act of 1933?

acquired the stock and suffered a loss because of a material misstatement in the registration statement ( dont need to prove fault of scienter)

how is the basis in a jointly-owned property calculated?

The original basis is divided by the two. If you have 350k in basis, each has 175k in basis. At the death of one person, that person's half is given to the survivor at FMV. So if the FMV of the property was 400k, then the survivor gets basis of 200k plus their original basis of 175k for a total basis of 375k in a property with a fmv of 400k

The standard deduction for a trust or an estate in the fiduciary income tax return is

There is no standard deduction for a trust or an estate (IRC Section 63(c)(6)(D)). Instead, the personal exemption for an *estate* is $600, for a *simple trust* is $300, and for a *complex trust* is $100,

if cash is received in a like-kind exchange what is the tax consequence?

a gain is recognized on whatever is the lower of the cash received or the gain on the exchange. If there was 5k received and the gain was only 3k, then a 3k gain is recognized. if the gain was 5k and cash received was 2k, then a gain of 2k is recognized

A court of appeals hears appeals from

US District Courts and US Tax Court

Under the Resource Conservation and Recovery Act (RCRA), the EPA is empowered to:

Under RCRA, the EPA is empowered to identify and list hazardous wastes. Ordering a responsible party to clean up a hazardous waste site, identifying U.S. sites where hazardous waste has been spilled, and creating a fund to clean up hazardous waste sites are all components of the Comprehensive Environmental Response, Compensation, and Liability Act (CERCLA), also known as Superfund.

Integral Corp. has assets in excess of $10 million, has 650 stockholders, and has issued common and preferred stock. Integral is subject to the reporting provisions of the Securities Exchange Act of 1934. For its 20X1 fiscal year, Integral filed the following with the SEC: quarterly reports, an annual report, and a periodic report listing newly appointed officers of the corporation. Integral did not notify the SEC of stockholder "short swing" profits, did not report that a competitor made a tender offer to Integral's stockholders, and did not report changes in the price of its stock as sold on the New York Stock Exchange. Under SEC reporting requirements, which of the following was Integral required to do? Incorrect A. Report the tender offer to the SEC. B. Notify the SEC of stockholder "short swing" profits. C. File the periodic report listing newly appointed officers. D. Report the changes in the market price of its stock.

Under SEC reporting requirements, the company is required to file the periodic report listing newly appointed officers. Under the Williams Act, which amended the Securities Exchange Act of 1934, it is the company that is making the tender offer that is required to file with the SEC and not the target company. "Short swing" transactions are illegal. It is the insider who bears the liability for failure to comply with the "short swing" rules under the Securities Exchange Act of 1934. The prices of stock traded on the New York Stock Exchange are a matter of public record—you only have to pick up a copy of the Wall Street Journal or most local newspapers! There is no reporting requirement by companies to report changes in their stock prices.

Under the uniform capitalization rules, which expenses are not included in the cost of inventory?

Under UNICAP, indirect costs that are not capitalized are non-manufacturing costs such as selling, research, and product liability. Generally, manufacturing overhead costs, such as indirect material, indirect labor, purchasing costs, cost recovery, rent, insurance, and utilities, as well as depletion, quality control, and storage are capitalized.

Under the liquidation provisions of Chapter 7 of the Federal Bankruptcy Code, certain property acquired by the debtor after the filing of the petition becomes part of the bankruptcy estate. An example of such property is:

Under the liquidation provisions of Chapter 7 of the Federal Bankruptcy Code, certain property acquired by the debtor after the filing of the petition becomes part of the bankruptcy estate. An example of such property is: Correct A. municipal bond interest received by the debtor within 180 days after the filing of the petition. B. alimony received by the debtor within one year after the filing of the petition. C. Social Security payments received by the debtor within 180 days after the filing of the petition. D. gifts received by the debtor within one year after the filing of the petition. the bankruptcy statute specifically excludes Social Security payments.

Smith contracted in writing to sell Peters a used personal computer for $600. The contract did not specifically address the time for payment, place of delivery, or Peters' right to inspect the computer. Which of the following statements is correct?

Unless otherwise stated in the contract, the Uniform Commercial Code (U.C.C.) permits the buyer to inspect the goods prior to payment or acceptance. ("Otherwise" might be a cash-on-delivery sale.) Also, when a contract is silent as to time of payment or time or place of delivery, the U.C.C. "fills in the gaps"—payment is construed as being by any reasonable means, such as personal check at the time and place of delivery; delivery to the buyer is to be within a reasonable time and at seller's place of business. If seller has no place of business, place of delivery is seller's home (never buyer's home unless stated expressly).

Once a dividend is declared by the board, shareholder status:

Unsecured creditor

What are the two tiers under Regulation A, Simplified Filing?

Up to $20 Million and up to $50 Million

Employer provided transit passes

Up to 255/month may be excluded

Employer-provided parking

Up to 255/month may be excluded

Qualified Tuition Programs (529 Plans)

Used for undergraduate and graduate. Earnings accumulate free of federal tax as long as money stays in the plan and is used for education. 2 Types: 1. Prepaid program (paid to the school) 2. Savings account plan Can contribute up to annual gift tax exclusion or can frontload 5 years (14000*5)=70,000 Can contribute to coverdell and QTIP in same year.

if a person is adjudicated incompetent by a court having proper jurisdictions- all future contracts entered into by this person are:

VOID

What are the three requirements for attachment of security interest (VAC)?

Value, Agreement (evidenced by either authenticated record or creditor taking possession of collateral), Collateral

Contract made by a party after adjudicated mentally incompetent

Void

Intoxiation

Voidable. Defense only if intoxication prevents the promisor from knowing nature and significance of promise and other party knew of the impairment

Wages paid for domestic services are subject to special rules for determining whether they are subject to payroll taxes. When are domestic wages subject to federal unemployment tax?

Wages paid for domestic services are subject to federal unemployment tax if they exceed $1,000 per quarter, aggregating wages paid to all employees.

Tom, a roofer, repairs a leaky roof on George's home. They had agreed that the cost of the materials and labor involved in the repair would be $200. If George fails to pay the $200 owed for the roof repair: A. Tom could create a mortgage lien on the home by filing written notice of the lien. B. Tom could create an artisan's lien on the home by filing written notice of the lien. C. Tom could create a mechanic's lien on the home by filing written notice of the lien. Incorrect D. Tom could not obtain any special type of lien against the home.

When labor, services, or materials are provided for the purpose of making improvements or repairs to real property, the creditor can place a mechanic's lien on the property if payment is not made. This is a statutory lien controlled by state law, and generally the lien holder is required to file a written notice of the lien within a specific time period. An artisan's lien is a lien arising from work done on personal property. A mortgage lien is a security interest in real property given by the owner as security for a debt owed to the creditor. No specific agreement to transfer a security interest exists between George and Tom.

Jane Pleasant had property repossessed after making an installment sale. Which of the following statements is true?

When property is repossessed after an installment sale, the taxpayer must figure the gain or loss on the repossession and the basis of the repossessed property. The kind of property repossessed determines the rules to follow for figuring these figures. IRS Publication 537 discusses the difference in the rules between repossessed personal property and repossessed real property.

What is not considered a primary authoritative source when conducting tax research?

The Internal Revenue Code, or IRC, is the most authoritative source when conducting tax research. Treasury Regulations and U.S. Tax Court cases are also primary sources. IRS publications are not considered a primary authoritative source when conducting tax research.

What is the exception to the participation in another qualified plan requirement?

The exception for taxpayers participating in another qualified plan is: An individual is not considered an active participant in an merely because the individual's spouse is a participant. Spouse's income can be used to qualify a spouse for additional contribution amount if spouse does not work.

Has all of the elements of fraud except for scienter

innocent misrepresentation

how do you deduct INVESTMENT INTEREST EXPENSE?

it is deductible to the extent of net investment income. So you take non-interest investment expenses and net them with investment income. You can then deduct investment interest expense up to the amount of net investment profit.if you have investment income of 10k and investment expenses of 8k, you can deduct up to 2k of investment interest expenses.

how is basis in a distribution of property from a partnership to an individual calculated?

it is the same as the partnership's basis in the property but it may not exceed the partner's basis in the partnership.Also, cash distributed reduces the partner's basis dollar for dollar.If you have a partner with basis of 50k and they get 20k in cash, and then some land with 40k of basis to the partnership, the partner can only take basis in the land of 30k. because of the 50k basis less the 20k received in cash, there's only 30k of basis left for that partner.

when you put property into a corp in exhange for stock, how is your basis in the stock determined?

it is your previous basis in the property plus any gain recognized on the property. if you had an asset you paid 100k for but is worth 150k when you transfer it, if the transfer qualifies as tax free you have basis of 100k in the stock. but if the transfer doesn't qualify as tax free and you have a 50k gain on the transfer, then your basis in the stock is 150k.

a bad debt can only be deducted to the extent that:

it was included in income.Bad debt expense is actually the removal of a previously recognized income that was never received

how is basis determined when a gift is given?

it's the basis of the gift in the hands of the gift-giver

what is a schedule A?

itemized deductions

In a general partnership, matters regarding ordinary course of business can be approved by a _________ vote

majority

if an agent breaches duty owed to a principal, the principal can recover _________

monetary damages

Are new partners admitted to a partnership personally liable for partnership debts that arose before they joined?

no

Are punitive damages available under the UCC?

no

Can a limited partner participate in management of the partnership?

no

Can perfection occur before attachment?

no

Does a general disclaimer, such as "goods are sold as is", disclaim warranty of title?

no

Does a partnership agreement have to be in writing?

no

Does a sole proprietorship agreement have to be in writing?

no

Does an offer have to be express in writing?

no

Does the mirror rule image apply to the UCC?

no

Is a principal generally liable for the torts of their agent?

no

Is a sole proprietorship considered a legal entity?

no

Is consideration required to form an agency relationship?

no

are estimated federal tax payments deductible on the tax return?

no

are insurance premiums on property for personal use deductible?

no

are members of an LLC free to transfer membership without the consent of others?

no

are punitive damages available for breach of contract?

no

do proposed regulations have the effect of law?

no

is a dividend received deduction allowed on a REIT investment?

no

If an employer and employee make a special agreement that the employer will not be held liable for the torts of the employee, does this protect the employer from the third party?

no (employer would still be liable but then just have to collect from employee)

Is failure of agreed-upon transportation a defense in a UCC contract?

no (must find different shipper)

Is there a 20 day grace period for PMSI in inventory?

no (to have priority it must be perfected before the debtor gains possession and notice must be given to other perfected parties in same collateral)

can a surety compel the creditor to collect from the primary debtor or go after the debtor's collateral?

no. the surety is primarily liable upon the debtor's default, and they can't compel the creditor to take either of these actions

Can the secured party retain the collateral as possession if the debtor has already paid 60% of the debt?

no; must sell (called Compulsory Disposition)

The Clean Air Act requires:

the preservation of natural visibility within major national parks and wilderness areas. Also under the Act, air quality may not decrease in those areas that currently meet national ambient air quality standards. For new sources of emissions in areas that have achieved national ambient air quality goals, the best available control technology (BACT) is required under the Act. The Act requires the lowest achievable emission rate (LAER) for new sources of emissions in nonattainment areas.

Chapter 13 bankruptcy was revised under the 2005 Bankruptcy Reform Act. Some of the more material changes and additions include

the repayment period cannot exceed five years; the time period is based on the family income as computed. the confirmation of the plan must result in a hearing no sooner than 20 days or more than 45 days after the meeting with the creditors. when considering disposable income of the debtor, the Bankruptcy Code excludes up to 15% of the debtor's gross income for charitable contributions. when considering disposable income of the debtor, reasonable costs of health insurance for the debtor and the debtor's family may be considered.

how is your taxable income and basis determined for property received as a dividend?

the taxable income is the FMV of the property received less any liabilities assumed.Your basis is always the FMV of the property.

what is the rule when a taxpayer make a charitable contribution in connection with admission to an entertainment event?

the taxpayer can only deduct the difference between the amount paid for admission and the fair value of admission. If you pay 100 but regular admission is only 50, you can deduct 50.

what are the rules on preferential transfers?

the trustee can set aside a payment to a creditor made within 90 days previous to the bankruptcy filing. A payment to an insider within a year before the bankruptcy can also be set aside as preferential. If you receive "new value" such as buying something before the bankruptcy, that won't be preferential.

which court is not of 'original jurisdiction'

the united states court of appeals

Who does the third party hold liable if the principal was undisclosed or partially disclosed?

their choice

what is the only thing that can cause a gain in a partnership distribution?

when cash is distributed that exceeds the partner's basis

Is there a trustee appointed for CH 7 Bankruptcy?

yes

are punitive damages available for fraud?

yes

do you have to file a financing statement for PMSI in equipment?

yes

do you have to file a financing statement for PMSI in inventory?

yes

is a suretyship required to be in writing?

yes

Can a partner assign their rights/transfer interest without the consent of others?

yes (assignee gets the rights, but does not become actual partner with power)

Is a seller allowed to demand cash, as opposed to check, for a UCC contract?

yes (but must give the buyer sufficient time)

how are corporate NOLs treated?

you can carry them back two years and ahead 20 years

what is the rule about deducting the either sales tax or state income taxes paid?

you can deduct the greater of sales taxes paid or state income taxes paid

if you get assigned a partnership interest, do you get to share in profits and management?

you get the share of the profits, but you don't get to participate in management unless the other partners agree

if you contribute property and services to gain an interest in a partnership, how is your basis determined?

you get your basis in the land plus basis equal to the value of the services contributed. BUT- you must recognize wage income for the services provided- that's what gives you tax basis

what is the basic hobby loss rule?

you have to make a profit in 3 out of the most recent 5 years. so 3 years of losses in a row means you're engaged in a hobby not a business.expenses related to a hobby are deductible only to the extent of revenues earned

if you contribute services to get a partnership interest, do you use the % of net assets or the % of the assets FMV of the partnership?

you use the fair market value of the partnership to determine the amount of ordinary income you recognize

how is a trade that is not like-kind treated for tax purposes?

your basis in the new asset is its fair market value, and you subtract the basis in the asset you traded and that difference is your gain

what can never be inferred?

quantity. price, subject matter, time for performance can all be inferred.

In carrier cases, where there has not been any specific agreement as to delivery or risk of loss, how will the goods normally be transferred?

by common carrier

internal rules on how the corporation is to be run

bylaws

Under the Securities Act of 1933, what kind of liability can be imposed on anyone who uses any type of fraud in connection with the issuance of a security?

criminal

voting system where each share is entitled to one vote for each director position that is being filled; the shareholders may cast the votes in any way, including selecting all for one candidate; helps minority shareholders gain representation on the board

cumulative voting

If there is a minor breach of contract under common law, what is the non-breaching party liable to?

damages only (not discharge)

In an involuntary CH 7 or Ch 11 bankruptcy, how many creditors must join together and how much must they be owed in aggregate?

debtor has fewer than 12 creditors - one creditor must be owed $16,750; more than 12 creditors - three creditors owed $16,750 in aggregate

how are a C corp's net capital losses used?

deducted from the corp's ordinary income up to 3k, then carried back 3 years and forward 5 years

even if the workload is uneven, if no agreement to the contrary exists, profits in a partnership will be:

divided equally- even if one partner does more work than the other. there needs to be an agreement if profits are different than 50/50

If a worker is paid salary, as opposed to by the job, are they likely an employee or independent contractor?

employee

what does rule 506 D allow?

that the securities are NOT advertised to the public, and that they are not sold to more than 35 non-accredited investors.There is NO LIMIT on the dollar value of the issue- and they can be either stocks OR debentures

how are charitable deductions calculated for corporations?

the amount deducted in one year is up to 10% of income before any dividends received deductions. this means that you would take income plus dividend income and take 10% of that, then subtract the DRD afterwards. any amount over that can be carried forward for 5 years

if someone is supposed to receive alimony and child support, but receives much less then they are supposed to during the year, how is it taxed?

the amount is applied to child support first. If they were supposed to get 5k during the year in child support and they received 6k total, 5k would be child support and 1k would be alimony.

If a partnership agreement is silent on the subject of how profits will be shared, they will be shared

equally among the partners. If a question poses a situation where the partners defined how losses would be shared, but they did not write down their intended distribution of profits, a formula for sharing losses will not be applied to sharing profits.

Ultramares rule:

established in a 1931 case of the same name, requires privity before an accountant is liable for negligence. Other rules, such as the Restatement rule, allow foreseeable users who rely on a negligently false statement to sue.

warranty that arises from any statement of fact or promise made by seller, any description of the goods made by the seller, or any sample or model shown by seller

express warranty

Who has priority between perfected (non PMSI) security interest and a judicial lien?

first to perfect (if security interest) or attach (if judicial lien)

once an S corps status is revoked or terminated, how long does it have to wait before making a new S election?

five years

how are gains/losses treated in related party transactions on the individual level?

gains are taxed but losses are not deductible

how are gambling gains and losses treated?

gambling winnings are included in gross income. Gambling losses DO NOT offset gambling winnings- they are deducted from AGI as a misc itemized deduction limited to the amount of gambling winnings

Any variation in a contract that changes a _____________ surety's risk (i.e. extension of time) will discharge the surety

gratuitous

surety who is not compensated for his or her promise to the creditor

gratuitous surety

uniform capilization rules amount

gross receipts of 10million or more

liable to the creditor only if the debtor does not perform his or her duty to the creditor

guarantor

when a corp sells an asset, what would create ordinary income?

if an asset was bought for 50k, had dep exp of 10k, and was sold for 60k, 10k would be ordinary income and 10k would be a 1231 gain. the dep exp is recaptured through the sale.

When does a partnership agreement have to be in writing?

if longer than 1 year

what is one reason for which the corporate veil might be pierced?

if the corp was undercapitalized when it was formed

what is the exception to the rule of stock dividends being non-taxable?

if the investor can choose between cash or a stock dividend, then it's taxable.If some owners are given common stock but some are given preferred stock, then it's taxable to all.

when would a corp want to distribute land instead of cash?

if the land is appreciated then the corp would have to pay tax on the appreciation.the amount of dividend income is the same either way to the shareholder

what is the gain and loss basis of stock that is given to you as a gift?

if the owners basis is less than FMV, then it is the owners basis. If the FMV is less than the owners basis, then you take the FMV basis for losses, and the owners basis for gains.For an inheritance the gift is valued at FMV.

when a partner sells his interest, what are the tax consequences?

if the partnership has unrealized receivables or substantially appreciated inventory, then the partner will recognize ordinary income on his share.Also, whatever cash amount the partner sells his interest for is taxable, plus a gain in the amount of liabilities that he is no longer responsible for.

what is the rule with donating 'ordinary income property' to charity?

if you've owned it for less than a year the amount you can deduct is the lower of cost or market.If you've owned it more than a year, then you can deduct the FMV

Which of the following statements is correct with respect to a limited partnership?

A. A limited partner may not be an unsecured creditor of the limited partnership. B. A general partner may not also be a limited partner at the same time. C. A general partner may be a secured creditor of the limited partnership. Incorrect D. A limited partnership can be formed with limited liability for all partners. A general partner may be a secured creditor of the limited partnership. A general partner may have a limited partnership interest while serving as the general partner. Under the Uniform Partnership Act, there must be at least one general partner in a limited partnership.

Credit for prior year minimum tax (AMT credit)

AMT paid in taxable year carried over as credit to subsequent taxable years. Only reduce regular tax, not future alternative minimum tax Carryforward is forever

A voluntary petition for bankruptcy relief may be dismissed by the court if the debtor: fails to provide necessary documents within the specified time period. was convicted of a drug trafficking offense or a violent crime and the victim files a motion to dismiss the petition. has income over the average median family income of the state where the petition is filed. fails to pay post-petition child support or alimony payments. A. I, II, and IV only B. II, III, and IV only Incorrect C. I, III, and IV only D. I, II, III, and IV

All of the items listed are correct. Under the 2005 Bankruptcy Reform Act, these items were added to eliminate perceived abuses of individuals using the bankruptcy forum to avoid debts they otherwise could or should pay. A voluntary petition for bankruptcy relief may be dismissed by the court if the debtor fails to provide necessary documents within the specified time period. This item is designed to encourage prompt and complete filings of paperwork, to speed the process, and to ensure accountability for all debts. (Bankruptcy Abuse Prevention and Consumer Protection Act of 2005, Section 316) A voluntary petition for bankruptcy relief may be dismissed by the court if the debtor was convicted of a drug trafficking offense or a violent crime and the victim files a motion to dismiss the petition. This item was included by Congress to prevent criminals from avoiding their otherwise just debts of retribution to the victim. (Bankruptcy Abuse Prevention and Consumer Protection Act of 2005, Section 102; 11 USC Section 707) A voluntary petition for bankruptcy relief may be dismissed by the court if the debtor has income over the average median family income of the state where the petition is filed. This item deals with high-income persons attempting to avoid debts that they otherwise could pay. After a computation based on certain variables, if the party has income that exceeds the state median income by $6,000, there is a presumption of abuse. The debtor can refute the presumption of abuse if they can show good cause—for example, known upcoming health costs or similar items. (Bankruptcy Abuse Prevention and Consumer Protection Act of 2005, Section 102) A voluntary petition for bankruptcy relief may be dismissed by the court if the debtor fails to pay post-petition child support or alimony payments. This item deals with so-called "deadbeat parents" who use the system to avoid child support or alimony. Notice, however, that it deals with post-petition domestic support obligations. (Bankruptcy Abuse Prevention and Consumer Protection Act of 2005, Section 215)

Cumulative preferred dividends

Although these dividends accumulate even if not declared, no dividend can be paid to common stockholders until all cumulative dividends are paid. No dividend is due until it is declared by the board of directors

Gain or loss on disposition of property

Amount realized < Adjusted basis of assets sold > = G/L realized

Define Amount Realized.

Amount realized includes: Cash received (boot); Cancellation of debt (boot); Property received at fair market value; and Services received at fair market value; Reduce the amount realized by any selling expenses

Who can file voluntary bankruptcy, but not be the subject of involuntary bankruptcy?

- Farmers - Non profit charitable organizations

Corporation liquidation: Corporation sells assets and distributes cash to shareholders 1. Corporation 2. Shareholder

1. Sale price < basis > =Taxable gain/loss 2. Proceeds < Stock basis > =Taxable gain/loss

General procedure for fundamental changes

1. Board resolution by majority 2. Corporation notifies all shareholders even if they aren't entitled to vote 3. Shareholder approval 4. Filing of articles

Consideration- there must be:

1. Something of legal value 2. Bargained for exchange

Tax exempt interest income

1. State and local government bonds/obligations 2. Bonds of a US Possession: Guam or Puerto Rico 3. Series EE Bonds 4. Interest on insurance dividends left on deposit with Veterans Administration Insurance

1. Dividend income 2. SS benefits (low income)

1. Taxable 2. Not taxable

1. To be a widow/widower, must be principal residence of dependent child for: 2. HOH: Must be principal residence of qualifying person:

1. Whole year 2. More than half of a year

How is FUTA tax computed?

6.0% of first $7,000 per year of compensation per employee

for 2019, employees were liable to make FICA contributions of ________% of their wages up to _________

6.2; $132,900

Partnership operations: Partner Basis Formula

Beginning capital account: Cash, FMV of services, NBV - Liability + % of All income: ordinary, capital, TAX FREE** - % of all losses: partnership may take a partnership loss as a tax deduction up to his/her basis - Withdrawals: (property distribution: reduce capital account by adjusted basis NBV of distributed property, Can't go past 0 in capital account =Ending capital account + % Liabilities (what you assume) (recourse and nonrecourse) = Year end basis

Cost depletion can't exceed

Beginning of the year basis For denominator: units at the beginning of the year

Which of the following would change if an asset is treated as personal property rather than as real property? Requirements for transfer Creditor's rights

Both requirements for transfer and creditor's rights would change if an asset is treated as personal property rather than as real property. Personal property includes such items as animals, merchandise, rights to stock, and rights to patents. Real property includes such items as the right to drill for oil and an easement across another person's property. A deed is required for real property to be transferred, while personal property does not require a deed. Real property is foreclosed on and personal property is repossessed. Therefore, the rules for each will be different for the creditors. Foreclosure rules are much different than rules for repossession.

Rescission

Cancel a contract and restore parties to original position

Surety doesn't have right to:

Choose which action creditor takes in the event of default. Only a guarantor of collectibility would have the right to require a creditor to first proceed against the debtor or against available collateral

Gift splitting: spouse

Creates 28000 exclusion per donee

AAA: Accumulated Adjustments Account

Cumulative amount of S corporation income or loss (separately and non separately stated items, excluding tax exempt income), since corporation most recently elected S status, less all cumulative distributions *DISTRIBUTIONS MAY NOT REDUCE AAA BELOW 0, BUT AAA MAY BE NEGATIVE FROM S CORPORATION LOSSES

Section 1231

Depreciable personal and real property used in taxpayer's trade or business and held over 12 months. Trade/business property involuntarily converted included also Allows special tax rates for long term capital gains from sales, exchanges, or involuntary conversions of certain noncapital assets

Define Alternative Minimum Tax.

Designed to ensure that taxpayers who take a large number of tax-preferenced deductions pay a minimum amount of tax on their income. It is the excess of the tentative AMT over the regular tax.

Nondeductible Medical Expenses

Elective Surgery Elective Cosmetic Operations Illegal Drugs Travel Vitamins SS paid for basic Medicare Funerals Cemetary lots Insurance against loss of earnings due to sickness or accident Life insurance Capital expenditures Health club memberships recommended by doctor Personal hygeine and other ordinary personal expenses (toothpaste, toiletries, OTC medicines, bottled water, diaper service, maternity clothes)

What types of medical expenses are not deductible?

Elective surgery Drugs that are against the law Vitamins Social security tax paid for basis Medicare Life insurance Capital expenditures Health club memberships Personal hygiene Funerals

The ____________ severely restricts collection agencies ability to call third parties, such as relatives of the debtor, to indirectly pressure the debtor; agency can contact third parties to discover debtor's whereabouts, but may not disclose that it is a collection agency or that the debtor owes a debt

Fair Debt Collection Practices Act (FDCPA)

Punitive damages

Fully taxable as ordinary income if received in business context or for loss of personal reputation Punitive damages received in personal injury case also taxable except in wrongful death cases

Unrealized built in gain tax rate

Highest corporate income tax rate: 35%

Under the Secured Transaction Article of the U.C.C. (Article 9), what would be the order of priority for the following security interests in consumer goods? I.Financing agreement filed on April 1 II. Possession of the collateral by a creditor on April 10 III. Financing agreement perfected on April 15

I, II, III The order of priority of security interests is generally determined by the date of perfection of the various interests. In this case, the filing of a financing statement on April 1 perfected security interest I on that date. Security interest II was perfected on April 10, the date that the creditor took possession of the collateral. Security interest III was perfected on April 15 (also by filing a financing agreement). Remember that perfection can be achieved in three ways: by filing, by possession, and by attachment (purchase money security interest).

Explain the tax treatment of property settlements in a divorce.

If the divorce settlement provides for a lump-sum payment or property settlement by a spouse: The spouse paying - no deduction allowed The spouse receiving - not included in income.

Warranties and tort liability

Independent. Anyone injured can be sued: privity not required

Real property

Land and building. Land and all items permanently affixed to the land (buildings, pavements, etc)

Dividends received deduction

Lesser of 1. 70 or 80 percent of dividends received Or 2. 70 or 80% of taxable income

Investment interest deduction

Limited to net taxable investment income (like gambling loss)

List the requirements for moving expenses to be deductible.

Must be work related Must be 50 miles or more (Distance must be 50 miles farther from OLD house than OLD workplace was) Must work in new location for 39 weeks during the 12 month period after the move Only direct costs allowed: Travel and lodging for the taxpayer and family Transporting household goods and personal effects to new location

is a purchase price and a description of the property required to have a valid deed?

NO- you only need a description of the property

are dividend-received deductions allowed for non-domestic companies?

NO. if you get dividends from a non domestic corp it's all taxable

If balance due after withholdings not > 1000

No penalty for underpayment of estimated taxes

What is the tax treatment of nondeductible passive activity losses?

Nondeductible passive activity losses that are unused are carried forward without any time limit. Suspended losses are used to offset passive income in future years. If still unused, suspended losses become fully tax deductible in the year the property is disposed of/sold. If the taxpayer becomes a material participant in the activity, unused passive losses can be used to offset the taxpayer's active income in the same activity.

Flexible Spending Arrangements

Plan that allows employees to receive a pre-tax reimbursement of certain incurred expenses. Pretax deposits into employee's account: part of employee's salary deposited in FSA. Deposits done by salary reduction by employer, and employee isn't taxed on that income. Employee can use funds to pay for qualified health care and or qualified dependent care costs and submits claims to plan administrator for reimbursement. Must forfeit funds not used within 2.5 months after year end

How is Purchased Property Basis calculated?

Purchased property basis = cost Generally, the basis of property is the cost of such property to the taxpayer. The basis should be adjusted for the following: Increase the basis for capital improvements Reduce the basis for accumulated depreciation

1934 act regulates

Purchases and sales of securities after initial issuance

Distribution to partner

Reduces partner's basis

Earned income credit

Refundable

Property acquired as a gift: cost basis

Retains cost basis in the hands of the donor at the time of the gift. Basis is increased by any gift tax paid attributable to the net appreciation in the value of the gift

a transaction exemption available for offerings made within one state. Issuer must do 80% of their business in the state and be a resident of the state, purchasers may not resell to nonresidents for 6 months and general solicitation is prohibited

Rule 147

Rule that allows issuance of up to $5,000,000 within a 12 month period for private offerings; no limitations on the amount of investors and no specific disclosures are required.

Rule 504

What self-employment deductions are FOR AGI and not on deducted on Schedule C?

Self-employment tax - 50% of tax (Boss' half) Self-employed health insurance - 100% Keogh (profit sharing) plans

List the requirements for the Retirement Savings Contribution Credit.

Taxpayers must be: at least 18 by the close of the tax year not a full-time student not a dependent Limited to the excess of the regular tax liability over the alternative minimum tax liability of the taxpayer, minus the taxpayer's nonrefundable personal credits. No carryover is allowed.

how are gains and losses handled on personal and investment property for individuals?

Taxpayers must report gains and losses on investment property (such as their investment stocks and bonds) but only gains on personal property are reported. For example, if a taxpayer sells personal furniture at a gain, that capital gain must be reported. However, if the same personal property is sold at a loss, no deduction is allowed.

Limitations on lifetime learning credit, Hope scholarship credit (American Opportunity Tax Credit)

The Lifetime Learning credit may not be claimed by a taxpayer for expenses of a student for any tax year for which a Hope credit is "allowed" for the same student. To be "allowed," the Hope credit and the Lifetime Learning credit must be elected by the taxpayer. Thus, a taxpayer has a choice as to which credit he wants to claim. Similarly, a taxpayer can elect to have an otherwise tax-free distribution from a Coverdell education savings account not excluded from income. Other limitations: (1) Married taxpayers must file jointly to receive these credits. (2) In a given tax year, only one of the following benefits may be claimed with respect to each student: (a) the American Opportunity Tax Credit, or (b) the Lifetime Learning credit. However, the American Opportunity Tax or Lifetime Learning credit can be claimed in the same year as distributions from a Coverdell Education IRA (see section 4512.37), provided that the proceeds from the distribution are not used to pay for the education costs used in claiming the American Opportunity or Lifetime Learning credit. (3) The credits are not available if the cost of the course may be deducted by the taxpayer as a business expense.

Webstar Corp. orally agreed to sell Northco, Inc., a computer for $20,000. Northco sent a signed purchase order to Webstar confirming the agreement. Webstar received the purchase order and did not respond. Webstar refused to deliver the computer to Northco, claiming that the purchase order did not satisfy the U.C.C. Statute of Frauds because it was not signed by Webstar. Northco sells computers to the general public and Webstar is a computer wholesaler. Under the U.C.C. Sales Article, Webstar's position is: A. incorrect because it failed to object to Northco's purchaser order. B. incorrect because only the buyer in a sale-of-goods transaction must sign the contract. C. correct because it was the party against whom enforcement of the contract is being sought. Incorrect D. correct because the purchase price of the computer exceeded $500. You answered D. The correct answer

The Uniform Commercial Code (U.C.C.) Sales Article generally requires that sales contracts in excess of $500 be evidenced by a written document. If both parties are merchants, a document confirming the existence of an oral agreement and signed by the party sending the document satisfies the writing requirement as to the recipient as well as to the sender if the recipient does not object to the writing within 10 days after receipt. Therefore, Webstar's position is incorrect because Webstar failed to object to Northco's purchase order.

Name the requirements for a "qualifying child" exemption.

The requirements include: Close relative Age limit - 19/24 Residency and filing requirements Eliminate gross income test Support test

UCC Sales Article applies to

The sale of goods- all things moveable. Tangible personal property: cars, cows, groceries. Regardless of price

Who is granted the exemption claimed under a "multiple support agreement"?

Where two or more taxpayers together contribute more than 50% to support a person, but none of them contributes more than 50%, the contributing taxpayers may agree among themselves which contributor may claim the dependency exemption. The individual who receives the exemption: Must be a qualifying relative Must have contributed more than 10% of the person's support Must meet the other dependency tests

what can cause a scholarship to be considered earned income?

if the recipient has to work as a requirement of the scholarship, such as teaching 2 classes a week

are punitive damages available for innocent misrepresentation?

no

Qualified mortgage interest paid

Schedule A- itemized deduction

regulates original issuances of securities

Securities Act of 1933

regulates purchases and sales after initial issuance

Securities Act of 1934

Under the Sales Article of the U.C.C., in an auction announced in explicit terms to be without reserve, when may an auctioneer withdraw the goods put up for sale? At any time until the auctioneer announces completion of the sale If no bid is made within a reasonable time A. I only Correct B. II only C. Either I or II D. Neither I nor II

"With reserve" means the auctioneer can actually withdraw the item at any time before the gavel falls, or via an announcement to that effect. On the other hand, "without reserve" means the goods cannot be withdrawn and must be sold to the highest bidder (notice this accordingly requires an actual bid). Accordingly, if a bid is not made after a reasonable "lull" in the action, then and only then may the auctioneer withdraw the goods. While this may not seem important, unfortunately, as many large companies fail, and auctions are held for their various assets and thus the terminology and its understanding become critical.

LT contracts

% of completion to determine taxable income. Exceptions: 1. Small contractors 2. Home construction contractors

c. Sale or exchange of a personal residence:

(1) Individuals may exclude $250,000 ($500,000 on a joint return) of gain on the sale or exchange of a personal residence. Gains in excess of the excludible amount will be taxed. These excess gains may not be postponed by adjusting the basis of a replacement residence. (a) The residence must have been owned and occupied by the taxpayer for an aggregate of at least two of the five years before the sale or exchange. (b) The exclusion may be used only once every two years. (c) The $500,000 exclusion is available to married taxpayers filing jointly if (1) either spouse satisfies the ownership test, (2) both spouses meet the occupancy test, and (3) neither spouse has used the exclusion within the last two years. i. When the spouse has used the exclusion within the past two years, an eligible taxpayer may still exclude $250,000 on either a joint return or a separate return. ii. When a husband and wife each sell a principal residence, they are each eligible to exclude $250,000 on the sale of their residences. They may claim their exclusions on either a joint return or separate returns. iii. A surviving spouse can exclude up to $500,000 from the sale of a principal residence if the sale is within two years of the date of death and the other requirements were met on the date of death. (d) If the ownership and occupancy tests are not met, a prorated exclusion is available if the sale or exchange is the result of (1) change of place of employment, (2) health, or (3) unforeseen circumstances. (2) Losses are not recognized; neither are they postponed. (3) The basis of any new residence is its cost. There is no basis adjustment for unrecognized gains and losses.

Deductions subject to limit on itemized deductions

- Taxes paid - Interest paid - Gifts to charity - Job expenses and certain miscellaneous deductions - Other miscellaneous deductions

Multiple Support Agreements

-One supporter gets full exemption -Must be qualifying relative or lived with the taxpayer the whole year -Must have contributed > 10% of person's support

People who are receiving Social Security benefits, and who have not reached full retirement age, are entitled to receive all of their benefits as long as their earned income is under $15,480. For people born in 1943 through 1954, the full retirement age is 66. The full retirement age increases gradually each year until it reaches age 67 for people born in 1960 or later. Earned income includes:

1. 1. Wages, salaries, tips, and other taxable employee pay; 2. 2. Union strike benefits; 3. 3. Long-term disability benefits received prior to minimum retirement age; 4. 4. Net earnings from self-employment if: 1. a. You own or operate a business or a farm 2. b. You are a minister or member of a religious order; 3. c. You are a statutory employee and have income

Business net taxable loss. Deduct loss against other income. If loss > income excess NOL

1. 2 year carryback 2. 20 year carryforward

Bonus depreciation 1. Qualified property 2. Percentage 3. When is bonus depreciation claimed?

1. 20 years or less recovery period and original use of property commences with taxpayer 2. 2015-2017: 50% 2018: 40% 2019: 30% 3. After 179 expense deduction but before regular depreciation expense deduction

MACRS Real Property: Salvage Value ignored/Subtract land cost (for building only) 1. Residential rental property 2. Nonresidential real property 3. Mid month convention

1. 27.5 year straight line 2. Office buildings and warehouses. 39 year straight line 3. SL depreciation computed based on # of months property in service. 1/2 month taken month property placed in service, 1/2 month taken for which property is disposed of

Corporate capital loss carryover: 1. Carry back 2. Carry forward 3. Carried over as

1. 3 years 2. 5 years 3. Short term capital losses and applied only against capital gains

Child and dependent care credit 1. One dependent 2. Two dependents

1. 3000 2. 6000 *But limited to lowest earned income of either spouse

Individual capital losses 1. Offset income? 2. Carryback 3. Carryforward

1. 3000 2. No 3. Forever

Individual net capital loss 1. Offset other income 2. Carryback 3. Carryforward

1. 3000 maximum 2. 0 3. Unlimited

For AGI adjustment: self employment tax 1. SS/Medicare 2. Self-employed health insurance

1. 50% of Social Security/Medicare tax deducted 2. May deduct 100% of medical insurance premiums paid for taxpayer, spouse, and dependents

Homeowner's exclusion 1. Amount 2. To qualify

1. 500000 MFJ/SS; 250000 Single, MFS, HOH 2. Must have owned and used the property as a principal residence for 2 years or more during 5 year period ending on date of sale -Periods of ownership and use don't have to be continuous or cover the same 2 year period, but gain eligible for exclusion may be limited because of nonqualified use

Computing estimated tax payments due 1. Annualized method: 2. Prior year method

1. 90% of current year's tax 2. 100% of last year's tax; 110% if AGI > 150,000

Likelihood of tax position being upheld by courts 1. More likely than not 2. Reasonable basis 3. Substantial authority

1. > 50% 2. > 20% 3. > 33% but < 50%

S Corp: Contribution is tax free if:

1. A contribution of property (NBV)- NOT services (taxable at FMV) 2. Solely in exchange for stock 3. After transfer, shareholder or group of shareholders has control of the corporation through 80% ownership

Piercing the corporate veil means that a shareholder, director, or officer can be held personally liable for corporate obligations. In order for the "lifting" of the corporate veil to occur, two elements must be present:

1. A shareholder, director, or officer must have controlled the corporation for his or her own benefit in an attempt to protect himself or herself from legal liability. 2. A shareholder, director, or officer must have used the corporation in an improper manner, doing such things as perpetuating fraud, not capitalizing the organization adequately, or looting the corporation of assets.

Accrual basis for tax purposes required for:

1. Accounting for purchases and sales of inventory 2. Tax shelters 3. Certain farming corporations 4. C corporations, trusts with unrelated trade or business income, partnerships who have C corp as partner, provided basis > 5 million average annual gross receipts for 3 year period ending with the tax year

Itemized deduction: home mortgage interest- 2 types of qualified residence interest

1. Acquisition indebtedness (1,000,000 or 500,000 MFS, maximum) -Interest in excess of these amounts: personal, not deductible - Incurred in buying, constructing, or substantially improving taxpayer's principal and second home -secured by home 2. Home equity indebtedness (100,000 or 50,000 MFS, maximum) -Debt secured by TP's first or second residence, but not acquisition indebtedness

Principal liable (bound) if:

1. Agent acted with actual or apparent authority 2. Principal ratified the transaction

Elements of a legally enforceable contract

1. Agreement made up of an offer and acceptance 2. Exchange of consideration (something of legal value) 3. Lack of defenses *Remedies available if one party breaches (doesn't perform as promised)

Three requirements for attachment

1. Agreement: evidenced EITHER by an authenticated record of security agreement OR creditor's taking possession or control of the collateral. Record must be signed by debtor, not creditor 2. Value: Must be given by secured party in exchange for security interest. Loan is value 3. Rights: Debtor must have rights in the collateral, doesn't necessarily have to own the collateral

Corporation: Deductible Taxes

1. All state and local taxes 2. Payroll taxes Federal income tax not deductible, foreign income taxes as credit

Complex trust 1. What is it? 2. May accumulate: 3. May distribute: 4. May deduct: 5. Exemption to arrive at taxable income?

1. All trusts that are not simple trusts. Trust may be simple one year and complex the next. 2. Current income 3. Principal 4. Charitable contributions 5. 100

S Corp: Tax Payers at risk amount decreased by:

1. Allocable share of losses 2. Distributions of cash or other property 3. Only increase by recourse loans but not increase by nonrecourse loans

Nonqualified stock options: Employee Taxation 1. Option has readily ascertainable value at the time of the grant 2. Option does not have readily ascertained value

1. Employee recognizes ordinary income in that amount in the year granted. Taxable when granted. If there is a cost to the employee, ordinary income is the value of the option - cost. No taxation date of exercise *Options lapse: capital loss based on value of options previously taxed 2. On date of exercise, employee recognizes ordinary income based on FMV of stock - amount paid for the option. Taxed at exercise *Options lapse: no tax consequences

Workers compensation 1. Employee can collect if 2. Cannot collect if

1. Employee was negligent, grossly negligent, or assumed the risk- injury occurred in scope of employment 2. Injuries from intoxication, fighting, or self inflicted wounds

Estates: medical expenses can be deducted

1. Final income tax return of decedent or 2. Liability (estate tax return)

Calculating the foreign tax credit limitation by category

1. Foreign income sourced into separate categories: passive income, general income, income earned from activities in sanctioned countries, certain income resourced by treaty, foreign source lump sum distribution from a pension plan 2. Pre-credit US tax on total taxable income x (Separate category foreign income/Total taxable income)

1) 3 periodic reports 2) Other required reports (5% TIP)

1. Form 10K- filed annually. within 60 days for large companies. 90 for small 2. Form 10Q: Quarterly report. 40 days large companies; 45 small 3. Form 8-K: major events 2) - Any person acquiring 5% or more beneficial ownership in equity security must file report with SEC - Tender offer - Insider trading - Proxy statements

No gain or loss to corporation issuing stock in exchange for property in these events:

1. Formation: Issuance of common stock 2. Reacquisition: purchase of treasury stock 3. Resale: Sale of treasury stock

****Gain/loss basis Rollover cost basis exception: Lower FMV at date of gift 1. Sale of gifts at price greater than donor's rollover basis 2. Sale of gift at price less than lower FMV 3. Sale less than rollover cost basis but greater than lower FMV

1. Gain is difference between sale price and rollover basis 2. Basis of gift for determining loss is FMV of gift at time gift was given. FMV - selling price 3. Neither a gain or loss is recognized. Basis to donee= Middle selling price

Other miscellaneous deductions not subject to 2% AGI test

1. Gambling losses: to extent of gambling winnings 2. Federal estate tax paid on income in respect of a decedent

Requisites for creation of an agency 1. Is writing required? 2. Consent 3. Capacity? 4. Consideration?

1. Generally not required unless agent is entering into land sale contracts 2. Need consent 3. Only principal needs to be competent- a minor or mentally incompetent person can be appointed as an agent 4. No consideration necessary

1. Loans affected by imputed interest rules: 7872 prevents parties from offering below market interest rates on loans that could be appealing to related parties

1. Gifts 2. Compensation-related loans 3. Corporation-shareholder loans 4. Tax avoidance loans 5. Other below-market loans 6. Loans to qualified continuing care facilities

De minimis exception to imputed interest rules

1. Gifts loans between individuals < 10000 2. Compensation related loans < 10000 3. Corporation shareholder loans <10000

Grantor Trusts 1. What is it? 2. Taxable income or deduction of a grantor trust 3. Grantor trust can be a qualified shareholder of ? 4. Taxable estate of the grantor upon death

1. Grantor (individual who established the trust) retains control over the trust assets 2. Disregarded entity. Reported on the income tax return of the grantor 3. S corporation 4. Grantor trust generally included

Realized gains recognized unless: HIDE IT

1. Homeowner's exclusion 2. Involuntary conversion 3. Divorce property settlement 4. Exchange of like-kind (business) 5. Installment sale 6. Treasury capital and stock

De minimis policy 1. Have a written policy to expense 2. Don't have written policy

1. If you have a written policy to expense certain property beginning of year. Allowed to expense items up to 5000 each 2. Expense items up to 2500 ***Property with economic useful life of 12 months or less

Surety: Debtor defaults Creditor may do any of the following:

1. Immediately demand payment from the surety 2. Immediately demand payment from the debtor 3. Immediately go after collateral, if there is any

Who must file?

1. Income greater than or equal to the sum of: Personal exemption + Standard Deduction (Except MFS) + Additional Standard Deduction (65 or older or blind) 2. Net earnings from self-employment > 400 3. Dependents -Unearned income: 1050 -Earned income: 6350 -Total income exceeds larger of 1050 or earned income + 350

Business interest expense 1. On business 2. On investment 3. Prepaid interest expense

1. Incurred & paid= deduction 2. Up to taxable investment income 3. Deduct later when incurred

S Corp: 1. Eligible shareholders 2. Shareholder limit 3. Class of stock

1. Individual, estate, or certain types of trusts (USA). Can't be a nonresident alien. CORPORATIONS AND PARTNERSHIPS ARE NOT ELIGIBLE. 2. May not be > 100 shareholders 3. May not be more than one class of stock outstanding. But differences in CS voting rights allowed. Preferred stock not permitted.

Partnership 1. Form 1065 2. Schedule K 3. Schedule L: 1065 4. Schedule M-1 5. Schedule M-2

1. Information return, no taxes 2. Totals for whole partnership. Each partner gets their own K-1 3. Balance sheet per books- beginning of tax year, end of tax year 4. Reconciliation of income (loss) per books with income (loss) per return 5. Analysis of partner's capital accounts

Sourcing foreign income Items of income that should be treated as sources of income from within the US

1. Interest 2. Dividends 3. Compensation for labor or personal services performed in the US 4. Rents and royalties 5. Gains, profits, and income from disposition of a US real property interest 6. Sale or exchange of inventory property within the US 7. Underwriting income 8. Social security benefits 9. Guarantees

for 2019, employees were liable to make Social Security contributions of ________ % of their entire gross wages

1.45%

what is the hierarchy of paying of dividing up cash to creditors/IRS after a bankruptcy?

1.security interests are paid first2.child support and alimony3.admin costs such as attorneys and accountants who helped DURING the bankruptcy4. employee wages 3 MONTHS PRIOR5. contributions to benefit plans6. claims on raising or storage of grain7. consumer deposits8. all taxes are paid next9. unsecured creditors who filed in a timely fashion are paid pro rata and MUST BE FULLY REPAID before anything is paid to unsecured creditors who DID NOT file in a timely fashion

Self Employment tax

92.35% of SE income, then multiply by 15.3%

Peters has a 1/3rd interest in the Spano Partnership. During Year 7, Peters received a $16,000 guaranteed payment, which was deductible by the partnership, for services rendered to Spano. Spano reported a Year 7 operating loss of $70,000 before the guaranteed payment. What are the net effects of the guaranteed payment? 1. The guaranteed payment increases Peters' tax basis in Spano by $16,000 2. The guaranteed payment increases Peters' ordinary income by $16,000.

=2 The guaranteed payment is treated much like self-employment income. It is declared as ordinary income, and subject to self-employment taxes. Since this income is not reinvested in the company, it does not increase the investment basis.

Jones, an individual taxpayer, must include $1,000 in gross income resulting from a state tax refund he received in the current year. Jones is in an alternative minimum tax situation for the year. Which of the following is the correct statement in regards to the tax refund received by Jones? Incorrect A. The $1,000 tax refund is a positive adjustment in the calculation of alternative minimum taxable income (i.e., will increase alternative minimum taxable income). B. The $1,000 tax refund is a negative adjustment in the calculation of alternative minimum taxable income (i.e., will decrease alternative minimum taxable income). C. The $1,000 tax refund is not an adjustment in the calculation of alternative minimum taxable income. D. There is not sufficient information to tell if it is or is not an adjustment to taxable income.

=B The $1,000 tax refund is a negative adjustment in the calculation of alternative minimum taxable income. The state tax refund is not included in income for AMT purposes. Also, state and local taxes are not deductible for AMTI purposes. "Taxes REDUCED by taxable refunds (IF refunds meet the tax benefit rule) are added back" So yes-tax expense is ADDED BACK...however if don't have an expense and just a refund then its REDUCES AMT. If you have both a tax expense and a tax refund-like becker says, it's netted and if the net amount is an expense then its an add back...if its a refund then its a negative adjustment.

Which of the following taxpayers may use the cash basis as its method of accounting for tax purposes? A. Partnership that is designated as a tax shelter Incorrect B. Retail store with a $2 million inventory C. An international accounting firm D. Incorporated office cleaning business with average annual income of $12 million

=C Tax shelters (regardless of ownership form) are generally prohibited from using the cash basis. Accrual basis accounting must be used for the sale and expensing of inventory (with limited exception for very small businesses). Even C corporations that perform accounting services can qualify as personal service corporations and are allowed to use the cash method. Corporations or partnerships with a C corporation partner with annual gross receipts in excess of $5 million for any year must generally use the accrual method. IRC Sections 446 and 448; Regulation Section 1.446-1 Small business exception: The IRS allows small businesses with average annual gross receipts of less than $1 million to use the cash method, even if they have inventories. The cost of items otherwise required to be inventoried are not deductible until sold. Revenue Procedure 2001-10 Qualified small business exception: The IRS also allows certain taxpayers with qualifying businesses to use the cash method if they have average annual gross receipts under $10 million. Ineligible businesses include mining activities, manufacturing, retail and wholesale trade, and information industries

Under the Sales Article of the U.C.C., which of the following statements regarding liquidated damages is (are) correct? I. The injured party may collect any amount of liquidated damages provided for in the contract. II. The seller may retain a deposit of up to $500 when a buyer defaults even if there is no liquidated damages provision in the contract.

=II. U.C.C. 2-718 provides that "damages for breach of either party may be liquidated in the agreement but only at an amount which is reasonable in the light of the anticipated or actual harm caused by the breach..." (U.C.C. 2-718). Thus, it is not correct to state that any amount of damages may be collected if provided for in the contract. This same article stipulates that the seller may retain a deposit of up to 20% of the value of total performance or $500, whichever is smaller.

S Corp status can be revoked if shareholders owning:

> 50% of total number of issued and outstanding shares consent. Any combination of voting and non voting stock

Which of the following actions is required to ensure the validity of a contract between a corporation and a director of the corporation?

A corporation is permitted to enter into a contract for services or goods with a board member (director). This type of a transaction is called a "related party transaction." This action could be seen by shareholders as preferential treatment to the director who receives the contract, and it could be interpreted as a lack of due care on the part of the directors in carrying out the corporation's business. In order to invoke the business judgment rule, where the directors are protected from shareholder lawsuits alleging a lack of due care, the board must: make an informed decision, eliminate conflict of interest, and have a rational basis for the decision. A rational basis for the decision could be that these services or products are not available elsewhere, or the director is offering the best quality for the lowest price (which would be in the shareholders' favor). In order to make an informed decision, the board must review all of its options and then come to the conclusion that the best decision is to contract with the director. Finally, to eliminate conflict of interest, the director must disclose his or her interest in the contract to the board and refrain from voting. It is not necessary that the contract be reviewed by an independent appraiser, that the shareholders approve the contract, or that the director resign.

Apex, Inc., is a small calendar-year corporation with annual gross receipts as follows: Year 1 ($4 million), Year 2 ($5 million), Year 3 ($6 million), and Year 4 ($7 million). Its 3-year average annual gross receipts are $5 million in Year 3 and $6 million for Year 4. Therefore, it is exempt from alternative minimum tax for Year 5. What happens if Apex has annual gross receipts of $8 million for Year 5?

A small corporation loses the AMT exemption only if its *average* annual gross receipts for three consecutive years exceed *$7.5 million*. Although Apex gross receipts for Year 5 exceeded $7.5 million, its 3-year average gross receipts for Year 3 through Year 5 were only $7 million. Therefore, it retains its status as a small corporation that is exempt from AMT.

What are the requirements of the Child and Dependent Care credit?

A tax credit of 20% to 35% for eligible expenses up $3,000 for one child; $6,000 for two or more children A qualifying child, under age 13, for whom an exemption can be claimed Both spouses must work Eligible expenses are for the purpose of enabling the taxpayer to be gainfully employed

John Johnson is 60 years old and has had a non-Roth IRA since Year 2. During Year 10, he rolls over his existing IRA into a new Roth IRA. What is the first year he can take tax-free distributions from his Roth IRA?

A tax-free distribution must take place after the 5-tax-year period that begins with the first tax year (Year 10) in which a contribution was made to a Roth IRA or rolled over from a non-Roth IRA to a Roth IRA. In this case, the 5-tax-year period would be from Year 10 through Year 14, inclusive. Thus, the first year that a distribution could be tax-free would be Year 15.

The individual partner rather than the partnership makes which of the following elections?

A. Election to amortize organizational costs B. Nonrecognition treatment for involuntary conversion gains C. IRC 179 deductions for tangible personal property Correct D. Whether to take a deduction or credit for taxes paid to foreign countries There are many income and expense items where the partnership must decide on the accounting treatment, especially where there are choices in the treatment of each item. These include the amortization of organization costs, treatment of involuntary conversions, and the option of Section 179 expensing of equipment. The choice of choosing the credit or the use of a deduction for foreign taxes is left to the individual partner.

When there has been no performance by either party, which of the following events generally will result in the discharge of a party's obligation to perform as required under the original contract? A. Accord and satisfaction Incorrect B. Mutual recission C. Both accord and satisfaction and mutual recission D. Neither accord and satisfaction nor mutual recission

Accord and satisfaction is carrying out an agreement between two contracting parties where some different performance will replace the original performance. Accord and satisfaction discharges the contractual obligation. Mutual rescission is a joint agreement to call off the contract and replace it with another. Both accord and satisfaction and mutual rescission will discharge a party's obligation to perform under the original contract. It is key in this question to note that there has been no performance by either party. If one party has fully performed, then an agreement to call off a contract will normally not be enforceable.

Which exempt organizations don't have to file annual information returns?

According to IRC Section 6033, churches, internally supported auxiliaries of churches, and organizations with not more than $5,000 in gross receipts do not have to file.

Fil and Breed are 50% partners in F&B Cars, a used-car dealership. F&B maintains an average used-car inventory worth $150,000. On January 5, National Bank obtained a $30,000 judgment against Fil and Fil's child on a loan that Fil had cosigned and on which Fil's child had defaulted. National sued F&B to be allowed to attach $30,000 worth of cars as part of Fil's interest in F&B's inventory. Will National prevail in its suit? A. No, because the judgment was not against the partnership B. No, because attachment of the cars would dissolve the partnership by operation of law Incorrect C. Yes, because National had a valid judgment against Fil D. Yes, because Fil's interest in the partnership inventory is an asset owned by Fil

According to Section 305(a) and (b) of the Uniform Partnership Act, "A partnership is liable for loss or injury caused to a person, or for a penalty incurred, as a result of a wrongful act or omission, or other actionable conduct, of a partner acting in the ordinary course of business of the partnership or with authority of the partnership....If, in the course of the partnership's business or while acting with authority of the partnership, a partner receives or causes the partnership to receive money or property of a person not a partner, and the money or property is misapplied by a partner, the partnership is liable for the loss." In the situation described in this question, Fil was not acting on behalf of the partnership nor did he act with the authority of the partnership when he cosigned on his child's auto loan; therefore, National will not prevail.

Under the Revised Model Business Corporation Act, which of the following conditions is necessary for a corporation to achieve a successful voluntary dissolution?

According to section 14.02(a) and (b) of the Revised Model Business Corporation Act (RMBCA), "The *board of directors* must *recommend* dissolution to the shareholders...and the shareholders entitled to vote must approve the proposal to dissolve." This *vote must be approved* by a *majority of all the shareholders*.

what is the half-year convention under MACRS?

According to the rules of MACRS, personal property is treated as placed in service or disposed of at the midpoint of the taxable year, resulting in a half-year of depreciation for the year in which the property is placed in service or disposed of by the company.

Personal service corporations

Accounting, law, consulting, engineering, architecture, health, actuarial science. Can't use graduated corporate rates. Taxed at a flat 35%

Corporation AMT: Regular taxable income: Preferences

Add back to increase income: 1. Percentage depletion 2. Private activity--issued post 1986. Tax exempt interest income. 3. Pre-1987 ACRS depreciation

Corporation AMT: Regular taxable income: Adjustments

Add or minus adjustment items to income: 1. Adjustments for gain/loss 2. Long-term contracts 3. Installment sales 4. Excess depreciation

State allocation of Federal Taxable Income

Allocable items of income are nonbusiness income. Remove nonbusiness income and assigning it entirely to the state where it should be taxed, generally state of taxpayer's commercial residence

Personal property tax

Allowable itemized deduction

Ratification

Allows a principal to choose to become bound by a previously unauthorized act of his agent *Agent must have apparent authority *Only purported principal may ratify *Undisclosed principal may not ratify

Amount that can be contributed to a Roth IRA

Amount remaining after subtracting any contribution made to a regular IRA

Describe the self - employment tax.

An adjustment to income is allowed for one-half of S/E tax paid. All self-employment tax is subject to the 2.9% Medicare tax. Up to $106,800 is subject to the 12.4% Social Security tax.

Creditor's composition

An agreement between the debtor and at least 2 creditors that the debtor pays the creditors less than full claims in full satisfaction of their claims. Results in debtor being discharged in full for debts owed the participating creditors after debtor has paid agreed amount

Mel purchased 100 shares of common stock in X Corporation for $1,000. X distributed a nontaxable stock dividend and Mel received 20 shares of preferred stock as a result. On the date of the dividend, the common stock had a value of $19 per share and the preferred had a value of $5 per share. After the distribution of the preferred stock, Mel's bases for the stock held in X Corporation are: Incorrect A. $1,000 common and $0 preferred. B. $950 common and $50 preferred. C. $830 common and $170 preferred. D. $792 common and $208 preferred.

Because the stock dividend was nontaxable, the $1,000 original basis of Mel's common stock must be allocated between the common and preferred shares based on their relative fair market value. Common stock has a value of $1,900 (100 × $19) and preferred has a value of $100 (20 × $5). $1,000 × (1,900 ÷ 2,000) = $950 basis of the common stock. $1,000 × (100 ÷ 2,000) = $50 basis of the preferred stock.

Advanced Pricing Agreement Program

Binding agreement with IRS that they will not seek transfer pricing adjustment if taxpayer files return consistent with the agreed transfer pricing method

Assignment for the benefit of creditors

Debtor transfers some or all of his or her property to a trustee, who disposes of the property and uses the proceeds to satisfy the debtor's debts. Debtor is not discharged from unpaid debts

Corporation: organizational expenditures and start up costs: Excluded costs

Costs of issuing and selling stock, commissions, underwriter's fees, costs incurred in transfer of assets to a corporation

Remedies of the guarantor or surety include the following:

Defense: Use a defense to avoid payment to the creditor. Reimbursement or indemnity: Get the principal debtor to pay the guarantor or surety for the amount the guarantor or surety had to pay the creditor. Subrogation: When the guarantor or surety discharges the principal debtor's obligation to the creditor, the guarantor or surety gets all the creditor's rights regarding the obligation. Contribution: From co-guarantor or co-sureties for paying more than legally obligated. It does not appear likely that any of these would be successful.

Guaranteed payments to the partners are

Distributive deductions to the partners via the partnership K-1 and also taxable income to the partner receiving the payments

Thomas filed a voluntary petition for bankruptcy on July 1, 20X1. As part of the petition, Thomas listed his assets, which include his permanent residence. The residence, which was recently appraised for $125,000, is subject to a mortgage in favor of Security Finance in the amount of $100,000. Assuming that Thomas properly claims a homestead exemption under the federal exemptions, he will be entitled to an exemption in the amount of: A. $7,500. B. $23,675. Incorrect C. $25,000. D. $125,000.

Each state has its own set of exemption laws (California has two sets). A handful of states allow bankruptcy filers to use the federal bankruptcy exemptions instead of their state bankruptcy exemptions. The amounts allowed under the federal bankruptcy exemptions are adjusted every three years ending on April 1 to reflect changes in the Consumer Price Index and were last adjusted in 2016; the federal homestead exemption is $23,675. If you are married and filing jointly, you may double all of the federal bankruptcy exemptions. This exemption is not effective against a secured creditor, but in this case, the value of the property is sufficient to pay the $100,000 due Security Finance as well as to pay Thomas the amount of the exemption ($23,675).

When is a taxpayer eligible for the Elderly or Permanently Disabled credit?

Eligible individuals are those that are: 65 years of age or older or Under 65 and retired due to permanent disability Credit of 15% of eligible income Reduce eligible income by: Social security payments One-half of AGI that exceeds: Single: $7,500 MFJ: $10,000 MFS: $5,000

501 C 3 Organization

Exempt. Community chest, community fund, foundation organized and operated exclusively for religious, charitable, scientific, public safety testing, literary, educational expenses, foundation organized to foster national or international amateur sports competitions (only if none of the activities involve providing athletic facilities or equipment)

Net section 1231 losses

Fully deductible against ordinary income

Liquidation of a partnership: 2. Sale of partnership interest

GR: partner has a capital gain or loss when transferring a partnership interest because partnership interest is a capital asset

Explain Gifted Property Basis.

General Rule: Donor's rollover cost basis Property acquired as a gift generally retains the rollover cost basis as it had in the hands of the donor at the time of gift. Basis is increased by any gift tax paid that is attributable to the net appreciation in the value of the gift. The recipient of the gift normally assumes the donor's holding period.

What is the statute of limitations applicable to taxpayers?

General: Three years from the later of: Due date of the return Date return is filed 25% understatement of Income: Six years from the later of: Due date of the return Date return is filed Fraud or false returns: No statute of limitations

Magic Corp., a regular C corporation, elected S corporation status at the beginning of the Year 3 calendar year. It had an asset with a basis of $40,000 and a fair market value (FMV) of $85,000 on January 1, Year 6. The asset was sold during Year 6 for $95,000. Magic's corporate tax rate was 35%. What was Magic's tax liability as a result of the sale? Incorrect A. $0 B. $3,500 C. $15,750 D. $19,250

Generally an S corporation is not subject to income tax. The company's taxable income or loss flows through to the shareholders and is reported on their individual income tax returns. One exception to this is the built-in gains tax. When a regular C corporation converts to S corporation status, a tax may be imposed on the net appreciation of the assets owned at the time of conversion if they are sold within five years. The built-in gains tax is at the highest corporate tax rate. Magic's tax liability as a result of the sale is as follows: Fair market value of asset at conversion $85,000 Basis (40,000) -------- Unrealized built-in gain $45,000 Sale price $95,000 Basis (40,000) -------- Realized gain $55,000 Built-in gains tax: Lower of unrealized built-in gain or realized gain $45,000 x Tax rate x 0.35 ------- Built-in gains tax $15,750

What is the tax treatment for income received from a vacation home?

If rented less than 15 days: Personal residence If rented more than 15 days and is used for personal reasons for the greater of more than 14 days or more than 10% of rental days: Personal/Rented residence Expenses are prorated between personal and rental use. A different pro-ration method is used for mortgage interest and property taxes. Rental use expenses are deductible only to the extent of rental income.

A first-time homebuyer, for purposes of favorable IRA distribution treatment, is one who has not had a present ownership interest in another principal residence for what minimum period before a new principal residence is purchased?

IRA distributions qualify as made for first-time homebuyer expenses if: the homebuyer is the taxpayer, spouse, child or grandchild of either, or ancestor of either, the home is used as a principal residence by the homebuyer, the homebuyer (and if married, the homebuyer's spouse) has not had a present ownership interest in another principal residence within the 2-year period ending on the date that the current principal residence is acquired, and the distribution is used within 120 days to pay for qualified acquisition expenses, such as buying, building or reconstructing the residence, as well as usual or reasonable settlement, financing, or other closing costs. Only $10,000 of aggregate distributions received by an individual can be treated as made for qualified first-time homebuyer expenses. This is a lifetime limit.

Explain how IRA distributions are taxed.

IRA income is taxed as ordinary income when received. (Regardless of what type of income was earned while the funds were invested) Roth IRA - All qualified benefits from a Roth IRA are non-taxable. Traditional Non-deductible IRA - Benefits received from this type of IRA are partially taxable. (1) Principal - nontaxable (2) Accumulated earnings - taxable

If the amount of a tax case is not more than $25,000, the taxpayer may:

IRS Publication 5 recommends filing a small case request if the amount of the tax is not more than $25,000. This is done in place of filing a formal written protest.

Which of the following statements is correct if a taxpayer agrees to changes made during an examination and he signs an agreement, but does not pay the taxes due?

IRS Publication 556 provides that a taxpayer's return may be examined for a variety of reasons, and the examination may take place in any one of several ways. After the examination, if any changes to a taxpayer's tax are proposed, either the taxpayer can agree with those changes and pay any additional tax owed, or the taxpayer can disagree with the changes and appeal the decision. Publication 556 states, in part, that if a taxpayer agrees with the proposed changes after the examination, the taxpayer can sign an agreement form and pay any additional tax he or she may owe. A taxpayer must pay interest on any additional tax. If the taxpayer pays when he or she signs the agreement, the interest is generally figured from the due date of the return to the date of the payment. If the taxpayer does not pay the additional tax when he or she signs the agreement, the taxpayer will receive a bill that includes interest. If the taxpayer pays the amount due within 10 business days of the billing date, the taxpayer will not have to pay more interest or penalties. This period is extended to 21 calendar days if the amount due is less than $100,000. Given the information above, if the taxpayer pays the amount due is within 21 calendar days of the billing date and the amount is less than $100,000, the taxpayer will not have to pay more interest or penalties.

How to calculate the kiddie tax:

If a child has interest, dividends, and other *unearned income* of *more than $2,000*, part of that income may be taxed at the parent's tax rate instead of the child's tax rate. The "kiddie tax" is a tax on unearned income paid to minors. For 2016: 1. the first $1,050 of such income is tax free 2. the second $1,050 is taxed to the child at his/her tax rate 3. all unearned income over $2,100 is taxed at the parents' tax rate. The kiddie tax rule now applies to children under age 19 and full-time college students under the age of 24. Since Chris has $3,000 of unearned income, the amount greater than $2,100, or $900, would be taxed at Chris's parents' maximum tax rate. The child must be required to file a tax return. The child's tax is calculated on IRS Form 8615, Tax for Certain Children Who Have Unearned Income, and included with the child's return

what is the rule on excluding a gain on the sale of a primary residence?

If certain rules are met, a couple filing a joint return can exclude up to $500,000 of the gain on their personal residence. The structure had to be their principal residence for at least two of the most recent five years. This exclusion cannot be taken in two consecutive years.

Means test for CH. 7 bankruptcy

If income is between 7700-12850 above the mean income for that state, then a presumption of abuse arises if the amount equals at least 25% of the debtors unsecured claims not entitled to priority payment.

Which of the following actions between a debtor and its creditors will generally cause the debtor's release from its debts? A. Composition of creditors B. Assignment for the benefit of creditors Incorrect C. Both composition of creditors and assignment for the benefit of creditors D. Neither composition of creditors nor assignment for the benefit of creditors

In a *composition agreement*, a debtor enters into an agreement with his creditors whereby the debtor agrees to pay the creditors some fraction of the amount of the outstanding debt. Such an agreement discharges the debtor once the debt is actually paid. An *assignment for the benefit of creditors* is a transfer by the debtor of all of his assets to a trustee, with the assigned funds to be used to pay off outstanding debts. This does not have the effect of releasing the debtor from the debts.

how does the corporate AMT small corp exemption work?

In the first year of a corp's existence it is automatically exempt from AMT.Year 2's test is the gross receipts from year one.If year one's gross receipts exceed 5million, then the corp is NOT exempt from AMT in year 2.Once the small corp test is failed, the corp is NOT exempt for all future tax years.The general test is 7,500,000- the above 5million test is for NEW corps.

Trusts subject to which tax?

Income tax only. Separate tax paying entities. Classified as either simple or complex Must use a calendar year

When determining income amounts for the support test, what types of income are not included?

Income types that are not included are: Social Security Tax - exempt interest income Tax - exempt scholarships To claim an exemption for a qualifying relative, a dependent's income must be less than the exemption amount. Only income that is taxable is included in the support test.

Stock basis for an S CORP is adjusted annually, as of the last day of the S corporation year, in the following order:

Increased for income items and excess depletion; Decreased for distributions; Decreased for non-deductible, non-capital expenses and depletion; and Decreased for items of loss and deduction.

What are the criteria for filing as Head of Household?

Individual is not married, legally separated if married, and has lived apart from his or her spouse for the last six months of the year Individual is not a "qualifying widower" Individual is not a non resident alien Individual maintained a home that, for more than half the taxable year is the principal residence of a: Son or daughter who is a qualifying child or qualifies as the taxpayer's dependent (qualifying relative); A dependent relative who resides with taxpayer; or A dependent father or mother, regardless of whether they live with the taxpayer.

Shareholder Basis in S Corporation stock:

Initial basis + Income items (separately and non separately stated items; includes tax free income) + Additional shareholder investments in corporation stock < Distributions to shareholders > < Loss or expense items > = Ending basis

Basis of contributing partner's interest=

Initial basis: Cash (amount contributed) + Property (Adjusted basis NBV) < Liabilities > (incoming partner's liabilities assumed by others- not 100%) + Services (FMV, and taxable to incoming partner is used in exchange for a capital interest) + Other partner's liabilities assumed by the incoming partner, loans guaranteed by the partner, loans made by the partner to the partnership

When should a corporation make estimated tax payments?

Installment payments are due by the 15th day of the 4th, 6th, 9th, and 12th months of the corporation's tax year. Installment payments are required if the estimated tax is $500 or more. If any due date falls on a Saturday, Sunday, or legal holiday, the installment is due on the next regular business day.

Involuntary conversions of property: gain may be deferred if

Insurance proceeds are reinvested in property that is similar or related in service or use within 2 years for personal property or 3 years for business property

Regulation A

Intended to allow small companies to make initial public offerings

Green is self-employed as a human resources consultant and reports on the cash basis for income tax purposes. Select the appropriate tax treatment on Form 1040 (U.S. Individual Income Tax Return) for interest expense on a home-equity line of credit for an amount borrowed to finance Green's business.

Interest expense on a home equity line of credit for an amount borrowed to finance Green's business is fully deductible in Schedule C, Profit or Loss From Business. Even though the home was used as equity for the loan, the interest is fully deductible as a business expense on Green's Schedule C. (This is possible by electing under Regulation 1.163-10T(o)(5) to not treat it as home mortgage interest but instead to use the tracing rules under Regulation 1.163-8T(c) to treat it as business expense since the proceeds were used for this business.)

Foregone interest=

Interest that would be payable for that period if interest accrued at applicable federal rate (AFR) and was payable annually on December 31 < Interest actually payable on loan for the period > ^^ the below market interest rate

Ordinary and necessary expenses paid by the fiduciary of an estate are deductible:

On the fiduciary income tax return only if the estate tax deduction is waived for these expenses

Morgan, a sole practitioner CPA, prepares individual and corporate income tax returns. What documentation is Morgan required to retain concerning each return prepared? A. An unrelated party compliance statement Correct B. Taxpayer's name and identification number or a copy of the tax return C. Workpapers associated with the preparation of each tax return D. A power of attorney

One of the requirements that the IRS imposes on tax preparers is that records be maintained for at least three years for each return. The records that the IRS requires to be maintained include the taxpayer's name and identification number or a copy of the tax return. A penalty of $50 per incident up to $25,000 can apply to preparers who fail to maintain the required records. (IRC Section 6695(d)) In addition to the requirements of the IRS, each state may impose record keeping requirements which may include retention of the tax return and workpapers and/or longer holding periods.

How is the payment of child support treated for tax purposes?

Payments fixed by the divorce agreement for the support of minor children: The paying spouse: not deductible The receiving spouse: not included in income. Note: If payments are for both alimony and child support, amounts will be allocated first to child support and then to alimony. (If spouse falls behind in payment)

Surety

One who agrees to be liable for the debt or obligation of another. Involves 3 parties: creditor, principal debtor, and the surety

UBI specific deduction

Only UBI in excess of 1000 is subject to tax

Who may be a debtor?

Only a person who resides, has a place of business, in the US Persona: individuals, partnerships, corporations

Tax Practitioner Privilege

Only asserted in noncriminal tax matter: IRC Not applicable to tax shelters or criminal cases

Half year convention: Depreciation in the year of sale

Only half is taken

Violation of section 11 of 33- what damages available?

Only monetary

Section 1250 recapture

Only portion of depreciation taken on real property in excess of straight line. *Generally 1250 applies only to assets placed in service under pre-1987 accelerated methods of depreciation

S Corp: taxability of distributions to shareholders

Only subject to single level of tax, distributions from an S corporation generally not subject to taxation for shareholders. However, if S Corp has E&P which carried over from the former C corp when S election made: distributions may be taxable

Substituted contract

Original parties both released from the original agreement but both are bound by a new agreement

Defendents for violation of section 11: Defense

Other than issuers- Not liable if they can prove they used due diligence: defendant had reasonable grounds to believe the facts in the registration statement were true and no material facts were omitted (auditor can defend that she complied with GAAS)

Describe the tax treatment of payments pursuant to a divorce.

Payments for the support of a spouse are income to the spouse receiving payments and are deductible to arrive at adjusted gross income by the contributing spouse. To be deemed alimony under the tax law: Payments must be legally required to a written divorce agreement; Payments must be in cash; Payments cannot extend beyond the death of the payee-spouse; Payments cannot be made to members of the same household; and Payments must not be designated as anything other than alimony.

Benefits received from traditional nondeductible IRA

Partially taxable. 1. Principle: nontaxable 2. Accumulated earnings: taxable when withdrawn

Ultramares: Liable to negligence to

Parties in privity of the contract

Why type of business entity can be formed without filing organizational documents with the state?

Partnership or sole proprietorship

Fraud in the inducement

Party is aware they are making a contract, but terms are materially misrepresented. Makes a contract voidable

Paul Pappas owns all of the stock of an S corporation which had previously been a C corporation. The S corporation had the following balances at the beginning of its tax year: Accumulated adjustments account $ 8,000 Accumulated earnings and profits 10,000 Paul's stock basis was $20,000 at the beginning of the tax year. The S corporation made a distribution of $19,000 to Paul during the year. What is Paul's stock basis at the end of the year? Incorrect A. $1,000 B. $2,000 C. $11,000 D. $12,000

Paul's basis is reduced by the distribution from accumulated adjustments account, but not by the distribution from accumulated earnings and profits which is taxable income to Paul. The distribution in excess of $18,000 is a tax-free return of capital and reduces Paul's basis ($20,000 - $8,000 - $1,000 = $11,000).

Payments to be made for both alimony and child support, but payments subsequently made fall short of fulfilling these obligations

Payments allocated first to child support until entire child support obligation is met, and then to alimony

The answer to each of the following questions would be irrelevant in determining whether a tuition payment made on behalf of another individual is excludible for gift tax purposes, except: A. was the tuition payment made for a part-time student? Incorrect B. was the qualifying educational organization located in a foreign country? C. was the tuition payment made directly to the educational organization? D. was the tuition payment made for a family member?

Payments must be made to a qualifying educational organization and may only be for tuition to be excludible for gift tax purposes. Therefore, if a student is part-time or full-time, if the student is located in a foreign country, or whether a family member made a tuition payment is irrelevant. The only relevant answer is if the tuition payment was made to the educational organization.

Discuss the Charitable Contributions deduction.

Payments or property given to a charitable organization during the year are deductible. The maximum allowable deduction is: Cash = 50% of adjusted gross income General property = Lesser of basis or FMV Long-term appreciated property = limited to the lesser of: 30% of AGI The remaining amount to reach 50% after cash contributions A taxpayer may only deduct the excess contribution over the consideration received. Excess charitable contributions can be carried over for 5 years

Retirement or death of partner

Payments to retiring partner or to the interest successor of a deceased partner in liquidation of his entire interest: allocated between payment for an interest in partnership assets and other payments Payments for interest in partnership assets: Capital gain or loss

Forfeited Interest

Penalty for early withdrawal of savings. Interest received is taxable, but amount forfeited is also deductible as adjustment in year penalty is incurred. Amount of forfeited interest is deducted separately and not netted with interest income on the tax return

List some other miscellaneous deductions FOR AGI.

Penalty on early withdrawal of savings - interest forfeited can be deducted. Alimony - deducted by contributing spouse. If payments are for both child support and alimony and the payer falls behind payments apply to child support first. Attorney fees paid in discrimination/whistleblower cases - deduction limited to the amount claimed as income from the judgment. Domestic Production Activities - deduct a percentage (9%) of qualified production activities income or the taxpayer's taxable income before the adjustment, whichever is less

Accumulated earnings tax

Penalty tax on C corporations whose retained earnings > 250000; improperly retained instead of being distributed as dividends. Personal services corporations: 150000 Additional tax rate: flat 20%

Noninventory PMSI: Priority if

Perfected within 20 days after the debtor receives possession of the collateral. Perfection relates back to the date of possession

Usually, the first security interest to have priority will be the first one __________.But what is the exception dealing with purchase money?

Perfected.A purchase money security interest in NON-INVENTORY collateral has priority if it is perfected before the debtor takes possession or within 20 days thereafter.

PMSI in inventory

Perfection requires filing. To have priority, must be perfected before the debtor gets possession and notice must be given to other perfected parties in the same collateral

Gift tax: tax is paid by

Person giving the gift

Rented < 15 days of the year

Personal residence. *Rental income excluded *Mortgage interest and real estate taxes: itemized deductions *Depreciation, utilities, repairs: not deductible

Details of personal use assets for tax purposes:

Personal use assets, such as a BOAT, are capital assets. BUT, if you have a loss when you sell a personal use asset IT IS NOT DEDUCTIBLE. It is of course taxable if you have a gain though.

Writ of certiorari

Petition for the Supreme Court to review a lower court's decision

Green is self-employed as a human resources consultant and reports on the cash basis for income tax purposes. Select the appropriate tax treatment on Form 1040 (U.S. Individual Income Tax Return) for qualifying contributions to a simplified employee pension plan. A. Fully deductible on Form 1040 to arrive at adjusted gross income B. Reported in Schedule A—Itemized Deductions (deductibility subject to threshold of 2% of adjusted gross income) Incorrect C. Fully deductible in Schedule C—Profit or Loss From Business D. Partially deductible in Schedule C—Profit or Loss From Business

Qualifying contributions to a simplified employee pension plan (SEP) are fully deductible on Form 1040 to arrive at adjusted gross income. Annual contributions of an employer under a SEP are excluded from the participant's gross income to the extent that they do not exceed the lesser of 25% of the participant's compensation (not exceeding $265,000) or $53,000 for 2016. (IRC Section 408(k)) If the employer exceeds this limit, the participant must withdraw the excess amount before the date for filing his tax return. If he does not, he will be liable for the 6% excise tax on excess contributions.

Danielson invested $2 million in DEC, a qualified small business corporation. Six years later, Danielson sold all of the DEC stock for $16 million and purchased an office building with the proceeds. Danielson had not previously excluded any gain on the sale of small business stock. What is Danielson's taxable gain after the exclusion? A. $0 B. $6 million C. $7 million Incorrect D. $9 million

RC Section 1202 permits a taxpayer, other than a corporation, to exclude in general 50% of the gain realized on the sale of a qualified small business corporation if the taxpayer holds the stock for more than five years prior to sale. The amount of gain which may be excluded in this manner is limited, on a "per issuer" basis, to the greater of $10 million or 10 times the taxpayer's basis in the stock. In this example, 50% of the gain is $7 million ($16,000,000 − $2,000,000 × 0.50). Compare that number to: 10 times the taxpayer's basis, which would be $20 million ($2,000,000 × 10) $10 million The gain excluded is limited to the greater of either (1) or (2) above—in this case, $20 million. However, since the gain exclusion is calculated at $7 million, the limitation is not met and $7 million is excluded.

Define Real Property.

Real property is land and all items permanently affixed to the land.

Related party gain

Recognized only to extent that future sale price > previous relative's cost basis Use relative's basis to determine gain.

Refundable Credits

Reduce tax and get a refund 1. Child tax credit (refund limited) 2. Earned income credit 3. Withholding taxes (W-2) 4. Excess Social Security paid 5. American Opportunity Credit (40% refundable) (Portion of Hope Scholarship paid)

Personal Tax credit (Nonrefundable)

Reduce tax; no refund 1. Child and dependent care credit 2. Elderly and permanently disabled credit 3. Education credits *Lifetime learning credit 4. Retirement savings contribution credit 5. Foreign tax credit 6. General business credit 7. Adoption credit

What does article 9 of the UCC deal with?

Secured Transactions (of personal property or fixtures not security interests in land)

Under the registration requirements of the Securities Act of 1933, which of the following items are considered securities? Incorrect A. Investment contracts B. Collateral-trust certificates C. Both investment contracts and collateral-trust certificates D. Neither investment contracts nor collateral-trust certificates

Registration is required for investment contracts and for collateral-trust certificates. The Securities Act of 1933 prohibits sales of securities to the public (in interstate commerce or through the mails) by an issuer or underwriter without registration with the SEC. A security is defined in the Act as "any note, stock, treasury stock, bond, debenture,...transferable share, investment contract, voting trust certificate...or in general any interest or instrument commonly known as a security..." Collateral-trust certificates are issued by banks or other trustees that hold assets as backing for collateral trust bonds issued by corporations.

What is the Alternative Minimum Tax calculation?

Regular Taxable Income ± Adjustments + Preferences Alternative Minimum Taxable Income (Exemption) Alternative Minimum Tax Base X Tax Computation Tentative Minimum Tax (Tax Credits) Tentative Minimum Tax (Regular Income Tax) Alternative Minimum Tax

Alternative Minimum tax computation

Regular taxable income +/- adjustments + Preferences =Alternative minimum taxable income < Exemption > = Alternative minimum tax base x AMT rate = Tentative AMT tax < AMT foreign tax credit > = Tentative minimum tax < Regular income tax > = Alternative minimum tax All TP except MFS: 26% of first 187800 of taxable excess over alternative minimum taxable income 28% on all taxable excess AMTI exceeding 187800

provides a transaction exemption for issuance of securities made through a crowdfunding process

Regulation Crowdfunding

Basis in distributed property:

Remaining basis in partnership after cash distribution

After serving as an active director of Lee Corp. for 20 years, Ryan was appointed an honorary director with the obligation to attend directors' meetings with no voting power. In 20X1, Ryan received an honorary director's fee of $5,000. This fee is: A. reportable by Lee as employee compensation subject to Social Security tax. B. reportable by Ryan as self-employment income subject to Social Security self-employment tax. Incorrect C. taxable as "other income" by Ryan, not subject to any Social Security tax. D. considered to be a gift not subject to Social Security, self-employment, or income tax.

Ryan is clearly not an employee of Lee Corporation. Thus, Ryan should report the director's fee as self-employment income, having provided professional services for compensation. This income is subject to Social Security self-employment tax as well as personal income tax. (A gift would be a donation out of detached and disinterested generosity which is not the case here.)

Constructive fraud (gross negligence) vs. Actual fraud

Same elements as actual fraud, except instead of intentionally deceiving, acts recklessly (making a statement without knowing whether it is true or false)

Investment interest expense limit

Schedule A miscellaneous itemized deduction subject to 2% of AGI

Matthews was a cash-basis taxpayer whose records showed the following: Year 5 state and local income taxes withheld $1,500 Year 5 state estimated income taxes paid December 30, Year 5 400 Year 5 federal income taxes withheld 2,500 Year 5 state and local income taxes paid April 15, Year 6 300 What total amount was Matthews entitled to claim for taxes on her Year 5 Schedule A of Form 1040? A. $4,700 B. $2,200 Correct C. $1,900 D. $1,500

Schedule A of Form 1040 is the "itemized deductions" schedule for an individual. Since Matthews is a cash-basis taxpayer, she is allowed to deduct all state income taxes paid in Year 5. These are: Year 5 state and local income taxes withheld $1,500 Year 5 state estimated income taxes paid December 30, Year 5 400 Total $1,900 ====== No deduction is allowed for federal income tax. The Year 5 state and local income taxes paid on April 15, Year 6, of $300 can be deducted in Year 6.

S Corp: Losses limited to:

Shareholder's adjusted basis in S corp stock + direct shareholder loans to the corporation Shareholder guarantees don't increase basis Disallowed losses carried forward indefinitely and deductible as shareholder's basis is increased

Evan, an individual, has a 40% interest in EF, an S corporation. At the beginning of the year, Evan's basis in EF was $2,000. During the year, EF distributed $100,000 and reported operating income of $200,000. What amount should Evan include in gross income?

Shareholders of an S corporation include a pro rata share of the corporation's nonseparately and separately stated items of income or expense on their personal tax return. In this case, Evan includes 40% of EF's operating income or $80,000 ($200,000 × 0.40 = $80,000) on his personal tax return. None of Evan's $40,000 share of distributions ($100,000 × 0.40 = $40,000) would be taxable since his distributions did not exceed his new basis. Beginning basis $ 2,000 Share of operating income 80,000 Share of distributions (40,000) -------- Ending basis $42,000 In determining whether a distribution exceeds basis (and is taxable), the ordering rules allow basis to be increased by current-year income before reducing basis by distributions.

Fox, the sole shareholder in Fall, a C corporation, has a tax basis of $60,000. Fall has $40,000 of accumulated positive earnings and profits at the beginning of the year and $10,000 of current positive earnings and profits for the current year. At year-end, Fall distributed land with an adjusted basis of $30,000 and a fair market value (FMV) of $38,000 to Fox. The land has an outstanding mortgage of $3,000 that Fox must assume. What is Fox's tax basis in the land?

Since Fox is the sole stockholder in the C corporation Fall, his basis in the land distributed to him will be equal to the fair market value of the asset ($38,000). The liability Fox assumes reduces the amount of taxable dividend. The amount of taxable dividend Fox received is $35,000 ($38,000 - $3,000).

What is the maximum basic earned income credit allowed?

Single taxpayer - 7.65% of earned income Taxpayer w/ one qualifying child - 34% of earned income Taxpayer w/ two qualifying children - 40% of earned income Taxpayer w/ three or more qualifying children - 45% of earned income

Corporations exempt from AMT

Smaller C corporations: annual average gross receipts from previous 3 periods 7.5 million or less

What is the formula used to determine deductible Casualty and Theft losses?

Smaller Loss 1. Lost cost or Adjusted Basis 2. Decreased FMV (Insurance Recovery) Taxpayer's Loss ($100) Eligible Loss (10% AGI) Deductible Loss Casualty loss must be sudden and/or unexpected. An insurance claim must be filed or the losses are not covered by insurance.

Amount of casualty and theft loss

Smaller loss (Lost cost/adjusted basis, decreased FMV) < Insurance Recovery > = Taxpayer's loss < 100 > = Eligible loss < 10 % AGI > =Deductible Loss

Total of itemized deductions reduced by

Smaller of 80% of itemized deductions affected by limit or 3% of amount by which AGI > 313800 MFJ and SS; 287650 HOH; 261500 Single; 156900 MFS

Smith is a member of the U.S. Armed Forces (an enlisted person) and is assigned to service in Iraq (a designated combat zone) for a period that begins on January 20 of the current year and ends on May 5 of the current year. How many months of military pay may Smith exclude from gross income for the current year?

Smith may exclude from gross income his monthly military pay received for any month or portion of a month that he was a member of the U.S. Armed Forces and serving in Iraq. Thus, even though he only served a portion of a month in both January and May, he may still exclude the full month's pay for these months. Smith may exclude his military pay received from January to May (five months).

Which of the following entities may not deduct fringe benefits for the owner/employee?

Sole proprietors may not deduct the cost of fringe benefits. The entity does not exist separately from the person for taxation purposes. Partnerships may deduct the benefits but the partners must include the amount in income. A more than 2% shareholder of an S corporation must include the benefits in income.

List some taxable miscellaneous income items.

Some miscellaneous taxable items include: Prizes and awards - FMV Gambling winnings and losses - losses to the extent of winnings Business recoveries - if compensation for lost profit Punitive damages - if received in a business context or for loss of personal reputation. Damages received by an individual in a personal injury case are taxable except in wrongful death cases.

Medicaid is a

State run program: Not a social security benefit

Child and dependent care credit:

Taxpayers are permitted a nonrefundable tax credit for expenses incurred in caring for dependents so that the taxpayer(s) may be gainfully employed. a. The credit is available on a three-tiered basis as follows: (1) Taxpayers with an adjusted gross income of $15,000 or less will be entitled to a credit of 35% of dependent care expenses. (2) The credit will be reduced by one percentage point for each $2,000 of adjusted gross income, or fraction thereof, above $15,000. (3) For taxpayers with an adjusted gross income over $43,000, the credit will be 20%. b. The maximum amount of dependent care expenses that may be considered for the credit is $3,000 if there is one qualifying child or dependent and $6,000 if there are two or more qualifying dependents. c. Expenditures for dependent care cannot exceed the earned income of the low-income parent. (1) Special provisions apply where one of the spouses is a full-time student or is incapacitated, and the other spouse works. In this situation, the nonworking spouse is considered to have earned at least $250 per month, where one dependent requires care and $500 per month, where more than one dependent requires care. d. The dependent must be: (1) a child under age 13 or (2) an incapacitated dependent or spouse. e. Married taxpayers must file a joint return unless they live apart for the last six months of the year. (1) For divorced or separated parents, the credit is available to the parent having custody of the child for the longer period. (2) A custodial parent may claim the credit even though the child may not qualify as a dependent. However, two taxpayers filing separate returns cannot claim separate credits for the same child. f. Expenditures that qualify for the credit include amounts paid for both in-the-home care and out-of-the-home care. (1) In-the-home care may include expenditures for household services if they were partly for the well-being and protection of a qualifying individual. (2) Expenditures for out-of-the-home care are eligible for the credit only if incurred for: (a) a dependent under age 13 or (b) any other qualifying person who regularly spends at least eight hours each day in the taxpayer's household. g. Expenditures do not qualify for the credit if they were made to: (1) a relative who is a dependent of the taxpayer or (2) the taxpayer's child who is under age 19.

The Browns borrowed $20,000, secured by their home, to pay their son's college tuition. At the time of the loan, the fair market value of their home was $400,000, and it was unencumbered by other debt. The interest on the loan qualifies as:

The $20,000 loan secured by the taxpayers' home but used for reasons other than to purchase or substantially enhance the residence is considered a home equity loan. The interest on the home equity loan is deductible qualified residence interest and can be deducted as mortgage interest expense on the taxpayers' Schedule A (IRS Form 1040)

When verifying a client's compliance with statutes governing employees' wages and hours, an auditor should check the client's personnel records against relevant provisions of which of the following statutes?

The *Fair Labor Standards Act* (FLSA) covers employers engaged in interstate commerce. It is also known as the Wage-Hour Law. Willful violations of the FLSA by employers can result in civil as well as criminal penalties. The NLRA established the National Labor Relations Board (NLRB), which has two major functions: to oversee union elections, and to determine if an employer's actions constitute an unfair labor practice (usually discrimination). The Taft-Hartley Act (the Labor Management Relations Act) established certain acts by labor unions as unfair labor practices. The Americans with Disabilities Act (ADA) prohibits discrimination in employment in such areas as hiring, firing, promotion, and pay against qualified persons with a disability.

What is the Alternative Minimum Tax Exemption?

The AMT Exemption amount is: Single: $33,750 MFJ: $45,000 MFS: $22,500 The exemption is reduced by 25% of excess AMTI over: Single: $112,500 MFJ: $150,000 MFS: $75,000

What is the American Opportunity Credit?

The American opportunity credit is available against federal taxes for qualified tuition, fees, and course materials paid for the student's first four years of post secondary education. The credit is equal to: 100% of the first $2,000 of qualified expenses plus 25% of the next $2,000 of expenses paid The student must be at least half-time NOT available for convicted felony drug offenses

"Abuse" under the 2005 Bankruptcy Reform Act as relating to the debtor's family income (i.e., if a level of income is exceeded, the trustee or interested party may move to dismiss the petition of Chapter 7 bankruptcy) includes elements of the following: The debtor's state median family income levels Six months' average income of the debtor's family, as adjusted for certain allowed or excluded expenses, multiplied by 12 If a calculation results in an income exceeding the state median income by at least $6,000, abuse is presumed. The presumption of abuse resulting from an income level over a certain amount is not rebuttable. A. I, II, and IV B. I and III Correct C. I, II, and III D. I, III, and IV

The Bankruptcy Code looks to the debtor's state median income level, not to that of other states. The code also looks at an adjusted monthly income level multiplied by 12 (i.e., an annualized amount basis). The code requires an annual income level exceeding the state median income level by $6,000 to presume abuse. (This income level is rebuttable.) The 2005 Bankruptcy Reform Act includes elements of a presumed, but rebuttable, income level that indicates the debtor is intending to use the bankruptcy process as a means to avoid legitimate debts in what would be considered an abuse of the Bankruptcy Code.

The purpose of the Clean Water Act is to eliminate the discharge of pollutants into the navigable waters of the United States. The term "navigable water" includes:

The Clean Water Act defines "navigable waters" as all waters of the United States that are used in *interstate* commerce. The definition also includes all *freshwater* wetlands that are adjacent to all covered waterways.

Which of the following employee benefits is exempt from the provisions of the National Labor Relations Act? A. Sick pay B. Vacation pay Incorrect C. Both sick pay and vacation pay D. Neither sick pay nor vacation pay

The National Labor Relations Act, also known as the Wagner Act (1935), and the Labor Management Relations Act, also known as the Taft-Hartley Act (1947), established the U.S. government's policy of protecting the right of workers to unionize and to bargain collectively with the employer. The Act requires the employer to bargain in good faith with regard to wages, hours, and other terms and conditions of employment. Neither sick pay nor vacation pay issues are exempt from the provisions of the Act.

Frey Corp. has 1,000 shares of issued and outstanding common stock. Frey's articles of incorporation permit a stockholder who owns 5% or more of the outstanding stock or who has owned the stock for longer than six months to inspect Frey's books and records. Ace, who has owned 25 shares of Frey stock for four months, wants to inspect the books and records. Under the Revised Model Business Corporation Act, which of the following statements is correct regarding Ace's right to inspect the books and records? A. Ace must wait two months before being allowed to inspect the books and records. Incorrect B. Ace must purchase an additional 25 shares of Frey stock before being allowed to inspect the books and records. C. Ace may, after giving five days' written notice, inspect the books and records to determine the value of Frey stock. D. Ace may, after giving five days' written notice, inspect the books and records to provide a list of Frey stockholders to Ace's broker.

The Revised Model Business Corporation Act gives stockholders the right to inspect the books and records with five days' written notice. Ace does not need to wait two months or purchase an additional 25 shares of stock, because the rights given by law (the Revised Model) are what Ace should follow, not the articles of incorporation. Ace may not provide a list of Frey stockholders as this is not allowed.

Under the Securities Exchange Act of 1934, a corporation with common stock listed on a national stock exchange: A. is prohibited from making private placement offerings. B. is subject to having the registration of its securities suspended or revoked. C. must submit Form 10-K to the SEC except in those years in which the corporation has made a public offering. Incorrect D. must distribute copies of Form 10-K to its stockholders.

The Securities Exchange Commission (SEC) may suspend trading in a stock if they believe it is required to protect investors and the public interest. Circumstances that could lead to suspension include the following: A lack of current, accurate, or adequate information about the company (e.g., when a company is not current in its filings of periodic reports) Questions about the accuracy of publicly available information, including in company press releases and reports, about the company's current operational status, financial condition, or business transactions Questions about trading in the stock, including trading by insiders, potential market manipulation, and the ability to clear and settle transactions in the stock A corporation with common stock listed on a national stock exchange is not prohibited from making private placement offerings, must submit Form 10-K to the SEC every year, and does not have to submit copies of Form 10-K to its stockholders.

What is the tax consequence when a corporation cancels a shareholder's debt without repayment by the shareholder?

The amount canceled is treated as a distribution to the shareholder if the corporation cancels the shareholder's debt without repayment by the shareholder. The distribution would be taxable

Dove and Eagle formed a business entity in which they are equal owners. Dove contributed cash of $100,000, and Eagle contributed land with a basis of $40,000 and fair market value of $100,000. For its first year of operations, the entity had taxable income of $60,000 and made no distributions. At year-end it had outstanding recourse liabilities to third parties of $10,000. Eagle had a basis of $70,000 in the entity at the end of the first year of operations. What type of entity was formed?

The basis of an S corporation shareholder's stock (generally, its cost) is adjusted as follows and, except as noted, in the order listed. In addition, basis may be adjusted under other provisions of the Internal Revenue Code: Basis is increased by (a) all income (including tax-exempt income) reported on Schedule K-1 and (b) the excess of the deduction for depletion (other than oil and gas depletion) over the basis of the property subject to depletion. Basis is decreased by property distributions (including cash) made by the corporation (excluding dividend distributions reported on IRS Form 1099-DIV and distributions in excess of basis) reported on IRS Schedule K-1, box 16, code D, minus the amount of such distributions in excess of the basis of the shareholder's stock. Basis is decreased by (a) nondeductible expenses and (b) the depletion deduction for any oil and gas property held by the corporation, but only to the extent the shareholder's pro rata share of the property's adjusted basis exceeds that deduction. Basis is decreased by all deductible losses and deductions reported on Schedule K-1. Note that the taxpayer can make an election to deduct items in (4) in the list above before nondeductible items in (3). In this case, *Eagle's basis at the end of the year in the S corporation* is $70,000, which is the $40,000 for his basis of the contributed land plus his share of the taxable income for the first year of operations of $30,000 (50% share of the S corporation's $60,000 income). If *the entity was a C corporation*, its taxable income of $60,000 would not increase Eagle's basis in his stock, so it cannot be a C corporation. If *the entity was a general partnership*, $5,000 (50% share of the general partnership's $10,000 of recourse liabilities) would have been added to Eagle's basis. The partner's contributions to the partnership and the increased share of, or assumption of, partnership liabilities increase a partner's basis. Thus, it cannot be a general partnership. The entity choice of a limited liability company (LLC) does not provide enough information. There is uncertainty as to its tax treatment as a corporation or a partnership since there are two equal owners, Dove and Eagle, and they could choose to be either a partnership or corporation. Thus, as first described above, the only entity that could result in an end-of-year basis for Eagle would be the S corporation.

Which group has ultimate responsibility for ensuring the effective operation of an entity's Environmental Management System?

The board of directors has final accountability to the outside world. Therefore, it has ultimate responsibility for ensuring the effective operation of an entity's Environmental Management System

Smith was an officer of CCC Corp. As an officer, the business judgment rule applied to Smith in which of the following ways?

The business judgment rule protects the directors from shareholder lawsuits alleging a lack of due care on the part of the directors in carrying out the corporation's business. This rule applies: when the board makes an informed decision, when there is no conflict of interest, and when there is a rational basis for the board's decision. Therefore, if Smith makes an informed, rational decision that has no conflict of interest, he is not liable for damages caused. An officer is hired by the board of directors, and the ordinary rules of agency apply.

What are the limitations of the Earned Income credit?

The credit is limited to those taxpayers that: Lives in the US for more than half the taxable year, Meest certain low-income thresholds, Not have more than a specified amount of disqualified income, Be over the age of 25 and less than 65 if there are no qualifying children, and File a joint return with spouse with certain exceptions.

What is the AMT Exemption and phaseout for corporations?

The exemption is $40,000, which must be reduced by 25% of the excess of AMTI over $150,000. Assume that the Brady Corporation had the following tax situation in Year 4: Sum of AMT preference items $ 40,000 Total positive AMT adjustments 85,000 Total negative AMT adjustments 15,000 Regular taxable income 100,000 Determine the amount of AMT exemption that will apply to the Brady Corporation. Start with: Regular Taxable Income $100,000 Plus: AMT Preference Items 40,000 Plus/Minus: AMT Adjustments 70,000 Equals: AMTI $210,000 AMTI $210,000 Phaseout threshold amount (150,000) Excess $ 60,000 x .25 Reduction in exemption $ 15,000 ========= Allowable exemption: $40,000 − $15,000 = $25,000

Under the Secured Transactions Article of the U.C.C., which of the following statements is correct regarding the filing of a financing statement? A financing statement must be filed before attachment of the security interest can occur. Once filed, a financing statement is effective for an indefinite period of time provided continuation statements are timely filed. A. I only Correct B. II only C. Both I and II D. Neither I nor II

The filing of a financing statement only gives public notice of the security interest. A financing statement is effective for five years, but can be continued with a continuation statement filing. Since a continuation statement can be filed indefinitely, this makes a financing statement effective indefinitely.

Wine purchased a computer using the proceeds of a loan from MJC Finance Company. Wine gave MJC a security interest in the computer. Wine executed a security agreement and financing statement, which was filed by MJC. Wine used the computer to monitor Wine's personal investments. Later, Wine sold the computer to Jacobs, for Jacobs's family use. Jacobs was unaware of MJC's security interest. Wine now is in default under the MJC loan. May MJC repossess the computer from Jacobs? Incorrect A. No, because Jacobs was unaware of the MJC security interest B. No, because Jacobs intended to use the computer for family or household purposes C. Yes, because MJC's security interest was perfected before Jacobs's purchase D. Yes, because Jacobs's purchase of the computer made Jacobs personally liable to MJC

The filing of the financing statement is to give public notice of MJC's security interest. Third parties are deemed to know of the security interest even if they do not actually have knowledge. MJC may repossess the computer. Jacobs's intended use is of no consequence. Jacobs has no personal liability.

List the types of Treasury and Capital Stock transactions exempt from gain tax.

The following corporate transactions are exempt from gain: Sales of stock by a corporation Repurchase of stock by a corporation Reissue of stock

What types of distributions are considered to be tax-free?

The following items are exempt from gross income and are not taxable: Return of capital - If company distributes funds but has no earnings or profits, the funds are not taxable. The taxpayer will reduce their basis in the stock held. Stock split - the shareholder will allocate the original basis over the total number of shares held after the split. Stock dividend - the basis of the shares after distribution depends on the stock received unless the option to receive cash or property is given. If the same stock is received, the original basis is dividend by total shares. Different stock received - basis is allocated based upon FMV of stock. Life insurance Dividend

What general rules apply to IRA income?

The general rules for IRA income are: Amounts cannot be withdrawn until the individual reaches the age of 59½ A taxpayer is required to start withdrawals by the age of 70½ Benefits are not taxable until they are withdrawn

Discuss Installment Sales.

The installment method is the method of reporting gains of sales made by a "nonmerchant" in personal property and "nondealer" in real estate. Revenue is reported over the period in which the cash payments are received. The method does not alter the type of gain to be reported. Reportable Installment Sale Gain/Income: Gross profit = Sale - Cost of goods sold Gross profit percentage = Gross profit/Sales price Earned Revenue(taxable income) = Cash collections X Gross Profit All depreciation recaptured shall be reported in income in the year of sale. Compute the gross profit percentage from the original sale and apply the profit percentage to cash received in the year to recognize realized profit for the year.

What are the limits on Nondeductible IRAs?

The less of: $5,000 or Individual compensation Contributions are limited to the amount of contributions not already made to other IRAs Earnings will accumulate tax-free until withdrawn.

Tom Lewis, a single taxpayer, received a $1,500 state tax refund in Year 5 for the prior-year overpayments. In Year 4, Tom was not subject to the alternative minimum tax and was not entitled to any credit against income tax. Tom's Year 4 taxable income was $40,000. Tom's total itemized deductions for Year 4 amounted to $6,400, which included $3,000 for state income tax withholding. Tom's standard deduction amount for Year 4 was $6,200. How much should Tom include in gross income for Year 5 as a result of the state tax refund?

The receipt of an amount that has formed the basis for an earlier year deduction or credit is considered a "recovery" and generally must be included (either partially or totally) in income in the year in which it is received (IRC Section 111). Recoveries of amounts that could be claimed as an itemized deduction (as in this question, a refund of state income taxes), are not included in income if the taxpayer did not itemize his or her deductions in the year in which the deduction could have been taken. Recoveries must be included (in full) in gross income if the recovery amount (i.e., the state tax refund amount as in this question) is equal to or less than the amount by which the taxpayer's itemized deduction amount exceeded his or her standard deduction amount in the prior year (assuming that the taxpayer had positive taxable income in the prior year). If the recovery amount exceeds the difference between the taxpayer's total itemized deduction amount and the standard deduction amount, only that smaller difference amount will be included in income. This is known as the "Tax Benefit" rule. In this case, Tom's total itemized deduction amount ($6,400) exceeded his standard deduction amount ($6,200) by only $200. Thus, only $200 of the $1,500 state refund amount will be included in income in the current year.

IRC Section 267 has a special rule for sales to a related party that are unpaid at the end of the year. In which of the following cases does the rule apply? A. When a cash-basis seller sells to a cash-basis buyer B. When an accrual-basis seller sells to an accrual-basis buyer C. When a cash-basis seller sells to an accrual-basis buyer Incorrect D. When an accrual-basis seller sells to a cash-basis buyer

The rule applies when the seller is on the cash basis and the buyer is on the accrual basis. The buyer may not deduct the expense until the seller has reported the income.

When a partner in a general partnership lacks actual or apparent authority to contract on behalf of the partnership, and the party contracted with is aware of this fact, the partnership will be bound by the contract if the other partners: Correct A. ratify the contract. B. amend the partnership agreement. C. ratify the contract or amend the partnership agreement. D. None of the answer choices are correct.

This question requires particularly careful reading. While normally the outside world need not be concerned with whether a general partner has the actual authority to bind the partnership (due to apparent authority), the facts of the question indicate that the other party is aware of this limitation. Usually in this situation only the person who contracted would be bound, not the partnership. Notice that normally a partner has the apparent authority to deal with outside parties, but this fact scenario calls for a different result as the facts note that the other party is aware of the partner's limitation. If the partnership determines it is in the best interest of the partnership to enter into the contract, the partners are free to ratify the contract, which at that time makes the contract a partnership obligation. Note carefully that the partnership agreement may be amended (assuming that the partners so agree) to allow the partner in question to have the actual authority to contract in the future, but it would not have any effect on the existing contract. The partners would still have to ratify this particular contract to make it binding on the partnership.

Special (lower) tax rate for Qualified Dividends What are qualified dividends?

To be qualified, stock must be held for more than 60 days during the 120 day period that begins 60 days before the ex dividend date (date on which a purchased share no longer is entitled to any recently declared dividends)

How do you determine if dividends received are taxable?

To determine if a dividend is taxable, the source of the dividend must be determined. There are four sources: Earnings & profits/Current Taxable dividend Earnings & profits/Accumulated Taxable dividend Return of capital = no earnings & profits Not taxable Capital gain distributions - no E&P/No basis Taxable at a favorable rate of 15% To qualify for the favorable tax rate, the stock must be held for more than 60 days.

Nolan agreed orally with Train to sell Train a house for $100,000. Train sent Nolan a signed agreement and a down payment of $10,000. Nolan did not sign the agreement, but allowed Train to move into the house. Before closing, Nolan refused to go through with the sale. Train sued Nolan to compel specific performance. Under the provisions of the statute of frauds: A. Train will win because Train signed the agreement and Nolan did not object. B. Train will win because Train made a down payment and took possession. Incorrect C. Nolan will win because Nolan did not sign the agreement. D. Nolan will win because the house was worth more than $500.

Train will win because Train made a down payment and took possession. While the statute of frauds requires a writing concerning the sale or purchase of real property, if performance has taken place and there had been acquiescence, then a contract will be enforced. This is exactly the case with Train. Train offered, and Nolan accepted, a $10,000 deposit and permitted Train to take possession of the property. The mere signing of a contract by one party does not result in a "meeting of the minds," even if the other party does not object. However, the fact that Nolan did not sign the contract will not result in his being able to evict Train because Nolan did accept the down payment and did permit Train to take possession of the property. Since this transaction does not involve the sale of goods, the Uniform Commercial Code's Statute of Frauds does not apply.

Effect of a partner transferring his interest in the partnership without the consent of the other partners

Transfer doesn't make the asignee a partner (can only be done with consent of all partners). Transferee has no power to manage partnership, inspect books and records, vote, etc. Assignee's only right is to get whatever distribution the assignor would've gotten. Creditor with a charging order and an heir who receives a deceased partner's interest: same thing

Estate tax is what kind of tax?

Transfer tax, not income tax. Imposed on value of property transferred by the decedent at death.

Moving Expense deduction

Work related 1. New workplace 50 miles farther from old house than old workplace was 2. Employee must work full time in new location at least 39 weeks during 12 month period following arrival. Self employed: 78 weeks during 24 month period after arrival

state run programs designed to enable employees to recover for injuries incurred while on the job

Worker's Compensation

Express warranties

Will arise from any statement of fact or promise made by the seller, any description of goods made by the seller, or any sample model shown by the seller. Express warranty is that goods will conform to the statement of fact. Can be oral or written; no statements of value or opinion UCC requires the express warranty to be PART OF THE BASIS OF THE BARGAIN (played part in buyer's decision to buy)

With regard to consolidated tax returns, which of the following statements is correct? A. Operating losses of one group member may be used to offset operating profits of the other members included in the consolidated return. Incorrect B. Only corporations that issue their audited financial statements on a consolidated basis may file consolidated returns. C. Of all intercompany dividends paid by the subsidiaries to the parent, 70% are excludable from taxable income on the consolidated return. D. The common parent must directly own 51% or more of the total voting power of all corporations included in the consolidated return.

With regard to consolidated tax returns, operating losses of one group member may be used to offset operating profits of the other members included in the consolidated return. What is the reason to elect to file a consolidated corporate tax return? Generally, the combined group will reduce the net taxable income and the tax due to a minimum by taking full advantage of the losses reported by some group members by offsetting the profits reported by other members of the group.

What are the requirements of the employment credits?

Withholding: All income taxes withheld from a taxpayer's paycheck are treated as a "credit" against their tax liability. Excess FICA: A credit may be claimed against income tax, if that excess resulted from correct withholding by two or more employers. If withheld by only one employer the employer must refund the excess to the employee.

World Corp. wanted to make a public offering of its common stock. On May 10, World prepared and filed a registration statement with the SEC. On May 20, World placed a "tombstone ad" announcing that it was making a public offering. On May 25, World issued a preliminary prospectus, and the registration statement became effective on May 30. On what date may World first make oral offers to sell the shares?

World may *first make oral offers* to sell the shares on *May 10*. Under the Securities Act of 1933 and related rules and regulations promulgated by the U.S. Securities and Exchange Commission, *once a company has filed a registration statement, it may make oral offers to sell the shares*. The *actual sale* of shares may not occur until the registration has *"gone effective,"* which is usually 20 days after the filing unless the SEC has issued a "comment" letter that requires changes

if a principal is either partially disclosed or undisclosed, is an agent personally liable for contracts entered into?

YES

Deduct when? Cash basis

Year item is paid or charged. Not in the year the tax is applicable

if you inherit property and the donor's basis is higher than FMV, how is it treated when you sell the asset for tax purposes?

You assume the lower FMV as your basis for computing losses, and you use the donor's higher basis for computing gainsIf you sell it at a price in between the gain and loss basis, no gain or loss is recognized

Under the U.C.C. Sales Article, the implied warranty of merchantability: A. may be disclaimed by a seller's oral statement that mentions merchantability. Incorrect B. arises only in contracts involving a merchant seller and a merchant buyer. C. is breached if the goods are not fit for all purposes for which the buyer intends to use the goods. D. must be part of the basis of the bargain to be binding on the seller.

all purposes for which the buyer intends to use the goods. The implied warranty of merchantability does not have to be part of the basis of the bargain to be binding on the seller. Remember that the term "implied" means that the warranty arises as a matter of law—the parties do not have to expressly discuss whether the good is "merchantable" for the warranty to arise.

Any variation in a contract that changes a _____________ surety's risk (i.e. extension of time) will discharge the surety only if the change is material and it increases the surety's risk

compensated

______ notice must be given by principal to potential customers to terminate apparent authority after end of agency relationship

constructive

executory contract

contract not fully performed on both sides

what right does one cosurety have against another cosurety?

contribution. this means that if one surety pays more than his fair share, he can recover from the other surety

when doing a life insurance as a fringe benefit question, the key thing to remember is that the first 50,000 of ______ is _____.

coverage is free. if it has 2.76 per $1000 of coverage, only apply it to the amount higher than 50k

What are two options a debtor has to alleviate debt outside of suretyship besides for bankruptcy?

creditor's composition; assignment for the benefit of creditors

In a general partnership, a partner's interest in specific partnership property is

subject to a surviving partner's right of survivorship. In the event a partner dies, his partnership interest becomes part of his estate. However, the deceased partner's interest in specific partnership property does not result in his estate having a claim on that property. The property stays in the partnership.


Kaugnay na mga set ng pag-aaral

geg 101 - Solar Energy and the Greenhouse Effect

View Set

Chapter 28 North Korea and South Korea quiz

View Set

Intro to Front End Development Questions

View Set

Chapter 29: Contraception and Unplanned Pregnancies

View Set

The Hound of the Baskervilles - El sabueso de los Baskervilles (A.C.Doyle)

View Set

Impacts of global change on terrestrial and aquatic ecosystems

View Set

History Chapter Nine Sections 1-2 Test

View Set